■ちょっとした物理の質問はここに書いてね143■

このエントリーをはてなブックマークに追加
1ご冗談でしょう?名無しさん
前スレ
■ちょっとした物理の質問はここに書いてね142■
http://kamome.2ch.net/test/read.cgi/sci/1302006367/

★荒らし厳禁、煽りは黙殺
★書き込む前に>>2の注意事項を読んでね
★数式の書き方(参考)はこちら>>3-5(予備リンク:>>2-10

===質問者へ===
重要 【 丸 投 げ 禁 止 】
丸投げは専用スレ「丸投げしたい問題を書くスレ」に
http://kamome.2ch.net/test/read.cgi/sci/1284096345/

・質問する前に
1. 教科書や参考書をよく読む
2. http://www.google.com/ などの検索サイトを利用し、各自で調べる
3. 学生は自分の学年、物理科目の履修具合を書く
4. 宿題を聞くときは、どこまでやってみてどこが分からないのかを書く
5. 投稿する前に、ちゃんと質問が意味の通る日本語か推敲する、曖昧な質問文には曖昧な回答しか返せない
・「力」「エネルギー」「仕事」のような単語は物理では意味がはっきり定義された言葉です、むやみに使うと混乱の元
・質問に対する回答には返答してね、感謝だけでなく「分からん」とかダメ出しでもOK
・質問するときはage&ID表示推奨
・高度すぎる質問には住人は回答できないかもしれないけれど、了承の上での質問なら大歓迎

===回答者へ===
・丸投げは専用スレに誘導
・不快な質問は無視、構った方が負け
・質問者の理解度に応じた適切な回答をよろしく
・単発質問スレを発見したらこのスレッドへの誘導をよろしくね
・逆に議論が深まりそうなら新スレ立てて移動するのもあり
・板違いの質問は適切な板に誘導を
・不適切な回答は適宜訂正、名回答は素直に賞賛
2ご冗談でしょう?名無しさん:2011/04/30(土) 22:56:56.22 ID:???
書き込む際の注意

1.) 以下のような質問に物理板住人は飽き飽きしているのでたぶん無視されます。
しないでください。
「相対性理論は間違っています」「量子力学は間違っています」
「宇宙論は間違っています」「シュレディンガーの猫は変です」
「永久機関を作りました」「タイムマシンについて教えて」

「どうして〜?・なぜ〜?」:物理で答えられる問題とは限りません。
質問によっては哲学板・雑談系板へ誘導されるかも。
意図的になされた物理と関係ない質問:スルーの方向で

参考サイト
Wikipedia http://ja.wikipedia.org/
物理のかぎしっぽ http://hooktail.sub.jp/
EMANの物理学 http://homepage2.nifty.com/eman/
ときわ台学 http://www.f-denshi.com/
以上のサイトの説明はすべてが正確なわけではありません。
このスレでの受け答えもそうですが。相互に補完しつつ精度を高めましょう。
3ご冗談でしょう?名無しさん:2011/04/30(土) 22:57:06.43 ID:???
数式の書き方 ※適切にスペースを入れると読みやすくなります
●括弧: (), [], {}を適切に入れ子にして分かりやすく書く
●スカラー: a,b,...,z, A,...,Z, α,β,...,ω, Α,Β,...,Ω,...(「ぎりしゃ」「あるふぁ〜おめが」で変換)
●ベクトル: V=(v1,v2,...), |V>,V↑, (混同しないならスカラーの記号でいい。通常は縦ベクトル)
●テンソル: T^[i,j,k...]_[p,q,r,...], T[i,j,k,...; p,q,r,...]  (上下付き1成分表示)
●行列: M[i,j], I[i,j]=δ_[i,j] M = [[M[1,1],M[2,1],...], [M[1,2],M[2,2],...],...], I = [[1,0,0,...],[0,1,0,...],...]
 (右は全成分表示。行または列ごとに表示する。例:M=[[1,-1],[3,2]])
●対角行列: diag(a,b) = [[a,0],[0,b]]
●転置行列・随伴行列:M^T, M†("†"は「だがー」で変換可) ●行列式・トレース:|A|=det(A), tr(A)
●複号: a±b("±"は「きごう」で変換可)
●内積・外積: a・b, a×b
●関数・汎関数・数列: f(x), F[x(t)] {a_n}
●平方根: √(a+b) = (a+b)^(1/2) = sqrt(a+b) ("√"は「るーと」で変換可)
●指数関数・対数関数: exp(x+y)=e^(x+y) ln(x)=log_e(x) (底を省略して単にlogと書いたとき多くは自然対数)
 括弧を省略しても意味が容易に分かるときは省略可: sin(x) = sin x
●三角関数、逆三角関数、双曲線関数: sin(a), cos(x+y), tan(x/2), asin(x)=sin^[-1](x), cosh(x)=[e^x+e^(-x)]/2
●絶対値:|x| ●ノルム:||x|| ●共役複素数:z^* = conj(z)
●階乗:n!=n*(n-1)*(n-2)*...*2*1, n!!=n*(n-2)*(n-4)*...
4ご冗談でしょう?名無しさん:2011/04/30(土) 22:57:29.85 ID:???
●微分・偏微分: dy/dx=y', ∂y/∂x=y_x ("∂"は「きごう」で変換可)
●ベクトル微分: ∇f=grad f, ∇・A=div A,∇xA=rot A, (∇^2)f=Δf ("∇"は「きごう」,"Δ"は「でるた」で変換可.)
●積分: ∫[0,1] f(x)dx = F(x)|_[x=0,1], ∫[y=0,x] f(x,y)dy, ∬[D] f(x,y)dxdy, ∬[C] f(r)dl 
("∫"は「いんてぐらる」,"∬"は「きごう」で変換可)
●数列和・数列積: Σ[k=1,n] a(k), Π[k=1,n] a(k) ("Σ"は「しぐま」,"Π"は「ぱい」で変換可)
文脈によっては単に同じ添字が2回出てきただけで a_i b_i = Σ[i] a_i b_i と積の総和をとることも(Einsteinの縮約)
●極限: lim[x→∞] f(x) ("∞"は「むげんだい」で変換可)
●論理・集合: "⇔⇒∀∃∧∨¬∈∋⊆⊇⊂⊃∪∩"は「きごう」で変換
●等号・不等号: "≠≒<>≦≧≪≫"は「きごう」で変換

読みやすい書き方の例:∫[-∞,∞] exp{ -Σ[i,j=1,n] A_[i,j](x_i)(x_j) } dx = √(π^n/det A)

読みにくい書き方の例:∫[-∞,∞]exp(-Σ[i,j=1,n]A_[i,j]x_ix_j)dx=√(π^n/det A)
5ご冗談でしょう?名無しさん:2011/04/30(土) 22:57:36.70 ID:???
a:加速度、昇降演算子 A:振幅、ベクトルポテンシャル B:磁束密度 c:光速 C:定数、熱・電気容量
d:次元、深さ D:領域、電束密度 e:自然対数の底、素電荷 E:エネルギー、電場
f:周波数 f,F:力 F:Helmholtzエネルギー g:重力加速度、伝導度
G:万有引力定数、Gibbsエネルギー、重心 h:高さ、プランク定数 H:エンタルピー、Hamiltonian、磁場
i:虚数単位 i,j,k,l,m:整数のインデックス I:電流、慣性モーメント j:電流密度・流束密度
J:グランドポテンシャル、一般の角運動量 k:バネ定数、波数、Boltzmann定数 K:運動エネルギー
l,L:長さ L:Lagrangian、角運動量、インダクタンス m,M:質量 n:物質量 N:個数、トルク
M:磁化 O:原点 p:双極子モーメント p,P:運動量、圧力 P:分極 q:波数
q,Q:一般化座標、電荷 Q:熱 r:距離 R:抵抗、気体定数 S:エントロピー、面積 t:時間 T:温度
U:ポテンシャル、内部エネルギー v:速度 V:体積、ポテンシャル、電位
W:仕事、状態数 x,y,z:変数、位置 z:複素変数 Z:分配関数

β:逆温度 γ:抵抗係数 Γ:ガンマ関数 δ:微小変化 Δ:変化 ε:微小量、誘電率 θ:角度
λ:波長 μ:換算質量、化学ポテンシャル、透磁率 ν:周波数 π:円周率 ρ:(電荷)密度、抵抗率
σ:スピン φ:角度、ポテンシャル、波動関数 ψ:波動関数 ω:角振動数 Ω:状態密度
6ご冗談でしょう?名無しさん:2011/04/30(土) 23:00:31.34 ID:???
NEO MEKOSUJIO
7ご冗談でしょう?名無しさん:2011/04/30(土) 23:30:40.75 ID:/GxoeQ7P
コイルガンを作ったときの話です。


1. [コンデンサ陽極]−−[サイリスタ]−−[コイル]−−[コンデンサ陰極]

と回路を作ったとき、サイリスタは壊れてしまったんですが、


2. [コンデンサ陽極]−−[コイル]−−[サイリスタ]−−[コンデンサ陰極]

この時はうまくいきました。

なぜ1.の時壊れてしまったのでしょうか?
8ご冗談でしょう?名無しさん:2011/05/01(日) 02:45:25.94 ID:???
宇宙の膨張の話で、遠い星ほど速く遠ざかっている
という事について質問なんですが、
話を簡単にするために、宇宙を1次元と考えます。
星A、B、Cを点と考えます。
僕らはAに住んでいて、わりと近くにBがあって、遠くにCがあります。
Aから見たら、BよりCのほうが速く遠ざかっているので加速度が生じているように見えます。
しかし、宇宙自体は(つまり直線は)任意の点で等速直線運動をしているにすぎないので
つまり、慣性の法則で、同じ速度で膨張しているにすぎない、
つまり加速度は生じていないようにも思えます。

いったい、宇宙膨張に加速度は(あるいは力は)生じているのでしょうか?
9ご冗談でしょう?名無しさん:2011/05/01(日) 02:47:16.84 ID:???
>宇宙自体は(つまり直線は)任意の点で等速直線運動をしているにすぎないので

宇宙自体は(つまり直線は)任意の点で等速直線「膨張」をしているにすぎないので
に訂正させてください。
10ご冗談でしょう?名無しさん:2011/05/01(日) 08:12:22.40 ID:???
>>7
回路図には見えませんがサイリスタのゲートに加わる電位が1と2で異なると
思います。
カソードとゲート間の最大定格条件を確認してみてください。
11ご冗談でしょう?名無しさん:2011/05/01(日) 09:30:03.31 ID:???
>>8
遠い星ほど速く離れる(等速)
遠ざかるほど速く離れる(加速)
の違いかと思いますが?
12ご冗談でしょう?名無しさん:2011/05/01(日) 09:50:13.08 ID:???
星A、B、Cが最初同一点にあり、星Aに対してBは 0.5 の速さで、Cは 1 の速さで離れ始める
最初から速いヤツは速いし、遅いヤツは遅く移動し続けて、それぞれ等速度運動している
その結果が今の状態
13ご冗談でしょう?名無しさん:2011/05/01(日) 10:42:22.70 ID:???
>>12
いや違うでしょ
Bが0.5の速さで移動して、Aから見た場合の現在のCの位置に達したら、
0.5ではなくて当然1の速さで移動しなくちゃならない(宇宙の膨張率が一定の場合)
だから、遠くの星は物凄く長い時間観測を続けていると、加速度運動しているように見える

>>8
「同じ速度で膨張しているにすぎない、」と「加速度運動しているように見える」は矛盾しないと思うけどな
14ご冗談でしょう?名無しさん:2011/05/01(日) 15:24:42.75 ID:YQsn3XiE
How fast has a sphere of diameter D to be moved to pass through a
circular hole (diameter d) in a plane sheet of paper?
という問題を考えているのですが、意味がわかりません。
半径Dの球が、平らな紙の半径dの円孔を通過するときどのくらいの速さになるでしょう?ということでしょうか?
15ご冗談でしょう?名無しさん:2011/05/01(日) 15:25:40.04 ID:???
>8
>Aから見たら、BよりCのほうが速く遠ざかっているので加速度が生じているように見えます。
>しかし、宇宙自体は(つまり直線は)任意の点で等速直線運動をしているにすぎないので
>つまり、慣性の法則で、同じ速度で膨張しているにすぎない
・・・
これが何言っているのか分からなかったんだけど
遠くほど膨張する速度が速くなっているから、時間が経つほど
離れる速度が速くなる=加速すると考えられる。
でも遠くの宇宙は過去の世界だからビッグバンの当初の
速度が速くて、今が遅くなったと考えても良いかもしれない。
光学的に見ると遠くの銀河ほど遠ざかる速度が速いということだけははっきりしている。
16ご冗談でしょう?名無しさん:2011/05/01(日) 15:36:57.09 ID:???
電磁波の波長や振動数ってどうやって計るんでしょうか?
例えば可視光って見えないぐらい小さい、見えたとしても比べるものがないような
大きさだし、振動数だって一秒間に1E+15とかカウント不能ではりませんか?
17ご冗談でしょう?名無しさん:2011/05/01(日) 16:11:54.16 ID:4DSzQuOk
色だ。

職人をつれてきて波長を判断してもらうんだよ
18ご冗談でしょう?名無しさん:2011/05/01(日) 16:40:07.23 ID:???
>>17
職人は色と波長をどう結びつけるんですか?
19ご冗談でしょう?名無しさん:2011/05/01(日) 17:38:57.15 ID:???
>>16
単色光の波長を干渉計で測る。
プリズムで分光した光と既知の光を比較する。
回折格子の格子間隔と回折角から波長を計算して求める。
など
>>18
職人の勘
20ご冗談でしょう?名無しさん:2011/05/01(日) 18:02:36.12 ID:???
>>19
ありがとうございます。
21ご冗談でしょう?名無しさん:2011/05/01(日) 18:50:32.00 ID:???
>>13
> Bが0.5の速さで移動して、Aから見た場合の現在のCの位置に達したら、
> 0.5ではなくて当然1の速さで移動しなくちゃならない

誰がそんな事を決めたの?
これから何億年、ずっと観測し続けたら、そのままの速度でCの位置に行くかもしれないじゃん
22ご冗談でしょう?名無しさん:2011/05/01(日) 18:50:49.95 ID:???
>>14
直訳:
直径Dの球が、平面内の直径dの穴を通過するには、どれくらい速く動かされるべきか?

図にするとこんな感じ↓?たぶん

 球D
  ●→→→→●
 ̄ ̄ ̄ ̄    ̄ ̄ ̄ ̄ ̄ ̄
      穴d
23ご冗談でしょう?名無しさん:2011/05/01(日) 21:34:24.26 ID:???
>>22
>直径Dの球が、平面内の直径dの穴を通過するには、どれくらい速く動かされるべきか?

↑の意味が良く解からないんだが、どんな速度でもいずれ通過するんではないの?
24ご冗談でしょう?名無しさん:2011/05/01(日) 21:44:39.53 ID:???
>>23
ゴルフ
25ご冗談でしょう?名無しさん:2011/05/01(日) 22:11:17.33 ID:???
>>13
>「同じ速度で膨張しているにすぎない、」と「加速度運動しているように見える」は矛盾しないと思うけどな

はい、矛盾しないと思います。
ということはこの宇宙は等速直線膨張をしていて、
その膨張というのは慣性の法則で、膨張していると見て良いでしょうか?
宇宙の外側には摩擦を生じるものが無いと思いますので、速度vで膨張したら
そのままvで膨張し続けている感じでしょうか。
26ご冗談でしょう?名無しさん:2011/05/01(日) 22:25:34.98 ID:???
>>23
D < d
少なくとも向こうの縁に当たるときに球の重心は穴より下に落ちててはダメだよね
2713:2011/05/01(日) 23:54:39.66 ID:???
>>25
「等速直線膨張」というのがどういう運動のことかよく分からないけど、もしかして>>21が書いているようなものなの?
もしそうなら現実の宇宙とは異なる。観測結果では、地球から離れるほど膨張速度は大きくなる。

>その膨張というのは慣性の法則で、膨張していると見て良いでしょうか?
「慣性の法則で膨張」というのがどういうことなのか全然分からない

>宇宙の外側には摩擦を生じるものが無いと思いますので、
「宇宙の外側」とは何かをあなたがまず定義してください

>速度vで膨張したらそのままvで膨張し続けている感じでしょうか。
宇宙のうち、特定の大きさの領域を取り出して、一定時間後の膨張率を測定するという意味で書いているなら正しい。
もちろん物体は地球から離れるに従って遠ざかる速さが加速することになることになる。
ただし実際には宇宙の膨張速度が近年は加速している。
28ID:jVWHjy8w:2011/05/02(月) 02:17:39.28 ID:8/lKNVnj
宇宙が等方的で、どこでも膨張率が同じなら、2点の離れる速度は2点の距離に比例するから、上に書かれてるように離れるほど速度は大きくなる。
そして膨張率は質量密度 (エネルギー密度) に依存し、密度ゼロで空間曲率もゼロという慣性の法則しか存在しない場合は膨張率もゼロになる。
だから「慣性の法則で膨張」はない。
現在の宇宙は曲率ゼロだからエネルギー密度による重力 (または反重力) が膨張源。
曲率ゼロでスケール因子を a として重力場方程式を解くと
 a'^2/a^2=(32πG/3c^2)T_{0,0}
   T_{0,0} はエネルギー密度
となるから、宇宙が膨張してもエネルギー密度が一定だと指数関数膨張 (または収縮) になる。
29ご冗談でしょう?名無しさん:2011/05/02(月) 03:29:16.20 ID:XqZ8EEDW
まるで科学に疎いだけどオカルトネタで量子の実験の話が出たんですけど スリットに量子?を当てて
波紋ができるとかいうやつ 詳しくはよく分からないんだけど カメラ設置したらまるで意思を持ったかのように計測結果が変わったてやつだけど
もちろん ねたですよね そんな事があれば世の中もっと騒いでるはずだし まじオカルトすぎるでしょう 説明されても分らないのでうそかホントかだけ教えていただければ
ありがたいのですが
30ご冗談でしょう?名無しさん:2011/05/02(月) 05:49:59.42 ID:???
>>29
こういった場で聞きかじった知識だけで論じるから、量子論がオカルト化
するのではないでしょうか?
量子論をわかりやすく解説した本などもありますので、正しく理解してから
質問されることをおすすめします。
31ご冗談でしょう?名無しさん:2011/05/02(月) 06:32:42.12 ID:???
>>29
細かい世界では観測する事で実験結果が変わってしまう事があるというだけの話。
例えばどこに電子があるか他の電子を使って探そうとしても
その電子が探したい電子を撒き散らしてしまうから分からなくなったりする。
確かに一般向けには気を引いたり紙面を確保するために
大げさに書いてあるよな。
32ご冗談でしょう?名無しさん:2011/05/02(月) 06:46:29.92 ID:EWzwxspC
前スレの>>918 さん
ありがとう。
自分で読み返しても、よくわからない表現になりすみません。
ようは、
@表面張力そのものによって支えられているのか。
A表面張力によって水が入ってこない船のような状態が実現されて浮いてるのか。
ということを疑問に思いました。
33ご冗談でしょう?名無しさん:2011/05/02(月) 09:20:34.44 ID:???
ファインマン物理の152ページの回転の話なんですが、
高校の頃に習った回転行列

cos -sin
sin cos

ではなく、ファインマン物理ではその逆行列

cos sin
-sin cos

で回転を表現しているようなのですが、それは何故なのでしょうか?

高校の頃は、回転前は(x,y)、回転後は(x',y')だったのですが、
ファインマンでは回転前が(x',y')、回転後が(x,y)で表現されているのですが、
何故そんなわかりにくい書き方をしているのかわかりません。
回転行列を「基本ベクトルの取り替え」という視点から見ると

cos -sin
sin cos

の行列で表現するほうが圧倒的に理解しやすいと思うのですが。
34ご冗談でしょう?名無しさん:2011/05/02(月) 09:52:15.24 ID:oSfYHvhO
>>33
何が分かりにくいのかが分からないが
単に逆に回転してるだけじゃないの?
どっちに回転させるかなんてどうでもいいじゃん
35ご冗談でしょう?名無しさん:2011/05/02(月) 09:59:08.14 ID:???
自分の場合、

cos -sin
sin cos

は何も見ないで導き出せるのですが、

cos sin
-sin cos

は導き出せないです。
あえて導くとするならいったん
cos -sin
sin cos
を自力で導きだしてからその逆行列を求める
という二度手間をしないと導き出せません。
36ご冗談でしょう?名無しさん:2011/05/02(月) 10:22:53.71 ID:oSfYHvhO
>>35
だから何?結局何が言いたいのかさっぱりだ
君は平行移動という操作を考えるときに
(x-a,y-b,z-c)とするか(x+a,y+b,z+c)とするか
こだわるの?

導出法をアドバイスすると
回転角θを-θに置き換えればいい
37ご冗談でしょう?名無しさん:2011/05/02(月) 11:30:11.11 ID:???
>>35
導出のクセはどうでもいいとして、

物理的にはまったく等価だから、自分の好きなほうで認識すればいい。
ファインマンだって所詮教科書なんだから、エッセンスが同じなら、
自分が認識しやすいほうで消化すればいいよ。

俺も、A21=-sinθの方がすっきりするけど、
それは情操的な問題であって、ファインマンを否定するつもりもなければ、自分を否定するつもりも無い。

そもそも、逆行列でファインマンの回転行列になるんだろ?
まさに、逆回転してるだけでθ→-θと置き換えたことと等価だということを君は証明したじゃないか。
38ご冗談でしょう?名無しさん:2011/05/02(月) 11:35:15.22 ID:???
>>35
その直感が数学の幾何学を発端にしてるとよくある疑問であったりする。

応用問題を実際にやってみると分かるけど、
座標を中心に考えるか、問題にしてる点(点Aを座標軸XからR(θ)Xの表現に変える)を中心に考えるかで、
分かりやすいθの向きってのが逆になるよ。
39ご冗談でしょう?名無しさん:2011/05/02(月) 11:39:53.01 ID:???
線四本と点を一つ考えるだけで(元の軸と変換後の軸)
変換行列はすぐわかるよ。後はθを置換すれば行き来できる
悩むのはたいてい−の位置だから
40ご冗談でしょう?名無しさん:2011/05/02(月) 11:57:42.32 ID:???
研究の次段階のために粘弾性体の基礎をまとめています。
それで上司に数式を説明しなくちゃならないのですけれど、
数式に含まれている数学的な記号の意味が把握できません。
検索しても記号では引っかからないようで、どなたかご助言頂けないでしょうか。

粘弾性体に振動ひずみを与えた際の応力とその位相の変位についてですが、
振動 γ(t)=γ0cos(ωt)
応力 σ(t)=σ0cos(ωt+δ) 
これを粘性成分と弾性成分に分解すると、応力 σ(t)=σ1cos(ωt)-σ2sin
(ωt)で
前者の弾性はフックの法則(変形)、後者の粘性はニュートンの法則から
導かれていることは判るのですが、

参考書にある『∝』という記号の意味が判りません。
弾性 σ(t)=G0 γ(t) ∝ cos(ωt)
粘性 σ(t)=η0γ・(t) ∝ -sin(ωt)    
     {ずり速度γ・=dγ/dtなので、式にγを代入すると、cos(ωt)は微分
されて-sin(ωt)}
という式です。
γを代入して過程の式を省略しているようなものと考えているのですけれども。
∝ は『つまりは』とか『すなわち』っていう意味で取れば良いのでしょうか。
ただしそれだと、弾性のG0と粘性
41ご冗談でしょう?名無しさん:2011/05/02(月) 12:01:17.34 ID:???
申し訳ありません、PCが規制中なんで一端携帯に本文メールして書き込んでるのです
が、
最後が途切れてしまいました。

γを代入して過程の式を省略しているようなものと考えているのですけれども。
∝ は『つまりは』とか『すなわち』っていう意味で取れば良いのでしょうか。
ただしそれだと、弾性のG0と粘性のη0が後ろの式には入っていない理由がわかりま
せん。
要領を得ない長文で申し訳ありませんが、どうかお力添えお願いします。
42ご冗談でしょう?名無しさん:2011/05/02(月) 12:07:55.23 ID:???
比例じゃないの?
"数学 比例 記号"でぐぐってみて
43ご冗談でしょう?名無しさん:2011/05/02(月) 12:10:01.02 ID:???
比例じゃね?
44ご冗談でしょう?名無しさん:2011/05/02(月) 12:27:03.85 ID:???
ありがとうございます!
45ご冗談でしょう?名無しさん:2011/05/02(月) 14:21:00.46 ID:XqZ8EEDW
>>30 31 どうもありがとうございました
ですよね 不覚にもちょっとびびりました。ちなみにこれの307がネタ元です。
46ご冗談でしょう?名無しさん:2011/05/02(月) 14:44:43.07 ID:XqZ8EEDW
>>45 アドレス張り忘れました http://blog.livedoor.jp/nwknews/archives/3696690.html (307)
他にも科学ネタがあるのでよろしかったら論破願います
47ご冗談でしょう?名無しさん:2011/05/02(月) 14:53:47.26 ID:CEThYg45
http://www.youtube.com/watch?v=iINHCgRQbz8に、ニッケルを化合して、賢者の石ができる
銅に、触媒として、化合させて、新物質ができる さらにできる。
登記
48ご冗談でしょう?名無しさん:2011/05/02(月) 16:26:59.52 ID:???
>>35
導出は例えば(x,y)をθだけ回転させたかったらe^(iθ)を掛ければ楽

回転の向きは座標系を回転させるか、それとも物理的な系全体を回転させるかによって変わってくる
49ご冗談でしょう?名無しさん:2011/05/02(月) 16:58:30.95 ID:???
>>40
>『∝』
=『無限大』じゃないの?
50ご冗談でしょう?名無しさん:2011/05/02(月) 17:06:40.54 ID:???
>>49
お前は何アホなことを言ってるんだ
511/2:2011/05/02(月) 17:24:24.59 ID:???
固体物理学を勉強していますが、わからないところがあったので質問させていただきます。

教科書の「ブラッグ反射とエネルギーバンド」という節で、
‘ブラッグ反射を簡単のため1次元の問題として考えると
θ=90°、k=±nπ/a
となる。この条件を満たす波としては、
ψ(+)=exp(iπx/a)+exp(-iπx/a)∝√(2/a)cos(πx/a)
ψ(-)=exp(iπx/a)-exp(-iπx/a)∝√(2/a)sin(πx/a)
があり、両者は進行波ではない’
とありました。

ここで生じた疑問として、
・なぜψ(+),ψ(-)は進行波ではないのか
・その前の節で導いた、周期的境界条件におけるψ=(1/L)^(1/2)・exp(ik↑・r↑)は
進行波を表している(らしい)のだから、ψ(+),ψ(-)も進行波ではないのか
522/2:2011/05/02(月) 17:25:26.83 ID:???
本文が長すぎて怒られてしまいました。

(続き)
また、新たな疑問として、
・フェルミ気体モデルを考たとき、周期的境界条件におけるψ=(1/L)^(1/2)・exp(ik↑・r↑)が
進行波を表しているとのことだが、なぜ時間に依存しないシュレーディンガー方程式の解なのに
定常波ではなく、進行波の解が現れるのか

同じような形の式なのに一方では進行波、一方では定常波と説明され混乱しています。
あと、これはどうでもいいことなのですが、ブラッグ反射と言っておきながらブラッグ回折という
言葉が教科書の同じ節で使われているのはなぜなのでしょうか。反射と回折は別の現象のはずなのに。

いろいろと質問してしまいましたが、お願いします。
53ご冗談でしょう?名無しさん:2011/05/02(月) 17:32:52.72 ID:???
物理化学の領域だからここで聞くけど

有機合成って今のところ理論的には完全に結果を予測できないけど
有機合成を完全に説明する理論って完成する見込みあるの?
54ご冗談でしょう?名無しさん:2011/05/02(月) 17:44:48.10 ID:???
>>51
周期的境界条件での自由電子が進行波だというのは時間依存性まで考えてのこと
時間依存性まで考えるとψ(x)=exp(i(kx-ωt))となるから進行波になっている。
またこのことは確率密度の流れjを計算しても分かる。
j=Im(ψ^*pψ)でこれがゼロじゃないから流れがあって進行波となっている。

一方でψ+,ψ-は時間依存性まで考えるとψ+=exp(-iωt)cos(πx/a)となっていて、
左向きの進行波と右向きの進行波が重なって定在波となっている。
これは流れjを計算するとj=0となっていることからも分かる。
55ご冗談でしょう?名無しさん:2011/05/02(月) 17:54:55.08 ID:???
>>54
>時間依存性まで考えるとψ(x)=exp(i(kx-ωt))
時間依存を考える=exp(-iωt)を掛けるということですか?

>左向きの進行波と右向きの進行波が重なって定在波となっている。
!?
なんのことやら…左向きや右向きってどういうことですか?

勉強不足で回答に対して質問で返してしまい、申し訳ありません。
今まで波を複素数で表すのは三角関数の計算を楽にするためとしか思っていませんでした…
確率密度ってのが虚部とかかわっていてただ今目を白黒させております。
56ご冗談でしょう?名無しさん:2011/05/02(月) 18:09:05.13 ID:???
>>53
ない。
57ご冗談でしょう?名無しさん:2011/05/02(月) 18:11:19.31 ID:???
あ、定在波となっていることは理解できました。
Grapes使って直感的に、ですがw
58ご冗談でしょう?名無しさん:2011/05/02(月) 18:18:33.95 ID:???
>>54
あ!わかったような気がします!
時間に依存しないシュレーディンガー方程式を解いたから時間の項は含まれていなくて
パッと見では定常波のように見えてしまうものの、exp(-iωt)を掛けて時間依存性を考慮すると
進行波か定常波かがよくわかるというわけですね。

でもy=Asinω(t-x/v)として波の式を高校で学習したのに、量子力学では時間の項にマイナスがつくって変な感じですねぇ…
まぁ、そう決まっているものだから仕方ないのでしょうが。
59ご冗談でしょう?名無しさん:2011/05/02(月) 18:39:33.51 ID:???
符号は重要じゃないよ、別に。
量子力学では慣例的にp=-ihd/dx, E=ihd/dt となっているから e^(i(kx-ωt) の形になるけど、
pとEの符号が逆に定義されていれば逆になる
60ご冗談でしょう?名無しさん:2011/05/02(月) 18:45:44.68 ID:???
>>59
そうだったのですか。
陽子の電荷をプラス、電子の電荷をマイナスと決めてしまうのと同じようなものですね。

本当に頭のいい方々のおかげでGWの勉強がはかどりました。
ここで聞いていなかったら延々教科書の意味不明な説明と戦う羽目になっていたと思いますw
どうしてexp(-iωt)を掛けて時間について考えるという説明が教科書でなされていないのか疑問ですねほんと。
61ご冗談でしょう?名無しさん:2011/05/02(月) 19:02:28.74 ID:???
何度も申し訳ありませんが、ラストに一つだけ。
時間依存性を考えるときにexp(-iωt)を掛けるというのはなぜでしょうか。
そういうものだと覚えてしまおうとしましたが、どうもしっくりこなくて…
量子力学の本にもそれらしいことが書いていないのですが、なぜでしょうか。
62ご冗談でしょう?名無しさん:2011/05/02(月) 19:27:54.91 ID:???
時間依存性は時間依存のシュレディンガー方程式 ih_bar dψ/dt = Hψ で決まる。
ハミルトニアンの固有状態を考えると、Hψ=Eψとなってるから、上の式は
idψ/dt = Eψ となってψ(x,t)=e^(-iEt/h_bar)ψ(x)となる。
ここでω=E/h_barとすると、固有状態の時間発展はψ(x,t)=e^(-iωt)ψ(x)となる。
63ご冗談でしょう?名無しさん:2011/05/02(月) 20:46:58.02 ID:???
経路積分みたいな直感的には正しいけど
理論的に正しいと証明するのは困難な例って例えばどんなのがあるっけ
64ご冗談でしょう?名無しさん:2011/05/02(月) 20:48:31.57 ID:???
摂動論全般
65ご冗談でしょう?名無しさん:2011/05/02(月) 20:51:18.32 ID:???
山本の解析力学難し過ぎワロタ。
クラスにこれ理解できるやついるのかよ…(学部三年)
66ご冗談でしょう?名無しさん:2011/05/02(月) 20:55:32.67 ID:???
>>32
前者。ようはアメンボとおんなじ。
67ご冗談でしょう?名無しさん:2011/05/02(月) 21:14:57.63 ID:???
>>65
アレは数学畑の奴が読むとすんなりいける
68ご冗談でしょう?名無しさん:2011/05/02(月) 21:23:16.22 ID:???
自分はあれでテンソルを学んだ
69ご冗談でしょう?名無しさん:2011/05/02(月) 22:25:39.40 ID:???
>>62
本当にありがとうございます。お世話になりました。
学部3回生で量子力学、量子化学、固体物理学、物理化学をやるとかホントうちの学科のカリキュラム意味不明すぎるw
7032:2011/05/02(月) 23:09:28.87 ID:EWzwxspC
>>66
ありがとう。解決してすっきりしました。
71ご冗談でしょう?名無しさん:2011/05/03(火) 02:02:48.29 ID:???
定説では電子って内部構造を持たないとされてますが
光子を吸収したり放出したりするのはなぜですか
72ご冗談でしょう?名無しさん:2011/05/03(火) 04:30:09.29 ID:???
よくある高度と大気の変化の表を見ると熱圏で温度が発散しています。
例えばこことか。
ttp://www.s-yamaga.jp/nanimono/taikitoumi/taikinokouzo.htm
ですが宇宙空間の温度は3Kという話も聞きます。
これはどうしてなのでしょうか?
73ご冗談でしょう?名無しさん:2011/05/03(火) 04:47:15.33 ID:GR7WViTQ
>>71
電子のエネルギー状態が変化するから

>>72
大気圏の外側までいくと空気分子がほとんど無いので温度は下がる
3Kというのは宇宙空間(のいたる所)に存在する電磁波のエネルギーを温度に換算すると
大体それ位の値になるということ
74ご冗談でしょう?名無しさん:2011/05/03(火) 06:53:11.63 ID:???
重力と慣性力が等価であるというのは無理があると思います。
北極の人は上に加速してるからgを感じている
とすると
南極の人は無重量状態になってしまいます。
等価ってどういう意味ですか?
必要十分条件という意味でしょうか?
重力ならば慣性力であるし
慣性力ならば重力である
というのが等価ということでしょうか?
75ご冗談でしょう?名無しさん:2011/05/03(火) 08:30:27.93 ID:wsyUyLZc
>>73
> 空気分子がほとんどない

つまり、あるということ?
粒子1つあたりの運動量は粒子の数と関係あるの?
それとも真空の温度ってこと?
76ご冗談でしょう?名無しさん:2011/05/03(火) 08:40:28.97 ID:adgdlSDS
>>74

> 北極の人は上に加速してるからgを感じているとすると
> 南極の人は無重量状態になってしまいます。

エレベーターで北極のひとは上の階へ南極のひとは下の階へ移動したとします。

↑(上に加速)
◯(地球)
↑(下に加速)

もしエレベーターが重力加速度で動作したならば、南極のひとは自由落下なので無重力に感じます。

> 等価ってどういう意味ですか?
> 必要十分条件という意味でしょうか?
発生原因は事なるが、測定すると区別出来ないと云う事です。

> 重力ならば慣性力であるし
> 慣性力ならば重力である
> というのが等価ということでしょうか?
その通りです。
77ご冗談でしょう?名無しさん:2011/05/03(火) 08:52:09.25 ID:???
>>76
北極の重力を地球が上方向へ加速度運動した結果生じた慣性力と見なすと
南極の説明がつかなくなるので
重力と慣性力は必要十分条件じゃないと思うのですが。
78ご冗談でしょう?名無しさん:2011/05/03(火) 08:57:36.95 ID:adgdlSDS
>>77
おい!人の話を聞けや!

> >等価ってどういう意味ですか?
> 必要十分条件という意味でしょうか?
発生原因は事なるが、測定すると区別出来ないと云う事です。
79ご冗談でしょう?名無しさん:2011/05/03(火) 08:58:09.93 ID:???
>>75
たとえば地球からたまたま飛び出してきた粒子があるかも
しれないが実質的には無いと考えていい
運動量よりも運動エネルギーを考えるべきだが、
粒子数に関係あるかどうかはその意味による

気体の温度=1粒子あたりの運動エネルギー
だから分子が無ければ定義できない
真空であっても電磁波は存在するので「真空の温度」という表現は
ある意味では正しい
まあその辺は真空の定義によるけど
80ご冗談でしょう?名無しさん:2011/05/03(火) 09:20:51.12 ID:adgdlSDS
>>77
貴方が宇宙空間にいて、地球へ向けて自由落下することを想像してみれば良い。
地球から力を受け続け徐々に加速するが、地面に到達すれば加速運動は停止する。
しかし、地上では、引き続き力の作用を受け続ける。

81ご冗談でしょう?名無しさん:2011/05/03(火) 09:26:19.67 ID:adgdlSDS
>>77
要は重力は力の作用で加速運動を引き起こしてるってことだ。
82ご冗談でしょう?名無しさん:2011/05/03(火) 10:01:59.58 ID:???
北極の重力

地球が上に加速した結果生じた慣性力

等価なんですよね?
ということは北極にいる人にとっては地球が上に加速してると考えていいんですよね。
そうしたら南極は無重量状態になるから矛盾が生じますね。
だから背理法により、
重力と慣性力は別物だと僕は思うのですが。。。
83ご冗談でしょう?名無しさん:2011/05/03(火) 10:12:29.84 ID:???
慣性質量と重力質量を同じ発生原理と見なしたのがアインシュタインであり、その思想により構築されてるのが一般相対性理論。
誤解を恐れず一般向けに表現するなら、重力とは物体がへこんだ四次元空間に落ち込むという加速度運動をする際の慣性と看破した。
なんで今の物理学じゃ大体の人間は両者を同じものとして見なす。

>>82
地球の中心に向かって加速度が生じてる。
宇宙を伸び縮みするゴム膜だとして、地球という質量が乗っかってへこんだ空間に落っこちようとする力が重力。
84ご冗談でしょう?名無しさん:2011/05/03(火) 10:26:47.49 ID:???
>>71です。
すみませんが全く意味が解りません。
もう少し詳しく説明して頂けませんか。
85ご冗談でしょう?名無しさん:2011/05/03(火) 10:27:01.60 ID:adgdlSDS
>>82
局所的には等価と云う意味だ。

例えば、貴方の身長が170Kmもあるようなら、足元と頭で重力加速度の値も事なる。

因みに地球が膨張するのと同等の慣性力が働いていると説明すれば判るのだろうか?

局所的には重力と慣性力はカノニカル同型
86ご冗談でしょう?名無しさん:2011/05/03(火) 10:32:43.05 ID:???
つまり北極の重力と南極の重力は働く方向が違うから等価ではないし、
少しでも強さが違えば、それも等価ではないと言ってるわけだな
87ご冗談でしょう?名無しさん:2011/05/03(火) 10:34:36.24 ID:???
>>84
電子の持つエネルギーが減少することで、その差額のエネルギーを持った光子が放出される
88ご冗談でしょう?名無しさん:2011/05/03(火) 10:40:29.17 ID:???
>>82
2つ目の力は本当に発生しているわけではなくて、飽くまでも「仮に地球がそういう運動をしたら」の話
実際に地球が地軸北方向に加速度運動しているわけじゃないから南極云々について考えても意味を成さない

俺も「等価」という表現はあまりいいとは思わない
「万有引力による力を、加速度運動による慣性力で再現できるよ!」ぐらいに留めておけばいいと思う
89ご冗談でしょう?名無しさん:2011/05/03(火) 10:58:49.55 ID:adgdlSDS
>>86
貴方の認識では月の重力と地球の重力とでは等価でないとおっしゃりたいのですね?貴方のことが良く理解出来ました。ありがとうございます。
90ご冗談でしょう?名無しさん:2011/05/03(火) 10:59:05.05 ID:???
>>87
いえ、エネルギーにより放出されるのは分かるのですが、どこから放出されるのかが疑問なんです。
91ご冗談でしょう?名無しさん:2011/05/03(火) 11:21:18.59 ID:???
なんか、自分の理解が足りないのか、それっぽい説明しかできなくて
質問者を納得させられず逆切れしてる奴がいるな。

こんな理解をすれば分かりやすいんじゃないかな。
慣性質量や重力質量の元になってるのは、物体がより自然な空間配置に収まろうとする力。
重力は二次元的に見れば擂り鉢状にゆがめられた空間の中心方向に落ち込もうとする。
慣性がある慣性系から別の慣性系へ加速するとき、物体は今現在いた慣性系に留まろうとする。

両者を空間の歪みにそろえて説明する事もできる。
エレベーターが加速度運動をする時、実は時間と空間が歪んでいる。
光速に近い速度の物体が縮む・時間の流れが遅くなるという逸話で有名な
ローレンツ収縮という現象だが、これにより歪んだ空間により発生する力が
慣性であるという表現もできる。
92ご冗談でしょう?名無しさん:2011/05/03(火) 11:27:49.14 ID:???
>>90
電子からとしか答えようが無い
エネルギーが光子として放出されるわけだから
93ご冗談でしょう?名無しさん:2011/05/03(火) 11:31:20.20 ID:???
>>92
そのエネルギーはどうやって電子から放出されるんですか?
94ID:8/lKNVnj:2011/05/03(火) 11:37:27.97 ID:dAGix/hC
>>71
電子は電荷を持ってるから電磁場をまとってる。
他の相互作用もあるから他の場もまとってる。
それらの場のエネルギーが出入りする。
>>72
宇宙空間を満たしてる輻射は 3 K の温度だが、飛び回ってる粒子は高エネルギーなので気体の温度と見れば高温。
>>74
「局所的に等価」の意味は、一点だけで加速度と重力が等価ということ。
他の場所も含めたら違う。
95ご冗談でしょう?名無しさん:2011/05/03(火) 12:10:12.19 ID:???
>>94
なるほど。
ありがとうございました。
96ご冗談でしょう?名無しさん:2011/05/03(火) 12:13:36.65 ID:???
いろいろ考えたのですが、等価の意味がわかった気がします。
まず宇宙は膨張しています。
地球も膨張しています。
北極は上に膨張していますので、下に慣性力を感じます。
南極は下に膨張していますので、上に慣性力を感じます。
この膨張は加速度を持っています。
重力の正体は空間の歪みというよりは、宇宙の加速度膨張による慣性力
と考えると、重力と慣性力は等価であるとなります。
97ご冗談でしょう?名無しさん:2011/05/03(火) 12:25:54.37 ID:???
96の理屈は思いついたのはいいのですが
何かこれは直感的に違うと思いました。
あとは自分で考えてみます。
ありがとうございました。
98ご冗談でしょう?名無しさん:2011/05/03(火) 12:28:27.10 ID:???
>>93
「電子のエネルギー準位が下がるときに光子が放出される」というのが物理的に得られる全ての知識
どうやったらエネルギー準位が下がるかというと、一般に高エネルギーであるほど不安定なので
低いエネルギー準位に落ちる傾向にあり、それは確率的にしか分からない
99ご冗談でしょう?名無しさん:2011/05/03(火) 12:35:25.29 ID:???
あーあ。>>85が誤解させる気満々な説明するから。
地球の周りの空間は膨張してない。収縮してる。
空間の曲率については幾何学でしっかり定義されてるんで、適当に見方を変える事はできない。
また、仮に膨張してると仮定すると光路の相対性が説明できない。
あと宇宙が膨張モデルじゃなく中立でも収縮でも重力は作用する。
100ご冗談でしょう?名無しさん:2011/05/03(火) 12:49:47.43 ID:???
つまり、質問者はこう考えたんだろ?
・エレベーターは動いてる。上昇という加速度運動をするから中の人は下向きの力を感じる。
・地球上では下向きの力を感じる。これを重力と呼ぶらしい。
・エレベーターと重力は同じ種類の力だそうだ。
・なら地表面も加速度運動をしてなきゃおかしい!
うん、その気持ちは凄くよく分かる。
ならこう考えてみよう。
エレベーターは競泳用プールだ。周りの空間は歪んでない(と、見なせる)。
加速度運動をするのはエレベーター自身だ。自分が泳ぐから水の抵抗を受ける。
地球上の重力は流れるプールだ。周りの空間が歪んでいる。
水が流れているから抵抗を受ける。
空間の方が歪んでいるから、その場で突っ起っているだけで加速度運動になってしまう。
当然、流れに身をまかせれば水の抵抗は感じない。(自由落下する飛行機の中の人は無重力状態の近似を体験できるよね)
101ご冗談でしょう?名無しさん:2011/05/03(火) 13:01:07.41 ID:wsyUyLZc
>>75
thx
予想どおりで安心した
102ご冗談でしょう?名無しさん:2011/05/03(火) 13:03:58.28 ID:???
大気圏外の大気(?)と宇宙の背景放射は互いに熱平衡になってないの?
103ご冗談でしょう?名無しさん:2011/05/03(火) 13:12:27.09 ID:adgdlSDS
>>99
じゃぁその幾何学について説明してくてや。
因みにおれの専門は可換環論で幾何はあまり詳しくないので、それを配慮してくれや。
104ご冗談でしょう?名無しさん:2011/05/03(火) 13:13:40.06 ID:???
>>102
大気圏を含む地球系と外側の宇宙は十分短い時間スケールでは平衡状態にあると言える
地球へ供給されるエネルギーは主に太陽からの電磁波で、同時に地球は外の宇宙空間に
電磁波を放出していて、それらがバランスしている
宇宙背景放射というのは周りに太陽などの光源がなくて電磁波が届かないような宇宙空間でも
3Kの電磁波が存在するということ
105ご冗談でしょう?名無しさん:2011/05/03(火) 13:13:44.81 ID:???
光子は通過する物質と干渉できないから宇宙空間の極小さな密度の物質に熱を与える確率は統計的にほぼ無視できる。
あと背景放射源に熱を与えるのは光円錐的にかなり困難っつーか無理。
106ご冗談でしょう?名無しさん:2011/05/03(火) 13:14:41.07 ID:???
意味不明すぐる
107ご冗談でしょう?名無しさん:2011/05/03(火) 13:19:15.73 ID:???
質問だけど、流体力学系の話題を扱うスレッドってどっかにない?
108ご冗談でしょう?名無しさん:2011/05/03(火) 13:25:28.73 ID:???
>>107 あまりねえな。

自作・フリーソフト流体シミュレーションスレ
http://kamome.2ch.net/test/read.cgi/sim/1303299935/
109ご冗談でしょう?名無しさん:2011/05/03(火) 13:28:06.35 ID:???
>>103
多様体とか幾何関係は物理の人間でも学ぶのに

>専門は可換環論で幾何はあまり詳しくない

とか自分で痛いと思わないの?
110ご冗談でしょう?名無しさん:2011/05/03(火) 13:31:20.99 ID:BXers3hG
ニッケルを、http://www.youtube.com/watch?v=iINHCgRQbz8に化合して、賢者の石ができる
銅に、触媒として、化合させて、新物質ができる さらにできる。
登記
111ご冗談でしょう?名無しさん:2011/05/03(火) 13:38:35.98 ID:???
>>108
トン。シミュレーション板かぁ。

>>109
いや、俺は専門家だぞってゴルァって皮肉かなにかのつもりなんだろ。
有名な話だが位相幾何学じゃコーヒーカップとドーナツが同じ形ってぐらいだしな。
>>103が寒い通り越して痛いのは同意だが。
112ご冗談でしょう?名無しさん:2011/05/03(火) 13:46:06.59 ID:???
xとpの交換関係だけからxのスペクトルが実数全体であることを看破する議論がありますけど、
rとp_r= 1/r d/dr(r) もやはりxとpの交換関係と同じ交換関係を満たすのでrのスペクトルは実数全体である、とならないのはどうしてですか?
113ご冗談でしょう?名無しさん:2011/05/03(火) 13:47:26.16 ID:???
すいません、
p_r = h_bar/i 1/r d/dr(r)
の間違いです
114一般人(高卒):2011/05/03(火) 15:01:36.33 ID:Tcl/+SRH
三日月は実在ですか?視点を全宇宙に同時に取れば、
満月と新月は同じですか?


月光と月影は同じですか? 光は?
115一般人(高卒):2011/05/03(火) 15:09:32.88 ID:Tcl/+SRH
時間やプランク定数と視点はどっちがほんものですか?
116ご冗談でしょう?名無しさん:2011/05/03(火) 15:40:30.43 ID:???
>>114
月光と月影は物質と光の相互作用の結果
新月、満月、三日月はそれを観測という相互作用により得られた結果
>>115
どちらも物理現象として証明可能というだけの事実
117101:2011/05/03(火) 16:07:06.99 ID:wsyUyLZc
すまん、安価ミスってた >>79 宛ね
118ご冗談でしょう?名無しさん:2011/05/03(火) 18:01:58.63 ID:???
解いた答えがルート60.8となったのですが解答はルートがとってある7.79でした
物理はルートをとらなければいけないのですか?
どうやってルートの中の数を少数単位まで導き出すのですか?
119ご冗談でしょう?名無しさん:2011/05/03(火) 18:30:36.42 ID:???
電卓使えば。
120ご冗談でしょう?名無しさん:2011/05/03(火) 18:59:02.28 ID:???
>>118
俺の時代は中学で習った。
こことか。
ttp://homepage1.nifty.com/moritake/sansu/6/heihoukon/PAGE001.HTM
121ご冗談でしょう?名無しさん:2011/05/03(火) 19:58:15.53 ID:ccxjkrae
前スレの989で光波が球面状に広がっていくとはどういう意味ですか?

もう一度分かりやすく説明して頂けるとありがたいです。ちなみに質問はマイケルソン干渉計で反射鏡を動かした時なぜ干渉縞が生成するように見えたのか?です
122ご冗談でしょう?名無しさん:2011/05/03(火) 20:28:48.51 ID:???
>>121
光が一点から放たれた(点光源)と仮定した場合、光源を中心とした球状に
光が広がる(同位相の光は球面上に広がる)と考えられます。

実際のレーザーなどの光源は、平面的な位相面によって広がらずに直進する
性質がありますが、レンズなどによって点状の光源に変換するこが可能です。
123ご冗談でしょう?名無しさん:2011/05/03(火) 20:45:46.00 ID:???
>>122
レーザーだって広がりますよ。
124ご冗談でしょう?名無しさん:2011/05/03(火) 20:47:34.06 ID:???
でっていう
125ご冗談でしょう?名無しさん:2011/05/03(火) 20:48:26.99 ID:ccxjkrae
マイケルソンの場合光を透過と反射光に分けてその二つを干渉させてるんですが、光路差による位相の違いはあるんですか?
126ご冗談でしょう?名無しさん:2011/05/03(火) 21:35:36.59 ID:???
>>125
光路差による位相の違いは、光の波長と関係します。
例えば、光路差が1/2波長の偶数倍であれば同位相で縞の明るい部分、
奇数倍であれば位相が反転して暗い部分なると考えられます。
127ご冗談でしょう?名無しさん:2011/05/03(火) 21:55:27.30 ID:???
>>123
ご指摘ありがとうございます。
レーザーなどの光源によっては・・・
と訂正します。
128ご冗談でしょう?名無しさん:2011/05/03(火) 21:59:47.79 ID:???
質量1kgの球を地面から上方30度の向きに初速度10m/sで投げた
最高点を達した時と地上に達する直前の重力がする仕事の仕事率を重力加速度を9.8として求めよ

答えは速度に初速度の10を使っています
なぜでしょうか?速度は変わらないのですか?
129ご冗談でしょう?名無しさん:2011/05/03(火) 22:12:25.94 ID:???
>>128
意味がわからない。
答えがどうなっているのか具体的に書いてみて。
130ご冗談でしょう?名無しさん:2011/05/03(火) 22:12:51.82 ID:???
>>128
計算してみりゃいいじゃん
131一般人(元尋牛):2011/05/03(火) 22:21:37.89 ID:qA6fsc/U
>>116 月光と月影は物質と光の相互作用の結果
新月、満月、三日月はそれを観測という相互作用により得られた結果


同じ月を、同時刻に視点を変えて見ただけ。 世界も?  
132ご冗談でしょう?名無しさん:2011/05/03(火) 22:30:10.98 ID:???
>>129
答えは最高点がP=1×9.8×0=0
地面につく直前はP=1×9.8×5=49[w]
です
133一般人(高卒):2011/05/03(火) 22:38:44.62 ID:qA6fsc/U
世界は視点が作った、プランク定数(単位選択公理)と月?

つうことはやべ。   視点のせいか?
134ご冗談でしょう?名無しさん:2011/05/03(火) 22:41:51.24 ID:???
>>132
位置エネルギー+運動エネルギーは保存されるだろ?
位置エネルギーが元と同じになったら、運動エネルギーも同じになってる。
質量に変化はないんだから、速さが同じになってなきゃおかしいだろ?
135ご冗談でしょう?名無しさん:2011/05/03(火) 23:43:11.32 ID:UFRXayfo
地球の自転による転向力はなぜ緯度が高くなると大きくなるんですか
赤道では働かないというのもよくわかりません
136ご冗談でしょう?名無しさん:2011/05/03(火) 23:48:29.60 ID:???
>>135
重力の働く向きと遠心力の働く向き
137ご冗談でしょう?名無しさん:2011/05/03(火) 23:59:29.99 ID:???
>>135
転向力そのものは地球上のどこでも同じです。
地面と平行に働く成分が異なるだけです。
138ご冗談でしょう?名無しさん:2011/05/04(水) 00:03:04.10 ID:ccxjkrae
干渉縞についての詳しい返答ありがとうございます!

あと干渉縞が同心円ではなく楕円になるときがあるそうなのですが、どんな時は分かりますか?調べても出てこないので…
139ご冗談でしょう?名無しさん:2011/05/04(水) 00:16:00.08 ID:???
>>138
2つの光軸を左右にずらして干渉させた場合でしょうか?

イメージとは違うかも知れません。
http://ja.wikipedia.org/wiki/%E5%B9%B2%E6%B8%89%E6%B3%95
140ID:8/lKNVnj:2011/05/04(水) 00:34:40.94 ID:3kD9rEpe
>>112
交換関係を満たす実数固有値の作用素の片方を i 倍して
他方を 1/i にしても交換関係を満たすけど、もはや実数固有値でない。
141ご冗談でしょう?名無しさん:2011/05/04(水) 00:43:09.42 ID:???
>>138
あるいは光源の性質が関係しているかもしれません。

半導体レーザーの広がりを補正しなかった場合など
ttp://sunatsubu.at.webry.info/200708/article_2.html
142ご冗談でしょう?名無しさん:2011/05/04(水) 01:24:43.60 ID:???
>>140
エルミート演算子の固有値が実数であることを示すのと同じ方法で、その場合はその演算子の固有値が純虚数であることを示すますよね。
rとp_rの場合、エルミート性と交換関係だけ見ると正準交換関係と全く同じように見えるのですが、その違いはどこから来るのでしょうか。
143ご冗談でしょう?名無しさん:2011/05/04(水) 06:18:21.42 ID:/+1iuSs6
>>139
>>141
ありがとうございます。やはり光源といったそもそもの部分を工夫しないと楕円にはならないみたいですね
144ご冗談でしょう?名無しさん:2011/05/04(水) 09:44:15.54 ID:AJl72T5d
工業高校に通う2年生です。今年から専攻を変えたばかりのため、熱力学がよくわからないです。
ゴールデンウィークの課題に出た問題の中で、特に以下の2問がわからないので解説、答えを教えてくれませんか?m(._.)m

@200万キロワットの原子力発電所での熱効率が0.5の場合、核燃料が発生する熱量は毎秒何ジュールか?
(ワットは1秒間に1Jのエネルギーを放出する能力)
ACOPが5.0の冷房機が電力を1.0kW消費する。冷房能力は毎時何キロカロリーか?また、外部に放出する熱量は
毎時何キロカロリーか?ただし、1キロカロリーは4.2kJであるとする。
145ご冗談でしょう?名無しさん:2011/05/04(水) 10:12:27.48 ID:???
>>144
熱力学というよりも、エネルギー単位の定義と効率の考え方を課題としているようですね。
エネルギー問題や環境問題を理解するのに良い問題ですね。
146ご冗談でしょう?名無しさん:2011/05/04(水) 10:22:52.99 ID:???
原発の熱効率が0.5って現実的なのか?
最新の火力発電だと59%とか行くが、その半分程度だろ?
147ご冗談でしょう?名無しさん:2011/05/04(水) 11:17:24.40 ID:???
誰か相対性理論の本スレ立てて
148ID:8/lKNVnj:2011/05/04(水) 11:22:35.14 ID:3kD9rEpe
>>142
p_r が自己共役作用素でない事は自力で証明できたよ〜ん。
149ご冗談でしょう?名無しさん:2011/05/04(水) 11:31:27.80 ID:???
>>146
少なくとも現在は熱効率0.5は実現できてないよね。
練習問題とはいえこの辺の数字は実際的な値を使ってほしいもんだ。
150ご冗談でしょう?名無しさん:2011/05/04(水) 11:47:57.30 ID:???
つーか熱効率0.5って熱機関の理論最大値じゃなかったか
151ご冗談でしょう?名無しさん:2011/05/04(水) 11:52:13.67 ID:AJl72T5d
皆さんが何を語り合っているのか全く理解できない^^
計算過程すらわからないし 説明できる方いませんか?
152ご冗談でしょう?名無しさん:2011/05/04(水) 11:54:03.60 ID:???
>>150
高温側と低温側の絶対温度比率を拡大させれば熱効率は上げられる。
0.5などという限界はない。

「MACC2 熱効率」でググれ。
153ご冗談でしょう?名無しさん:2011/05/04(水) 12:14:27.15 ID:???
>>150
理論限界は可逆熱サイクルの近似だが。

>>151
別に極初歩的な事しか喋ってない。
分からない単語をググっても理解できなきゃ質問どうぞ。
154ご冗談でしょう?名無しさん:2011/05/04(水) 12:35:52.80 ID:???
>>151
教科書に熱機関とヒートポンプの効率を表す式が載ってるはず。
155ご冗談でしょう?名無しさん:2011/05/04(水) 12:49:12.59 ID:???
いや、この問題は熱機関の効率じゃなくて単位の意味の理解だろ。
しかも答えに限り無く近いヒントがでてるし。
>ワットは1秒間に1Jのエネルギーを放出する能力
>1キロカロリーは4.2kJであるとする
156ご冗談でしょう?名無しさん:2011/05/04(水) 13:13:22.07 ID:???
ところで200万キロワットは電気出力だよね?
熱出力だったら熱効率出した意味ないし。
157ご冗談でしょう?名無しさん:2011/05/04(水) 13:19:10.52 ID:???
猪木・河合の量子力学での、群と表現のところがよく分からない。
ペースが速いというか飛ばし飛ばしというか。
近々の問題を、道具として使う分にはあれで十分というのも理解は出来ますが。

その部分を補完できるような独学者向けの教科書ってありますか?
リー群について、やや数学によってても構いませんが
158ご冗談でしょう?名無しさん:2011/05/04(水) 16:27:02.44 ID:???
速度v=v(t)のとき運動エネルギーmv^2/2を時間積分したいのですがどう積分すればいいんでしょうか?
159ご冗談でしょう?名無しさん:2011/05/04(水) 16:30:01.58 ID:???
>>158
vってなんだったっけ?
160ご冗談でしょう?名無しさん:2011/05/04(水) 16:41:47.35 ID:???
>>159
vは速度ですがv=dx/dtを代入すればいいんですか?
バカですいません・・・
161ご冗談でしょう?名無しさん:2011/05/04(水) 17:06:19.32 ID:???
xってどこから出てきたんだ?
162ご冗談でしょう?名無しさん:2011/05/04(水) 17:43:46.00 ID:???
>>158
運動エネルギーを時間積分したいのか?
正直何を求めたいのかよく解らん。
163ご冗談でしょう?名無しさん:2011/05/04(水) 17:48:31.85 ID:???
多分v=v(t)を時間積分してエネルギー保存則を求めたいんだろうけど
164ご冗談でしょう?名無しさん:2011/05/04(水) 17:52:37.41 ID:AJl72T5d
>>144を書いたものです

@の答えは72ジュールで合ってますか?@は自力で解けましたが、Aがどうしてもわかりません
何方か教えてくれませんか?
165ご冗談でしょう?名無しさん:2011/05/04(水) 17:54:41.70 ID:???
>>161
自分で勝手に仮定した座標値です。説明なしに用いて分かりにくくしてすいません。
>>162
問題文全部書くと
質量mの質点が一定の力を受けて直線運動している。
時刻t1、t2における質点の速さをv1、v2とするとき、その間の平均の運動エネルギーを求めよ
という問題なんですが、解説文には
(t1-t2)^(-1)*∫[t1,t2](mv^2)dtを計算せよと書いてるだけで
この積分がわからないのでここで質問しました。
答えはm(v1^2+v1v2+v2^2)/6です。

分かりにくい書き方をしてすいません以後気をつけます
166ご冗談でしょう?名無しさん:2011/05/04(水) 17:56:40.58 ID:???
>>165の訂正
t1-t2はt2-t1の間違いです
167ご冗談でしょう?名無しさん:2011/05/04(水) 18:03:27.33 ID:???
>>165
>質量mの質点が一定の力を受けて直線運動している。

等加速度運動なんだからv(t)が具体的に時間の関数で書ける
あとは積分の計算するだけ
168ご冗談でしょう?名無しさん:2011/05/04(水) 18:08:58.66 ID:???
>>164
> @の答えは72ジュールで合ってますか?

あってません
なにかものすごい勘違いをしているのでは?
169ご冗談でしょう?名無しさん:2011/05/04(水) 18:13:26.30 ID:???
>>164
W・J・calといった単位の意味と、熱効率・COPの意味をもう一度確認してください。
170ご冗談でしょう?名無しさん:2011/05/04(水) 18:16:43.34 ID:AJl72T5d
>>168
マジすか
というか、COPという存在自体知らないまま課題出されたんです(泣)
171ご冗談でしょう?名無しさん:2011/05/04(水) 18:18:51.66 ID:AJl72T5d
熱効率というのもよくわかりません

もしかして、@は200万×0.5=100万ジュールですか?
172ご冗談でしょう?名無しさん:2011/05/04(水) 18:21:02.61 ID:???
>>170-171
意味が判らないなら、教科書読むなりググルなりしろよ。
173ご冗談でしょう?名無しさん:2011/05/04(水) 18:22:35.17 ID:AJl72T5d
教科書を調べると

熱効率=外部になす仕事/与えられた熱量
COP=移動させる熱量/外部からなされた仕事

仕事と熱量の関係がわかりません
174ご冗談でしょう?名無しさん:2011/05/04(水) 18:26:03.70 ID:???
エネルギーと仕事、仕事率の関係くらいは中学の課程だと思う。
175ご冗談でしょう?名無しさん:2011/05/04(水) 18:27:22.17 ID:AJl72T5d
>>171
は合ってますか?
答えを直接教えてくれないなら私の考えは合ってるかだけは教えてください・・・
176ご冗談でしょう?名無しさん:2011/05/04(水) 18:31:31.00 ID:???
>>175 間違っている。
177ご冗談でしょう?名無しさん:2011/05/04(水) 18:31:56.83 ID:AJl72T5d
Aは見当も付きません
@で1ワットは1秒間に1秒間に1秒間に1Jのエネルギーを放出する能力と書いてあるのに、
Aでは電力を1キロワット消費すると書いてあります さっぱりです
178ご冗談でしょう?名無しさん:2011/05/04(水) 18:32:19.96 ID:???
>>175
あってません

>熱効率=外部になす仕事/与えられた熱量

この「与えられた熱量」を求めろ、という問題だよ(1)は
179ご冗談でしょう?名無しさん:2011/05/04(水) 18:32:35.42 ID:???
>>167
今解けました!レスくださった方ありがとうございました!
180ご冗談でしょう?名無しさん:2011/05/04(水) 18:33:49.03 ID:AJl72T5d
>>178
200万キロワットという数字は「外部になす仕事」に当たるんですか?
181ご冗談でしょう?名無しさん:2011/05/04(水) 18:35:55.63 ID:???
>>180
>>156の言うように、200万kWが電気出力なら、そのとおり。
182ご冗談でしょう?名無しさん:2011/05/04(水) 18:40:56.49 ID:AJl72T5d
なるほど
200万キロワット=200000万(20億)ワット=1秒間に20億Jのエネルギーを放出する能力って意味ですか?
183ご冗談でしょう?名無しさん:2011/05/04(水) 18:41:19.50 ID:???
横レスなんだけど、
COPって、高温側・低温側の温度を定義もせずに使われるものなのかね?
184ご冗談でしょう?名無しさん:2011/05/04(水) 18:47:33.07 ID:???
>>182 OK
「エネルギーを放出する能力」より「仕事をする能力」の方がいいと思うけど、問題にもそう書いてあるしなあ。
185ご冗談でしょう?名無しさん:2011/05/04(水) 18:49:45.94 ID:AJl72T5d
>>184
@の答えは4.0×10^9 J/s で合ってますか?

今からAを考えます(多分わからないと思いますが)
186ご冗談でしょう?名無しさん:2011/05/04(水) 18:53:58.52 ID:???
>>183
省エネ機器の指標にも使われてんだから、
10モード燃費みたいに現実的な設定(室温26℃、外気30℃とか)がされてんじゃね?
187ご冗談でしょう?名無しさん:2011/05/04(水) 19:02:41.99 ID:???
>>185 OK
>>177
例えば、あるエアコンが1Jの仕事で5Jの熱を低温側から高温側へ移動させたら COP5ね。
188ご冗談でしょう?名無しさん:2011/05/04(水) 19:10:29.02 ID:AJl72T5d
ACOPが5.0の冷房機が電力を1.0kW消費する。冷房能力は毎時何キロカロリーか?また、外部に放出する熱量は
毎時何キロカロリーか?ただし、1キロカロリーは4.2kJであるとする。

Aの冷房能力ってどういう計算で求めるんですか?
189ご冗談でしょう?名無しさん:2011/05/04(水) 19:16:19.79 ID:???
この場合、冷房能力:ある環境から熱を取り去る能力
あと>>187
190ご冗談でしょう?名無しさん:2011/05/04(水) 19:23:50.82 ID:AJl72T5d
全然わからないんですけど(泣)

冷房能力のほうはCOP関係なく出すんですか?
冷房能力をx(キロカロリー/h)とすると、
冷房能力は4.2x(kJ/h)といえる
・・・???わけがわからない
191ご冗談でしょう?名無しさん:2011/05/04(水) 19:24:23.04 ID:???
熱力学やってない人間に高温から低温とか理解しろっていうほうが無理
192ご冗談でしょう?名無しさん:2011/05/04(水) 19:29:49.16 ID:???
>>190
水を低い場所から高い場所に写すのに仕事をする必要があるように、熱を低い場所から高い場所へ移すには仕事をする必要がある。
温度差を力学的な仕事に、またはその逆に変換してやる装置を熱機関とか熱サイクルとか呼ぶ。
ここまではOK?
193ご冗談でしょう?名無しさん:2011/05/04(水) 19:31:50.54 ID:AJl72T5d
1.0kw=1秒間に1kJのエネルギーを放出する能力
これを毎時何キロカロリーかに変換すればいいの?

1秒間に1kJだから1時間に3600kJ
3600÷4.2≒857

答えは857(キロカロリー/h)ですか?
194ご冗談でしょう?名無しさん:2011/05/04(水) 19:33:20.43 ID:???
>>190
そこまで解らないんだったら、先生に「解りません」と言って教えてもらう方が吉。
195ご冗談でしょう?名無しさん:2011/05/04(水) 19:35:30.92 ID:AJl72T5d
>>192
何となく・・・

>>193は合ってますか?

>>194
勘弁してください・・・
196ご冗談でしょう?名無しさん:2011/05/04(水) 19:35:51.60 ID:???
>>193
この場合、「エネルギーを放出する能力」より「仕事をする能力」
197ご冗談でしょう?名無しさん:2011/05/04(水) 19:39:03.06 ID:AJl72T5d
COP=吸収熱量/仕事

の式で、COPに0.5、仕事に1.0kWを単位をそろえて代入して、吸収熱量=冷房能力(A前半の答え)を出すのですか?
198ご冗談でしょう?名無しさん:2011/05/04(水) 19:39:29.61 ID:AJl72T5d
間違えました
COP5.0です
199ご冗談でしょう?名無しさん:2011/05/04(水) 19:42:15.40 ID:???
>>197
基本はOK、で何でCOPは0.5なの?
200ご冗談でしょう?名無しさん:2011/05/04(水) 19:47:03.23 ID:AJl72T5d
5.0=x/y

yに代入するのは
1.0kW=1秒間に1kJ=1時間に3600kJ=1時間に857.14キロカロリー

求めるx=5×857.14 ≒4285.7 ≒4286

前半の答えは 4286(キロカロリー/h)ですか?
201ご冗談でしょう?名無しさん:2011/05/04(水) 19:51:38.34 ID:???
>>200
違う
202ご冗談でしょう?名無しさん:2011/05/04(水) 19:52:42.10 ID:AJl72T5d
>>201
マジすか^^
もうだめぽwww
203ご冗談でしょう?名無しさん:2011/05/04(水) 19:53:32.42 ID:???
今北産業
204ご冗談でしょう?名無しさん:2011/05/04(水) 19:54:40.74 ID:???
>>200
まOK、有効数字は2桁。
205ご冗談でしょう?名無しさん:2011/05/04(水) 21:01:18.74 ID:AJl72T5d
バイ猿食らってました

>>204
ということは、4.3×10^3(キロカロリー/h)ってことですね??

最後の最後、>また、外部に放出する熱量は 毎時何キロカロリーか? の計算はどうすればいいのですか?
206ご冗談でしょう?名無しさん:2011/05/04(水) 21:03:37.81 ID:???
>>205
内部から外部へ移動する熱量はわかるよな、他に何かあるか考えてみ。
207ご冗談でしょう?名無しさん:2011/05/04(水) 21:10:27.14 ID:AJl72T5d
W=Q(供給)−Q(吸収) って式を使って

857.14=Q−5×857.14 

∴Q=6×857.14 ≒5.1×10^3(キロカロリー/h)

5.1×10^3(キロカロリー/h)
これで合ってますか?
208ご冗談でしょう?名無しさん:2011/05/04(水) 21:28:51.14 ID:???
答えはいいとして、思考過程が熱機関を逆に回してるよな、
(その式、エンジンなど外に仕事をする熱機関に関する物だろ。)
可逆過程として捉えていいの?俺 熱力取った事無いから解んないや。
209ご冗談でしょう?名無しさん:2011/05/04(水) 21:31:51.24 ID:AJl72T5d
え、考え方間違ってますか?
自分の思考過程は教科書の式に数値を代入してるだけですから・・・
答えは合ってるんですよね・・・あれ?
210ご冗談でしょう?名無しさん:2011/05/04(水) 23:26:49.97 ID:???
計算面倒だったけど誰も答えないからグーグル電卓に問い合わせてみた。
上 4 285.71429
下 5 142.85714
あとは適当に有効数字合わせて。

>>208
可逆サイクルとして考えていいのはカルノーサイクルだけ。
効率が与えられている場合、そこから効率100%だった時の数字が得られる。
211ご冗談でしょう?名無しさん:2011/05/04(水) 23:44:31.52 ID:SV1frKn/
鉛直情報に投げたボールを、2.0秒後に自分で捕球した。次の問いに答えよ
(1)ボールのはじめの速さv0を求めよ。
(2)投げた地点からボールが達した最高点の高さyを求めよ。

(1)を解くにあたりv=v0-gtを使い
vを考慮せずに0とし、v0=gtより
v0=9.8*2
v0=19,6
となったのですが解説には
v0=gt/2=9.8m/sとありました。
これは、y=y0+v0t−gt^2/2より
yとy0を0と考慮してから変形したと思われます。
安直にy0とyは0にしてよいが、vは0にしてはならないと考えたのですが
(2)で
最高点の高さにおける速度vは0だから
v^2=v0^2−2g(y−y0)
この方程式を利用し解くことを考えたのですが「0と考えることができない」vが式に存在します。
しかし解説にはy=v0^2/2g とあります。
このあたりが釈然としないのですがご教授ください
212ご冗談でしょう?名無しさん:2011/05/04(水) 23:47:34.23 ID:???
仕事とエネルギーの勉強をしているのですがある問題の答えで物体を持ち上げるとき腕に加える力が物体のエネルギーの増加になるとか書いてあるんですがこれはなんの公式何ですか?
また、失ったエネルギー=得たエネルギーと書いてあったりするのですがこれも何の公式を使えばこんなことになるのですか?摩擦力が含まれる問題なのですが保存力以外の力が含まれていてもこんな技が使えるならこっちを公式としてのせるべきだと思うのですが
213ご冗談でしょう?名無しさん:2011/05/05(木) 00:02:40.76 ID:???
>>212
あなたは公式病にかかっている。
物理学習でまず大切なのは「概念」。
公式は確かに便利なんだが運用するためにはまずいろいろな現象を
概念、意味と結びつけて理解することに努めましょう。
214ご冗談でしょう?名無しさん:2011/05/05(木) 00:10:52.49 ID:???
>>213
なるほど、公式病でしたか
薬ありがとうございました
215ご冗談でしょう?名無しさん:2011/05/05(木) 01:20:10.87 ID:???
>>211
>v0=9.8*2
なんの断りもなく*2としてるけど、空気抵抗なしなら頂点に達するのは1秒後。
高さyは頂点ではv=0より
0^2=v0^2+2(-g)y
∴y=v0^2/2g
>0と考えることができない
ってどうして?
216ご冗談でしょう?名無しさん:2011/05/05(木) 02:10:14.31 ID:???
>>212
運動エネルギーの変化とされた仕事の関係って高校物理で公式になってなかったっけ
217ご冗談でしょう?名無しさん:2011/05/05(木) 03:58:36.47 ID:???
>>212
りんごを1と置いて1+1=2とするのは何の公式ですか?
100gと200gのりんごは合わせて300gなのですがこっちを公式として載せるべきだと思うのですが

的な。
218ご冗談でしょう?名無しさん:2011/05/05(木) 04:42:01.70 ID:???
大学における実験レポートの書き方について質問します。

まず手書きで書く場合、グラフはグラフ用紙を切り貼りして本文を書いたレポート用紙のたとえば上半分だとかに貼るのか、
それともグラフだけでA4用紙一枚を使うものなのでしょうか。
また、グラフはコピーではなくグラフ用紙をそのまま付けるべきでしょうか。
一般的にどうか、あるいは特に一般的規則はないのかをお願いします。

また、パソコンで書く場合ですが、教官が「本文はパソコンでも構わないが、グラフは手書きの方が望ましい」という旨のことを言っていました。
この場合、グラフだけで一枚使うなら構わないのですが、たとえばページの一部に貼りたい場合、
そこだけ空けておいて糊付けするか、スキャンして印刷するか、どちらが適切でしょうか。
(こちらはどちらでもよいかあるいは教官による気もしますが、もし一般的な常識があればお願いします)
219ご冗談でしょう?名無しさん:2011/05/05(木) 05:25:18.98 ID:???
究極的には上司や担当教官の好みにあわせる。次に優先すべきは自分の好み。
ただ、普通は手書きグラフならグラフ用紙を一枚単位で付録させちゃうな。
小さいの一個だけとかじゃないと切り貼りはあまりしない。
最後のページに図表をまとめて表1参照とか注記したり。
以上は俺の好みだが。
220ご冗談でしょう?名無しさん:2011/05/05(木) 06:00:50.78 ID:???
ありがとうございます。

さっきのレスを書いた時には忘れていたのですが、そもそもグラフは実験中に書き、ノートに貼るように指示されています。
この場合、ノートからコピーして使って構わないんでしょうか。
221ご冗談でしょう?名無しさん:2011/05/05(木) 09:30:59.29 ID:???
プロットしなおす理由がないならコピーでよくね?
つうかそこまで気になるなら問い合わせた方がいいよ。
適切に判断を仰げるというのも実力の内。
222ご冗談でしょう?名無しさん:2011/05/05(木) 10:42:48.59 ID:???
>>213
これは3流の考えだから参考になる
223ご冗談でしょう?名無しさん:2011/05/05(木) 10:52:37.48 ID:???
ありがとうございます
コピーは失礼とか糊付けは失礼とかいう常識がないか気になって質問したんですが、
特にそういうことはなさそうですね
224ご冗談でしょう?名無しさん:2011/05/05(木) 11:07:11.28 ID:???
江戸時代は、天皇のことを京都では「天ちゃん」「天ちゃん」呼んでいたと、
幕府に金をねだってたわけだが、
新政府は、潮流に乗ってたが、実力はまだまだだったから、
不平士族などを抑えるために 神話を協調し天皇絶対で権威付け、
陰から事実上実権を握り叛乱を抑える手段にしただけ、
それが天皇神権説。


アーネスト・サトウ
http://oshiete.goo.ne.jp/qa/4525233.html
謁見前日の1868年3月22日(慶応4年2月29日)
天皇は、頬には紅をさし、唇は赤く塗られ、お歯黒で染められていたと書いています。

江戸時代の天皇の立場がよく分かる。
225ID:8/lKNVnj:2011/05/05(木) 11:33:25.57 ID:iYae6lVL
>>216
>>212
運動エネルギーの変化とされた仕事の関係
 ΔE=F・Δx の事?
226ご冗談でしょう?名無しさん:2011/05/05(木) 11:59:42.17 ID:???
超ひも理論ってあと何年で完成しそうなんですか?
今の高校生が物理の最も難しい理論を完成させようって思って勉強を始めても、すでに手遅れ?
227ご冗談でしょう?名無しさん:2011/05/05(木) 12:01:40.05 ID:???
とりあえず、祭祀とやらを他の何よりも重要のように思ってる奴らは、卑弥呼の時代の祈祷文化から何も進歩してないってことだな。

皇紀2670年だっけ?
じゃあ西暦200年前後と言われる卑弥呼の時代とか何してたんだよという話。
なんで天皇が統治してたのに邪馬台国と言う独立国を勝手に作られちゃってるんだよ?wという話。

はっきり言うと、天皇の歴史は1000年以上サバ読んでて、実在確認されてる天皇は西暦500年ぐらいの人だよ。
神武天皇から漏れなく「漢」字で名前が付いていることには目をつぶるお前ら、いい加減理解しろよ。
紀元前600年とかの日本にいた架空の原始人にとりあえず「チャーリー」とか「ミヒャエル」とか「ロドリゲス」とかそういう名前付けてるのと一緒なんだよ。

松本智津夫と同レベルのインチキ祈祷師の洗脳から早く醒めろよ。
228ご冗談でしょう?名無しさん:2011/05/05(木) 13:01:56.68 ID:???
なぜダンセイリョクの仕事だけ1/2がつくのですか?
229ご冗談でしょう?名無しさん:2011/05/05(木) 13:04:28.00 ID:???
>>228
導出してみればわかる
230ご冗談でしょう?名無しさん:2011/05/05(木) 13:05:28.84 ID:???
>>226
超ひも理論の完成というのが量子論をもれなく説明できるという意味なら恐らく予見可能な未来には無理。
量子論はここ100年ぐらいずっと世界中で模索してるが、それでも完成の目処は一向にたたない。
まさか鉄の熱放射と温度の関係を調べる研究(+α)がここまで発展するとはな……。
231ご冗談でしょう?名無しさん:2011/05/05(木) 13:06:39.79 ID:???
>>228
バネを引き伸ばすのに要する仕事のような場合?それだったら、伸びと力が比例するから。
232ご冗談でしょう?名無しさん:2011/05/05(木) 13:20:38.36 ID:???
大東亜戦争敗戦後、たった一人で抜け駆けして、占領軍と対峙して命乞いした昭和天皇
         ,,-――-ヘ    敗 戦 記 念 ♪
       . /./~ ̄ ̄ヽ ミ 
        | |    . | |  
        | |━- -━| |  捕獲されたわんこw
 General . |/=・=| =・=ヽ|
        (6|   |__i . |9)        / ̄ ̄~ ̄\
. MacArthur |   ._,  |         /./ ̄ ̄ ̄\ |
         \_____丿        |┏━  .━┓| <アメリカの兵隊さんありがとうワン!
      ,--―|\   /|¬―、  .   |/‐(◎)-(◎)‐|.|
    /   .|/~\/\|    \  (6|    |_」  . |9)
   /    __ /・  __|   |   丶  ━━  /
 /   /| | ・ | |・  | ・  |   |    \ ' ̄~ /
<   .<  |  ̄ ̄ |・   ̄ ̄~| . )   /■■■■\
. \  \ .|___|・____/  /  /\| 8 ̄|/\
  .\  \|____回___|  /  /   | 8▲/|   ヽ
   . \ /          \/  /.戦犯 | 8▼  | ヒロヒト |
      /            |8. |     8 |~ ̄      |
      |      i      | ∞∞|  ∞ o  . |   |
      |      |      |  |  ∞  . | o     |   |
      |      |      |  |  |  .  ∧    |   |
    .  |     ||    .  |  |___|   / ヽ .  |___|
    .  |     ||      |   | |   /  | .\   ||
       |    . | |     |.  .巛|u  /. / |  ヽ  u|》》
     . |    | |     |      |/  /  |   \|
       |   . |  |    |  .  . |    | . |    |
     .  |    | . |   . |      |   |  |    |
     .  |___|  |_.__|    .  |__| |___|
       (  )   (   )       (_丿  ヽ_)
        `ー"    `ー"
233ご冗談でしょう?名無しさん:2011/05/05(木) 13:26:04.45 ID:LSh5g3lY
1、ベクトル、スクエアー、ベクトルでだしてもいいけど、2、時間で分って、ベクトル、スクエア、ベクトルでだしても良い 順序変えて、応用してもいいけど。
登記
234ご冗談でしょう?名無しさん:2011/05/05(木) 13:38:12.45 ID:xGfmC68e
エンタルピーって等温変化でゼロですかね??
エンタルピー変化は等圧モル比熱×モル数×温度変化だからゼロだと思うんですけど
本当にそんな単純なものでいいんですか??
なんかゼロじゃないみたいなこと言ってる人がいたから・・・
235ご冗談でしょう?名無しさん:2011/05/05(木) 15:48:59.83 ID:???
>>234
>エンタルピー変化は等圧モル比熱×モル数×温度変化
これが成り立つのは等圧変化のときのみ。
温度一定でも圧力が変わればエンタルピーは変わる。

ただし理想気体に限るならば話は別で、
等温でありさえすればエンタルピーは一定
236ご冗談でしょう?名無しさん:2011/05/05(木) 16:10:50.32 ID:3LPxhubC
光は質量ゼロとはいえ、エネルギー換算の質量はゼロでないのに
どういう仕組みで光速まで加速されるのでしょう
237ご冗談でしょう?名無しさん:2011/05/05(木) 16:13:42.20 ID:???
>>236
加速されるんじゃなくて最初から光速で進む
それにエネルギーがあるからといって質量があるわけではない
238ご冗談でしょう?名無しさん:2011/05/05(木) 17:20:16.23 ID:???
>>236
光は静止質量が0。
はじめから光速の代わりいかなる慣性系においても静止できない。
239ご冗談でしょう?名無しさん:2011/05/05(木) 17:44:22.33 ID:xGfmC68e
>>235
実在気体のときも頭に入れておかないといけませんね・・・
ありがとうございました
240ご冗談でしょう?名無しさん:2011/05/05(木) 22:49:30.37 ID:???
>>238
ライフゲームのグライダーみたいだ
止まると死ぬ
241ご冗談でしょう?名無しさん:2011/05/06(金) 00:42:16.42 ID:mZYl7zW+
ポテンシャルって
水素原子2個が近づくような現象では
ポテンシャルも時間で変化する(電子と近づいてくる陽子の座標によって)として考えないといけないんですか?
242ご冗談でしょう?名無しさん:2011/05/06(金) 03:17:40.62 ID:???
熱力学の問題ですけど
dF=(∂F/∂x)dx+(∂F/∂y)dy
みたいな全微分表記があたえられてて
「dFを(1,1)→(1,2)→(2,2)の経路で定積分せよ」って言われたらどういう計算をすればいいんでしょうか?
243ご冗談でしょう?名無しさん:2011/05/06(金) 03:25:39.67 ID:???
ただの線積分
244ID:8/lKNVnj:2011/05/06(金) 03:34:56.53 ID:echv2dqn
>>241
もちろん
245ご冗談でしょう?名無しさん:2011/05/06(金) 04:03:16.17 ID:???
>>241
見たい現象の時間スケールとポテンシャルの変化の時間スケールの比較による
典型的なのがその例で、電子と原子核の時間スケールが大きく異なるから、
電子の運動見たいときは原子核は止まってると思っていい
246ご冗談でしょう?名無しさん:2011/05/06(金) 11:19:34.10 ID:???
>>227
>神武天皇から漏れなく「漢」字で名前が付いていることには目をつぶるお前ら、いい加減理解しろよ。
おまえこそ大口叩く前に神武とか仁徳のような古代天皇の漢風諡号は奈良末期にまとめて
追贈されたもので、それまではなかったということくらい知っておけ
247246:2011/05/06(金) 11:23:40.73 ID:???
つーか、ここは物理板だな。すまん。続けたいなら適当な板に移るべし
248ご冗談でしょう?名無しさん:2011/05/06(金) 13:42:26.36 ID:???
原子核の安定性は
陽子と中性子の個数で決まる
(シェルが閉じている状態=安定)
と聞いたのですが、
具体的にはどういう原理なのでしょうか?
249ご冗談でしょう?名無しさん:2011/05/06(金) 15:07:11.90 ID:???
>>248
原子核が陽子と中性子の無秩序な寄せ集めではなく、何らかの構造をもって
いるという考えから、その性質をうまく説明できる様々な模型が提案されて
いる中のひとつで、shell(殻)模型という魔法数や核崩壊をうまく説明する、
核子が軌道を回転しているとするモデル。電子の安定殻に相当する考えに類似
するのでしょう。
250ご冗談でしょう?名無しさん:2011/05/06(金) 16:15:10.70 ID:GtYZcgrd
高さhの塔から鉛直に投げあげられた物体Aが同時に同じ場所から初速度0で落下した物体より4秒遅れて地上に達した。Aの初速度を求めよ。って問題ですが解がむちゃくちゃ汚くなったんですが実際どうなりますか?
251ご冗談でしょう?名無しさん:2011/05/06(金) 16:20:48.86 ID:???
物体の位置をy、時刻をtとしてy=0が地面、t=0に物体を落とすとする。
高さhから初速0で落下する物体の時刻t1での位置はy=-1/2gt1^2+h
高さhから初速v0で上に投げ上げた物体の時刻t2での位置はy=-1/2gt2^2+vt+h
これをそれぞれy=0としてt1,t2について解いて、t2=t1+4とすればいい。
tについて両方とも二次方程式なんだから答えは当然汚くなる
252ご冗談でしょう?名無しさん:2011/05/06(金) 16:40:12.72 ID:GtYZcgrd
デスよね!ありがとうございました!
253ご冗談でしょう?名無しさん:2011/05/06(金) 22:59:12.51 ID:???
電荷にプラスとマイナスがあるように質量にもプラスとマイナスが無いとおかしくないですか?
電荷の式と万有引力の式って比例定数が違うだけでまったく同じですよね?
254ご冗談でしょう?名無しさん:2011/05/06(金) 23:00:09.62 ID:???
訂正:電荷じゃなくてクーロン力の式です
255ご冗談でしょう?名無しさん:2011/05/06(金) 23:07:32.82 ID:???
>>244
>>245
ありがとうございました
256ご冗談でしょう?名無しさん:2011/05/06(金) 23:30:51.02 ID:???
>>253
高校レベルではクーロン力も万有引力も同じだが本当は違う
大学に入って一般相対論を勉強すれば分かる
257ご冗談でしょう?名無しさん:2011/05/07(土) 00:16:19.38 ID:???
>>256
それの最終的な結論は、まだこの世の誰も分かってないと思うが。
258ご冗談でしょう?名無しさん:2011/05/07(土) 00:55:17.49 ID:???
>>257
そりゃそうだ。「現在の科学では」という言葉が全てにくっつくな
259ID:8/lKNVnj:2011/05/07(土) 00:55:51.38 ID:9Wo7m/P1
>>250
高さ h から初速 0 での落下時刻 t1 を先に求めると
 g t1^2/2 = h より t1= √(2h/g)
初速 v の落下時刻 t2=t1+Δt では
 g t2^2/2 − v t2 = h より
v = (g t2^2/2 − h)/t2 = (g t2^2/2 − g t1^2/2)/t2 = g (t1+t2)Δt/(2 t2)
260ご冗談でしょう?名無しさん:2011/05/07(土) 01:55:45.92 ID:ni63cDCx
innermost stable circular orbit てなんですか?
261ご冗談でしょう?名無しさん:2011/05/07(土) 05:47:43.72 ID:???
>>253-258に便乗して。
クーロン力は光子が伝達するということですが、
光子は1種類しかないのになぜ同符号の電荷間には斥力、
異符号の電荷間には引力が働くのでしょうか?

ある電荷から出た光子が、相手の電荷に届いて力が働くのだとすると、
光子は自分の出身電荷が+か−か覚えていないと、
相手の電荷に対して引力、斥力のどちらを及ぼすのか分からないように思いますが。
262ご冗談でしょう?名無しさん:2011/05/07(土) 07:26:25.64 ID:???
>>260
英単語(4つ)
263ご冗談でしょう?名無しさん:2011/05/07(土) 09:42:19.55 ID:???
>>260
最も内側の安定した円軌道
264ご冗談でしょう?名無しさん:2011/05/07(土) 10:11:34.03 ID:???
>>261

こんなイメージと、∧∨
こんなイメージ  ∧∧
265ご冗談でしょう?名無しさん:2011/05/07(土) 10:35:33.50 ID:???
>>261
光子は電磁場という場の変動が光速で伝播する現象を量子化したもので
電荷を動かしたときにはじめて光子が生じる。
場の静的な状態を表すものではない。
266ご冗談でしょう?名無しさん:2011/05/07(土) 10:51:26.15 ID:G95hUS1N
青森の原発も全部ダメだって
公表発表 海外 新聞
267ご冗談でしょう?名無しさん:2011/05/07(土) 12:22:31.13 ID:ln0yTsse
大学での物理でいい参考書とかある?教科書だけでまかなってるかんじ?
268ご冗談でしょう?名無しさん:2011/05/07(土) 13:54:27.32 ID:Bz/GIJA4
269ご冗談でしょう?名無しさん:2011/05/07(土) 14:04:19.28 ID:5fMcpcu/
現代の理論物理(数理物理)で
現代の科学力では実験でその成立を確かめることが出来無そうな予想は今どんなものがあるか教えてください。
そういった理論は既存の他の物理理論や実験データと抵触しないことで正当性を得ているわけですが、
携わる方々は、理論体系を構築しながらも、物理理論として成立しないことでもどかしい思いをしてるのでしょうか?
270ご冗談でしょう?名無しさん:2011/05/07(土) 14:06:01.54 ID:5fMcpcu/
>>269
三行目
×そういった理論
○そういった予想
271ご冗談でしょう?名無しさん:2011/05/07(土) 14:23:24.40 ID:???
>>265
縦光子とスカラー光子が静的なクーロン力を伝えているのではなかった?
(観測にはかからんが)
272ご冗談でしょう?名無しさん:2011/05/07(土) 14:45:45.40 ID:???
>>269
すげー分かりにくい文章だな
要するに理論的に予言されている現象で実験的にも検証可能なものは?ってことか?
最近話題なのはCERNのヒッグス粒子に関するLHC実験じゃないかな

後半は理論家が現実と整合しない結果を導いてしまって困ることはないのか?ってこと?
まあその不整合を解消することで大発見につながった例もあるわけだが、普通は
その理論の不備を疑って修正したりするんだけど。。。というか、そういうのが
理論家の仕事であってもどかしいとか以前の問題
273ご冗談でしょう?名無しさん:2011/05/07(土) 15:07:47.18 ID:???
出来なさそう、のほうだと思われ
274ご冗談でしょう?名無しさん:2011/05/07(土) 15:14:53.73 ID:???
>>269
理論的に予測可能な現象は現実に起こる。
失敗する可能性のあるものは失敗する。
by Murphy
275ご冗談でしょう?名無しさん:2011/05/07(土) 15:15:08.77 ID:???
超弦とか当分は無理だろ
重力波も提唱当時は無理だと言われてきたが、近年ようやく手が届くかもしれない領域に入ってきた感じか

物性の方では、実験的検証が難しい予測とかってあるのかなあ
276ご冗談でしょう?名無しさん:2011/05/07(土) 15:23:33.29 ID:???
出来なさそうか
基本的に宇宙論のブラックホールに関する最近の研究は実証しようがないものが多いな
277ご冗談でしょう?名無しさん:2011/05/07(土) 15:37:31.30 ID:???
フランクヘルツの実験について質問させて下さい。
あの実験では、ヒーター電流を固定しているので、陰極から出る電子の数は一定(時間平均を取れば)だと思います。
しかし、加速電圧を上げると、プレートに届く電流(単位時間当たりにプレートに届く電子の数)が(極大極小を経て)多くなっていきます。これはなぜでしょうか。
電子の持つエネルギーが多くなると、ガスを励起させても逆電圧に打ち勝つだけのエネルギーを持っているという説明では納得してもらえませんでした。
逆電圧を掛けなくても、同じ形のグラフが得られたからです。
小耳に挟んだ話ではどうやら希薄なガスが抵抗の役割をしていて、オームの法則が成り立っているかららしいのですが、古典論のオームの法則を量子論的に言えばどうなるのでしょうか?

あと、封入されている気体が希薄なのはプレートに電子を届きやすくするため、逆電圧を掛けるのはグラフの極大極小が綺麗に出るようにだと思うのですが、
他の理由があれば教えて頂けると嬉しいです。
278ご冗談でしょう?名無しさん:2011/05/07(土) 16:06:40.27 ID:???
フレーム・ドラッギング効果確認!!
279ご冗談でしょう?名無しさん:2011/05/07(土) 16:54:26.58 ID:???
理論家は100個デマ言って1個当たれば上出来

らしいよ
280ご冗談でしょう?名無しさん:2011/05/07(土) 17:20:42.87 ID:???
ホーキングなんかが当てはまるなw
281ご冗談でしょう?名無しさん:2011/05/07(土) 17:43:23.19 ID:???
地震の加速度が981ガル(重力加速度と同じ)というのは体感でどれくらいの大きさと考えればいいのですか?
たとえば自由落下の向きが上下に替わるって感じ?
282ご冗談でしょう?名無しさん:2011/05/07(土) 17:57:04.30 ID:???
>>281
地震の揺れの体感は速度、加速度、振幅などが総合されて決まります。加速度だけでは条件不足です。
同じ加速度でも揺れのメインとなる周波数領域が違えば、体感は大幅に異なります。
気象庁震度階級の値は人間の感じる揺れに近いと考えられていますから、加速度よりも震度を重視するべきです。
283ご冗談でしょう?名無しさん:2011/05/07(土) 19:20:15.24 ID:5fMcpcu/
>>272
前半:例えば超弦理論(含M理論)・ブラックホール理論の一部・無のゆらぎ理論とかがそうですね。
その他にはありますか?
アホー知恵遅れで聞いたら実証できないのはぶちゅりじゃない!
最終的に実験でその是非を決することが出来ないと
ぶちゅりは発展しない!(数理物理はどうなんだよ…)ってアホな答えが返って来ました

後半:>>272の言う理論家の仕事が現実に検証不可能だということに
もどかしさを感じているかどうかということです。

284ご冗談でしょう?名無しさん:2011/05/07(土) 19:23:34.86 ID:5fMcpcu/
>>283追加
普通は実験で検証までしてその是非を確かめるまでが
一般的に考える物理屋の仕事ですよね?
285ご冗談でしょう?名無しさん:2011/05/07(土) 19:25:58.63 ID:5fMcpcu/
ウィッテンは自分の理論が検証されることよりも
超弦理論を修正したM理論を構築する中で得られる
数学的構造の方に興味があるのかな〜?と思って
286ご冗談でしょう?名無しさん:2011/05/07(土) 19:35:16.67 ID:???
>>283
おまいの質問は物理屋には手に負えない
287ご冗談でしょう?名無しさん:2011/05/07(土) 19:36:42.89 ID:???
>>286
国語の問題だもんな
288ご冗談でしょう?名無しさん:2011/05/07(土) 19:54:28.59 ID:???
>>287
国語の問題もそうだけど
俺含めて実験できない領域に至った物理理論(予想)の話を
わかりやすく出来る頭脳がいない
289ご冗談でしょう?名無しさん:2011/05/07(土) 20:11:31.83 ID:9kAUgyux
新型加速器でさえ何の成果も上げることができていないんだから素粒子物理はもうおしまい。
290ご冗談でしょう?名無しさん:2011/05/07(土) 20:12:50.82 ID:???
>>289
論文嫁
291ご冗談でしょう?名無しさん:2011/05/07(土) 20:16:10.41 ID:???
まあLHCで出てきうるのはヒッグスとスーパーパーティクルくらいで
超弦の検証はできないけどな

つか超弦の検証は1000年くらい先だとかリサたん言ってたね
292ご冗談でしょう?名無しさん:2011/05/07(土) 20:23:19.92 ID:5fMcpcu/
>>291
>1000年くらい先
その間まではM理論やその他の未解決予想関係はどうやって発展させるんでしょうかね
既存理論との整合性(一見破綻してるように見えるものも含め)を意識し
どうにもならない矛盾が出るまで
数理物理的・数学的に辻褄を合わせながら発展させていくのでしょうか。
293ご冗談でしょう?名無しさん:2011/05/07(土) 20:26:15.70 ID:5fMcpcu/
>>292について
そういった方法で研究を進めている人は世界の物理学者では
どの位いるのですか?数十人レベル?
294ご冗談でしょう?名無しさん:2011/05/07(土) 23:56:00.67 ID:???
明日、模試があるのですが物理の小問集合は何がでるんですが?また電流が範囲に入ってるのですが教科書でいうと電がつくもの全て範囲でしょうか?
295ご冗談でしょう?名無しさん:2011/05/08(日) 00:31:30.11 ID:FqI47FKv
センター模試なら教科書でも読め
296ID:8/lKNVnj:2011/05/08(日) 01:01:23.13 ID:WY/Q40JB
>>291
直接は出来なくても、高エネルギー現象が陽子崩壊のように低エネルギー低確率で影響する現象もあるから、間接検証できる。
>>293
大学院の学生も研究して論文のネタにしてるから大勢いるだろ。
297ご冗談でしょう?名無しさん:2011/05/08(日) 01:06:44.28 ID:FqI47FKv
>>296
現在の科学力では検証できない理論はどのようにして展開されるんですか?
なんとかして間接的にでも検証できる方法を考えるというのもあるとは思いますが。
298ご冗談でしょう?名無しさん:2011/05/08(日) 01:18:35.83 ID:???
1Aの電流が1秒間に運ぶ自由電子の数はなんこか 電子一個のの電荷=-1.6×10^-19

まず1秒間に移動する電荷は1になるのは分かりますがこれは符号がプラスなので正の電荷ですよね?なのになぜ1/-1.6×10^=19になるのですか?
1秒間に流れる電荷が-1なら電子なので-1.6×10^-19で割ってもおkだと思いますが1という正に電荷で割れるのはなぜでしょうか?よろしくお願いします
299ご冗談でしょう?名無しさん:2011/05/08(日) 01:53:01.21 ID:???
>>297
1.現在説明できない現象を説明できる
2.実験で正しいと確認された理論と同じスケールで矛盾しない
3.理論が自己矛盾を含んでいない
4.物理的に実現しえない解を導かない

あたりだったかと。
これを満たしてればあとは基本的にフリーダム…だったはず。
まあ俺は実験屋なのでこれ以上に細かいことは知らないが。

要するにホラ吹きのごとくいろんな説を考えてとにかく説明をつけたい、
モノの解釈パターンを網羅したい理論屋と、とにかく現実の仕組みが何なのかが
知りたい、実際に確かめてみないと気が済まない実験屋の二本柱で
今の物理は回ってる。
300ご冗談でしょう?名無しさん:2011/05/08(日) 03:18:03.26 ID:A9PEqBSW
301ご冗談でしょう?名無しさん:2011/05/08(日) 05:22:49.40 ID:A9PEqBSW
放射能除去装置は、1回使ったら、一週間半から、2週間、光水を周辺において、(六法星の形にしておく)、休ませなければならない。そうでないと壊れる。メンテナンスも慎重をきする。精密だから、取り扱いには、十分注意すること 
京都大の大学院生、博士課程、ポスドク、教授が最新注意をはらって、メンテナンス、管理をすること 絶対命令 東北大なども同じ以上に慎重に 絶対命令 
302ご冗談でしょう?名無しさん:2011/05/08(日) 08:42:51.25 ID:???
>>298
歴史的な経緯により電流は電子の流れと逆向きに流れると定義されているだけ。
地球の北極が磁極で言うとS極であることも、そのように定義されただけ。
303ご冗談でしょう?名無しさん:2011/05/08(日) 09:39:50.48 ID:A9PEqBSW
304ご冗談でしょう?名無しさん:2011/05/08(日) 10:36:49.68 ID:A9PEqBSW
東大文系養護院の子達ありがとう!!この星の海はプルサーマルの海だね!!凄まじい夏が到来したよ。
305ご冗談でしょう?名無しさん:2011/05/08(日) 10:44:33.66 ID:FqI47FKv
>>299
わかりやすいです。ありがとう。
あと、2については、例えば光速度を3.0m/sと考える(もしくはそういう結論に至る)理論
4については、2つの状態が両立する
(シュレディンガーの猫でなくA∧¬Aみたいな)解でしょうか?
その理論物理の成立要件はどの教授の話していたこと、もしくは何の本を参照にしたんですか?
306ご冗談でしょう?名無しさん:2011/05/08(日) 10:47:34.36 ID:???
>>305
なんか怖いんだが
307ご冗談でしょう?名無しさん:2011/05/08(日) 10:50:17.05 ID:A9PEqBSW
東京大学文系養護院の子達のこと。
308ご冗談でしょう?名無しさん:2011/05/08(日) 10:52:07.34 ID:FqI47FKv
>>306
何が怖いんですか?
物理に携わる者のはしくれならもっと明確に説明してください
309ご冗談でしょう?名無しさん:2011/05/08(日) 11:13:23.56 ID:???
>>305
たとえばだけど、2については重力を量子化する理論は必ずマクロな
スケールにおいて一般相対論と無矛盾でなければいけないってこと。
一般相対論の重力を弱くしていくと近似でニュートン力学が出てくるように。

4については、たとえばタキオン。素粒子論ではタキオンが出てくる理論も
構築できるんだけど、理論学者間での解釈によってタキオン込みの理論は
解が不安定になるのでタキオンを出せる理論は信憑性が怪しいって風潮がある。
いくらいろんなホラが可能といっても、理論だけの考察でもこれはさすがに
まずいだろうってのがコンセンサスとして存在してる部分があって、まっとうな
理論はそういうのを含んじゃいけないらしい。

たしかリサ・ランドールの「ワープする宇宙」で読んだ記憶がある。
310ご冗談でしょう?名無しさん:2011/05/08(日) 11:22:50.60 ID:???
>>308
いや、なんか支離滅裂に見えるんだよ、君の文章は
俺は理論屋だけど最終的に実証されないかぎりは無意味だとはおもってる
しかしストリングでも研究を続ければ将来的に実験可能な予言をすることが
出来るかもしれないから、いくら机上の空論だと叩かれても意義はあると思う
最低限、現在分かっている自然現象と矛盾しないことが条件として課されるが、
実際のモチベーションは理論が追いついていない現象を説明する方が現実的では
と思いますよ
311ID:8/lKNVnj:2011/05/08(日) 11:32:13.81 ID:WY/Q40JB
>>299
この 4 要件は当たり前と感じるが、知らない内に常識になってて見たり聞いたりした覚えは無いな。
>>309
タキオンが出てくる理論は、理論の「真空」が本当の基底状態じゃなく、もっとエネルギーの低い状態が存在する場合にタキオンが出るとか。(それをタキオン凝縮というらしい)
312ご冗談でしょう?名無しさん:2011/05/08(日) 11:38:58.38 ID:???
>理論だけの考察でもこれはさすがにまずいだろうって
宇宙項を付けるんですね分かります
313ご冗談でしょう?名無しさん:2011/05/08(日) 17:21:59.41 ID:???
>実際のモチベーションは理論が追いついていない現象を説明する方が現実的では
と思いますよ
数学的構造フヒヒってやってる奴はHENTAIだってことか


314ご冗談でしょう?名無しさん:2011/05/08(日) 17:57:17.91 ID:???
>>313
あやまれ!ヒルベルトに(ry
315ご冗談でしょう?名無しさん:2011/05/08(日) 19:45:27.18 ID:???
物性論と言えば
ボーズ・アインシュタイン凝縮や二次の相転移っていい加減理論的に説明できないの?
316ご冗談でしょう?名無しさん:2011/05/08(日) 20:08:11.14 ID:???
程度によるだろ
理想ボーズ気体のBECなら統計の初歩で習うし
317ご冗談でしょう?名無しさん:2011/05/08(日) 20:55:03.72 ID:B1ByzyG1
>>233
魔法少女まどか★マギカ
http://www.youtube.com/watch?v=ZIoyp2bE5iw
318ご冗談でしょう?名無しさん:2011/05/08(日) 21:54:22.05 ID:C2c8R2Z+
光学について質問致します。

顕微鏡は当てている波(光波・電子波とも)の波長の物体までしか見ることが出来ないと教わって、
何となく解った気になっていたのですが、具体的(理論的?)に、
波長以下の構造を見ることが出来ないとはどういうことなのか、解り易くお教え頂けましたら幸いです。

また、これと関連あるかどうか判りませんが、光導波路への光の入射について、
例えば細いファイバーを使うとはいえ、
1550nmの近赤外光を幅400nm程度の導波路に通すことが出来るのは何故でしょうか。
導波路よりはるかに長い波長の光を導波路に通せる原理を、
これも判り易くお教え頂けましたら幸いです。

以上、諸賢の皆様、どうぞ宜しくお願い致します。
319ご冗談でしょう?名無しさん:2011/05/08(日) 21:57:23.96 ID:???
>>318
目盛りが1m単位でしかついていないものさしでcmを計るのは難しいようなものじゃないか?
320ID:8/lKNVnj:2011/05/08(日) 23:57:33.03 ID:WY/Q40JB
>>318
波を当てて見る場合は回折現象により光学像に波長程度のボケが生じる。
ただしこれは波を当てる場合だけで、狭い所に波を閉じ込める事は可能だから、
閉じ込めた光 (近接場光) を使った顕微鏡は波長以下の解像度が可能。
321ご冗談でしょう?名無しさん:2011/05/09(月) 00:05:23.74 ID:63XS+Khd
巨大な宇宙船が地球に向かってきているという。地球外知的生命体探査(SETI)は3つの巨大な
宇宙船が地球に向かってきていると発表した。ロシアのメディア「pravda」が22日伝えている。
SETIによれば最も大きい宇宙船は直径が240kmの超大型で、残り2つはこれよりも小さい規模。
現在、物体は冥王星軌道の向こう側にあると推定され、まもなく火星軌道に達すると研究所は
予測した。SETI関係者は「今回の宇宙船観察は米国、アラスカに位置したHAARP探査システムで
発見された。宇宙船は2012年12月に地球に到着する可能性が高い」と話している。

米国政府も該当事実に対する報告を受けたとのことだ。

screenshot:english.pravda.ru
■参考リンク
Three giant spaceships to attack Earth in 2012?(pravda)※英文
322ご冗談でしょう?名無しさん:2011/05/09(月) 00:17:50.78 ID:63XS+Khd
323ご冗談でしょう?名無しさん:2011/05/09(月) 00:28:03.80 ID:63XS+Khd
光速より早いのは、場の宇宙船です。
324ご冗談でしょう?名無しさん:2011/05/09(月) 10:27:44.45 ID:tpkWNMs3
院へ進んだら、博士号取るのに論文は3本書かなくてはいけないのですか?
理論物理で査読付きを3本書くなんて
信じられないような気もしますが。。。
325ご冗談でしょう?名無しさん:2011/05/09(月) 12:14:18.48 ID:???
>>324
超特大花火を1発打ち上げてやれw
326ご冗談でしょう?名無しさん:2011/05/09(月) 12:15:07.42 ID:???
【OCN規制?】1000人BINGOでお試し●ゲット【無料】
327ご冗談でしょう?名無しさん:2011/05/09(月) 14:03:39.56 ID:???
>>324
制度は大学によって違うような気がしたが…
328ご冗談でしょう?名無しさん:2011/05/09(月) 15:24:50.64 ID:???
>>324
某旧邸は3本だった。
だいたい一つの壮大な話を小分けにして何本か出すから数自体はそれほど
負担じゃないんだよ。
329ご冗談でしょう?名無しさん:2011/05/09(月) 16:03:15.39 ID:???
3本3本て言うたびにオバQの顔がチラつく
330ご冗談でしょう?名無しさん:2011/05/09(月) 16:19:13.91 ID:8iVlDM9D
スクエア テーゼでしょう。
331ご冗談でしょう?名無しさん:2011/05/09(月) 16:55:20.25 ID:???
332ご冗談でしょう?名無しさん:2011/05/09(月) 18:21:08.56 ID:8iVlDM9D
離反、次元、空間に、バランスをとっている物質、粒子がある。ベクトル、スクエア、ベクトル 計算。
333ご冗談でしょう?名無しさん:2011/05/09(月) 20:30:38.73 ID:???
隣の家の窓ガラスに石を投げれば、ガラスが割れるのは、まあ予想できる。
だが、破片が床にどう飛散するか、現代の科学では正確に予想出来ないよな。

 そ れ を だ よ 、

地中深くの高温高圧環境にある岩盤の破壊だけ、なぜ予想できるというのだ。
30年以内の発生確率87%って、なんだそりゃ?
関係するパラメーターの精度が、全て二桁以上で明らかになってると言うのか?

物理板の皆さん、こりゃ一体どういう訳だ?

地震研の連中は未来の英知でも持ってるのか?
それとも壮大なオカルトなのか?

・・・まさか、単なるバカなのか?
334ご冗談でしょう?名無しさん:2011/05/09(月) 20:34:50.64 ID:???
別に物理的な観点から地中の力学を計算して87%って言っているわけじゃないと思うけど。
335ご冗談でしょう?名無しさん:2011/05/09(月) 20:39:12.29 ID:???
>>334

そか。つまり奴らは考古学者なんだな。
その割に史書を軽視してないか?
336ご冗談でしょう?名無しさん:2011/05/09(月) 21:12:51.81 ID:???
>>334
うん、ある意味天気予報も似たアプローチだよな。
数値予測も取り入れられているけど、手法としては基礎理論からの
ボトムアップアプローチじゃない。
337ご冗談でしょう?名無しさん:2011/05/09(月) 21:29:01.51 ID:???
>>333
地球のマントル対流やガラスの破片の散乱は有限要素法で普通に近似値を算出可能。
諸元の入力や電算機の割り当て時間に現実的な天井があるから完璧な再現は無理だが。
他にも飛行機や新幹線の設計や工学、気象など幅広い分野で応用されてる。
338ご冗談でしょう?名無しさん:2011/05/10(火) 00:21:19.17 ID:???
>>333
まさか、単なるバカなのか?
339ご冗談でしょう?名無しさん:2011/05/10(火) 00:24:04.17 ID:???
>>333
>地中深くの高温高圧環境にある岩盤の破壊だけ、なぜ予想できるというのだ。
>30年以内の発生確率87%って、なんだそりゃ?
>関係するパラメーターの精度が、全て二桁以上で明らかになってると言うのか?

こんな馬鹿なこといってる暇があったら
実際にどう予測してるのかを学んでだ上でこれこれこういう理由で間違ってると批判しろ
340ご冗談でしょう?名無しさん:2011/05/10(火) 00:31:43.51 ID:???
>>334 と >>337 は意見が大分違うな。
後者の方がァャゥィ感じがするのは何故だろう。
341ご冗談でしょう?名無しさん:2011/05/10(火) 01:30:06.40 ID:???
ただまあ、どこでいつをがっちりと予測するってのは不可能だとは
地震予測研究の第一人者の先生が言ってたけどな
342ご冗談でしょう?名無しさん:2011/05/10(火) 02:13:35.24 ID:???
てか、評価に値する予測って一度でも実績あるのか?
343ご冗談でしょう?名無しさん:2011/05/10(火) 07:53:16.15 ID:???
>>342
今回、一応役に立った例もあるよ。
「予測を大幅に超えている」→「この避難場所じゃ危ないから、もっと逃げろ。」ってのはあったらしい。
344ご冗談でしょう?名無しさん:2011/05/10(火) 11:49:05.46 ID:???
現在行われてる地震予知と言うか、予測は飽くまで過去の情報に基づいた統計的なものだと思うけどな。
もちろんプレート境界にひずみが溜まって云々と言うメカニズムを説明するモデルはあるが、
そっちは飽くまで定性的なもので、「何年以内に起きる確率が何パーセント」なんてそこから計算できるものじゃ無いと思う。
345ご冗談でしょう?名無しさん:2011/05/10(火) 13:07:46.95 ID:7Sz0fUN2
>>325
>>327
>>328
(>>329???)
ありがとうございます。
小分けにして3本にすればいいわけですね。
またはものすごい論文なら1本でもいいということですね。

論文を3本に分けるとすると、例えば、
「宇宙を創造する方法」、
「宇宙の維持の方法」、
「宇宙の破壊方法」
という具合にすればいいわけですね?

僕には無理だと思いますけど頑張ってみます。
万一の場合を考えて教職をとっておこうと思います。
そうすると、高校の教員ぐらいにはなれるかもしれませんので。。。
346 忍法帖【Lv=27,xxxPT】 :2011/05/10(火) 15:35:47.05 ID:???
1光年の長さの完全剛体棒があるとして、それを押したり引いたり
してモールス信号にすると、丁度光の速さで情報が向こうにつく、
って言うのを相対論で証明したいんだけどどうすればよいかな?
押した一年後に向こう側が押されるはずだと思うんだけど。。
347ご冗談でしょう?名無しさん:2011/05/10(火) 16:32:37.11 ID:???
>>346
証明なんて無理。だってそんな結果にならないから。

字義通りの完全剛体棒なら1年どころか瞬時に届く。
(したがって相対論が正しければ完全剛体は存在できない)

現実的な棒だと押したり引いたりが伝わるスピードは音速で、
1年よりはるかに長くかかる(数万年オーダー)
348ご冗談でしょう?名無しさん:2011/05/10(火) 19:34:54.76 ID:GO1+jJM0
光速でうんこ
するとどうなるか
教えて下さい
349ご冗談でしょう?名無しさん:2011/05/10(火) 19:50:02.56 ID:???
出るものも出なくなる。
350ご冗談でしょう?名無しさん:2011/05/10(火) 20:40:08.71 ID:4TykbR/U
量子条件って仮定だよね
351ご冗談でしょう?名無しさん:2011/05/10(火) 20:49:58.50 ID:GO1+jJM0
そうです
352ご冗談でしょう?名無しさん:2011/05/11(水) 16:27:28.08 ID:???
>>347
あってるようで間違ってるな。
完全剛体棒なら届くのは瞬時ではない。
何日、何百日か前に届くことになる。それが特殊相対性理論。
瞬時に届くとすれば、それは剛体棒ではない、光の速度で押し引きの運動が伝わる
超過密合金棒(たとえば俺のちんこ)といえよう。
つまり、完全剛体棒の伝達速度は、完全剛体棒の長さに依存するのである。
長ければ長いほど、伝達する運動は昔に行く事になる。
ということは、運動の始点から順に近いほうから最近の過去に移動していく事になるから、
完全剛体棒はバラバラになると、言うことである。
353ご冗談でしょう?名無しさん:2011/05/11(水) 16:29:23.38 ID:???
というか力を加えた時点でローレンツ収縮みたいな現象起こるんでないの?
354ご冗談でしょう?名無しさん:2011/05/11(水) 16:30:23.61 ID:???
シラネーヨ
355ご冗談でしょう?名無しさん:2011/05/11(水) 16:49:22.23 ID:???
暗号解読班!早く来い!
356ご冗談でしょう?名無しさん:2011/05/11(水) 17:25:24.49 ID:???
>>352
完全剛体をどういう意味で使っている?
普通はどんな外力に対してもいささかも変形しないものを指すのだけど。

いったい完全剛体をどんな意味で使ったら
>長ければ長いほど、伝達する運動は昔に行く事になる。
などいうことになるのかさっぱりわからん
357ご冗談でしょう?名無しさん:2011/05/11(水) 18:16:23.11 ID:???
いるよね、相対性理論を根本から誤解してるのに、自分はわかったつもりのやつ
358ご冗談でしょう?名無しさん:2011/05/11(水) 18:27:28.87 ID:???
物理板って脳内理論振りかざすヤツがいるからなあ。理系板でも特異。
化学板だと基本的に知識勝負になるので独自理論はないが、爆発物とか麻薬系のバカが出る。
数学板だと勝敗が歴然としてるので、コンプ抱いてるヤツが腹いせに荒らしに出る。
359ご冗談でしょう?名無しさん:2011/05/11(水) 18:38:31.74 ID:???
>>358
>数学板だと勝敗が歴然

それ、かっこいいな。
だが、個人的には実験系にしておいて良かったと痛感。
360ご冗談でしょう?名無しさん:2011/05/11(水) 18:55:57.24 ID:???
>>358
というか他の理系学問に
相間みたいのがいないからなあ

何かあるっけ? 他の理系学問での独自理論体系って
361ご冗談でしょう?名無しさん:2011/05/11(水) 19:21:07.12 ID:???
>>359
実験系って扱うもの次第だと基本的にニュートンの運動方程式でおk
みたいなことだってあっちゃったりするからなあ

地球重力の精密検証とかだと、使う装置次第じゃ理論に関してはほとんど
高校物理程度で済んであとはひたすら装置の作動原理について
ガリガリお勉強とかな
362ご冗談でしょう?名無しさん:2011/05/11(水) 19:26:20.80 ID:???
>>352
v=r/t
t=0のとき、vは定まらない。

>長ければ長いほど、伝達する運動は昔に行く事になる。
他の部分は酷いがここだけは言いたい事は分かる。
完全剛体なら光円錐の向こう側を突っつける!って事だろ?
その下の行は論外だが。
363ご冗談でしょう?名無しさん:2011/05/11(水) 19:34:38.79 ID:???
光円錐の外側だからって過去だとは限らんぞ。
座標系によっては過去になりうるというだけで
364ご冗談でしょう?名無しさん:2011/05/11(水) 19:36:27.93 ID:???
>>362
コーザリティが破綻してる
365ご冗談でしょう?名無しさん:2011/05/11(水) 19:52:36.21 ID:???
>>363-364
その通りだが忍法帳が消し飛んで言及できんかったんや
366ご冗談でしょう?名無しさん:2011/05/11(水) 20:07:43.35 ID:???
>>360
有機化学なんかは古くていい加減な理論がまだかなりはびこってるよ。
だれも触らないから温存されている感じ。
367ご冗談でしょう?名無しさん:2011/05/11(水) 20:17:10.45 ID:???
>>366
有機化学って逆に
触ったからって新しいものが出てくるわけでもないだろ…
368ご冗談でしょう?名無しさん:2011/05/11(水) 20:19:03.37 ID:???
実験に失敗したら新材質ができたみたいな話は何か聞いたような気がするが
369ご冗談でしょう?名無しさん:2011/05/11(水) 20:27:04.81 ID:???
は?
物質中の衝撃の伝わる速度が光と同じになった瞬間にやっと00000...1秒狂いなく押し引きの運動が1光年先の場所に伝わる訳だが。
(まあ光速に限りなく近いC>Vの範囲ならどんだけ、光速に近くとも1光年先には最低でも1年より多く時間がかかるけど。)
んでもって理論上光速を超えたら時間逆行するんだけど。(それ以前に物質は光速を超える事はできないが)
衝撃も一種の運動で、速度も決まってるんだけど^^
完全剛体なんだから衝撃も糞も外からの衝撃で物質中を運動が伝わる現象は発生しないんだが。
だからそのまま始点から端のほうまで運動が伝わるんだけど。
完全剛体でない限りはその物質の性質だけで、体積関係なく衝撃が伝わる速度が一定。
完全剛体だから、一定じゃなく物質そのものが一体で剛体を伝わっていく、つまり剛体が伝える運動の最高速度は
剛体の長さそのものに比例する以外にありえないんだが。
まあ、君のようなちんちくりんの知識じゃ、理解できなくて当たり前かな。
そんな無知な君を許してあげよう。
今日から君は「オチンポチンコロイチン」に命名
370ご冗談でしょう?名無しさん:2011/05/11(水) 20:27:18.85 ID:???
>>368
実験的にはそうだけど、有機化学の理論に触ったところで何か新しい概念が生まれるとは思えないって話
371ご冗談でしょう?名無しさん:2011/05/11(水) 20:27:39.61 ID:???
>>367
教科書レベルの話がかなり眉唾なんだよ。
たとえばこれとこれがこういう条件で反応するとこれになるという部分は正しいんだけど
それはこういう理由であるという部分が取って付けたようなインチキくさい説明なんだ。
実際自分は化学系で学位も取ったけどこの思いはいまでも変わらないw
372ご冗談でしょう?名無しさん:2011/05/11(水) 20:34:35.32 ID:???
>>369
物理以前に日本語として論外
373ご冗談でしょう?名無しさん:2011/05/11(水) 20:35:50.04 ID:???
まあ、
お前の論理的思考能力も論外だけど
374ご冗談でしょう?名無しさん:2011/05/11(水) 20:39:01.11 ID:???
>>369
いろいろ頭悪い記述満載だが最低限これだけはつっこんでおきたい。
「時間」と「完全剛体」の定義が現代物理の範疇から外れすぎてる。
375ご冗談でしょう?名無しさん:2011/05/11(水) 20:40:21.53 ID:???
は?頭わいてんのか?
お前物理マジでやってんの?その定義っての教えてくれよ
376ご冗談でしょう?名無しさん:2011/05/11(水) 20:44:24.79 ID:???
ググレカス
377ご冗談でしょう?名無しさん:2011/05/11(水) 20:44:36.91 ID:???
お前の定義こそ問題の命題からはずれてるんだけど。
お前のいってる事は完全剛体を過程する以前に
完全剛体の存在否定に他ならないんだが。
378ご冗談でしょう?名無しさん:2011/05/11(水) 20:48:57.17 ID:???
時間の方も絶対時間の存在を前提にしてるな。
絶対静止系信奉者がなんでメタ相対論前提の超光速による時間逆行を語ってるだよ。
379ご冗談でしょう?名無しさん:2011/05/11(水) 20:59:49.77 ID:???
どこをどう読んだら絶対時間(今ググってきたけど)の存在を前提としてると認識できるの?
絶対時間こそ正しかったらそれこそ特殊相対性理論の破綻なんだけど。
何、俺の妄想内だと地点Aにいる観察者と一光年離れた地点Bの観察者の時間の扱いは絶対時間を適応してるとでも言いたいのかな?
地点Aから剛体を動かすだけで地点Aの観察者が一切の運動をしなければ、絶対時間以前に時間に狂いは存在しないんだけど。
それとも場所によって重力の影響が違うやらで一般相対性理論でも持ち出すつもりか?
380ご冗談でしょう?名無しさん:2011/05/11(水) 21:05:42.18 ID:???
>>379
「〜にとっての同時刻」といった概念を持ち出してない時点でダウト。
381ご冗談でしょう?名無しさん:2011/05/11(水) 21:37:01.97 ID:uYk2K6uZ
物理の「力の合成と分解」についての問題です

☆上腕二頭筋(ひじ関節の屈筋)が発生する力(f)のうち、実際にひじを曲げるために使われる力(図中の矢印w)の割合は、曲げ始める前のひじの角度(θ) とどのような関係にあるか?

角度を横軸、力の大きさを縦軸にとってグラフ化しなさい

http://yfrog.com/gysg8guj

※図はURLを参照して下さい


おせーてエロい人(´;ω;`)
382ご冗談でしょう?名無しさん:2011/05/11(水) 21:50:07.91 ID:???
マルチすんな
383ご冗談でしょう?名無しさん:2011/05/11(水) 22:19:36.71 ID:vZms6l2m
…光速うんこ スルーですk
384ご冗談でしょう?名無しさん:2011/05/11(水) 22:22:34.18 ID:???
>>371
実際後付けで、合成屋とかが使う側も先に経験があるから問題ない
逆に有機合成をあまりしない方の人間が理論から入ろうとすると壊滅する
385ご冗談でしょう?名無しさん:2011/05/11(水) 22:27:27.90 ID:???
テスト
386ご冗談でしょう?名無しさん:2011/05/11(水) 22:36:01.68 ID:???
化学反応は時間スケールは短くエネルギースケールは小さくて理論的に扱うのは難しいからな
387ご冗談でしょう?名無しさん:2011/05/11(水) 23:43:58.59 ID:DlU1US7L
物質・エネルギーを完全にこの世から消滅させることは出来ますか?
完全に無に帰すという意味で。
388ご冗談でしょう?名無しさん:2011/05/11(水) 23:49:00.83 ID:???
無理
389ご冗談でしょう?名無しさん:2011/05/11(水) 23:51:51.04 ID:eJGd8QiP
門外漢なので当を得ない質問かもしれませんが
Wikipediaで「シュレーディンガーの猫」の項目を読んでいると
http://ja.wikipedia.org/wiki/%E3%82%B7%E3%83%A5%E3%83%AC%E3%83%BC%E3%83%87%E3%82%A3%E3%83%B3%E3%82%AC%E3%83%BC%E3%81%AE%E7%8C%AB#cite_note-2
> 現在では「シュレーディンガーの猫」のような巨視的に量子力学の効果が現れる実験系が知られており
とか
> 現在ではシュレーディンガーの猫のように(猫ではないが)、
> 巨視的に量子力学的現象が観測されたと報告されることがある。
というような記述を見つけて、大変驚きました。
具体的にはどういう実験対象の重ね合わせが確認されたのでしょうか?
また巨視的なレベルで量子力学的現象が観測されたということは、
この世界は実際に常識的な物の見方からはとても想像できないような
奇妙で不確定的なものだということになるのでしょうか?

よろしければ御教唆ください。
390ご冗談でしょう?名無しさん:2011/05/12(木) 00:14:56.04 ID:???
超流動
超伝導
391ご冗談でしょう?名無しさん:2011/05/12(木) 00:22:21.95 ID:???
BEC
レーザー
392ご冗談でしょう?名無しさん:2011/05/12(木) 00:37:49.05 ID:???
>>390>>391
ありがとうございます
超流動のように特別な条件下で限定的に確認されているということでしたか
393ご冗談でしょう?名無しさん:2011/05/12(木) 00:42:49.12 ID:IRdRwufY
大学3年です。
キッテル固体物理の教科書を読んで勉強しているのですが、
7章の「周期ポテンシャル中の電子の波動方程式」という項目で(22)式の後に出てくる「裸のクーロンポテンシャル」というのが初耳でどういうことかわかりません。
その前に「実際の結晶のポテンシャル〜」とあり、比較のようになってるので実際の結晶じゃないポテンシャル?となんとなく考えましたが検索サイトで探してもわかりませんでした。
よろしくおねがいします。
394ご冗談でしょう?名無しさん:2011/05/12(木) 01:07:55.90 ID:???
結晶中の電子は原子核からのポテンシャルと周りの電子からのポテンシャルを受けながら運動している。
「裸のポテンシャル」と呼んでるのは、原子核からのポテンシャルだけを考えたもので、
これは単純なクーロンポテンシャルになる。

現実に結晶中の電子を考えるときは原子核のポテンシャルだけじゃなくて、他の電子からのポテンシャルも考えないといけない。
そういうとき普通はポテンシャルを、「原子核と内殻電子」をまとめた「イオン」のポテンシャルと、外殻電子からのポテンシャルという風に分ける。
これがキッテルの(24)式の後で言ってる「イオン殻のポテンシャル」と「他の伝導電子の作るポテンシャル」に対応する。
(22)式の後で「実際の結晶ポテンシャル」と言ってるのはこの「イオンのポテンシャル」のこと。

キッテルではこの後、他の伝導電子からのポテンシャルは無視(一電子近似)して、
イオンのポテンシャル中での一電子問題を考えていく。
395ご冗談でしょう?名無しさん:2011/05/12(木) 01:13:52.41 ID:???
>>393
普通のクーロンポテンシャルのことだと思うけど。
結晶を構成する分子の相互作用ポテンシャルってクーロンポテンシャル以外にも色々な要因があってよく分からないけど
少なくともr→0でV→∞でなければならないし、これはクーロンポテンシャルとは全く違う。
396ご冗談でしょう?名無しさん:2011/05/12(木) 02:20:25.51 ID:BoYhnWZx
神奈川お茶からセシウム
ttp://www.chunichi.co.jp/s/article/2011051101000627.html
神奈川の「足柄茶」が基準値超え 県内産の農産物で初
2011年5月11日 17時07分
 神奈川県は11日、同県南足柄市で9日に採取した「足柄茶」の生葉から、暫定基準値を超える放射性セシウムが検出されたと発表した。同県産の農産物が暫定基準値を超えたのは初めて。
 県は南足柄市などに対し、今年産の茶の出荷自粛と自主回収を呼び掛けた。県によると、足柄茶の生葉は県内17市町村で生産しており、他市町村の生葉の検査も早急に進める。
 県によると、南足柄市の生茶から1キログラム当たり550〜570ベクレル(基準値同500ベクレル)の放射性セシウムを検出。放射性ヨウ素は検出されなかった。今年収穫された足柄茶は、6日に出荷が始まったばかりだった。
397ご冗談でしょう?名無しさん:2011/05/12(木) 08:41:43.76 ID:???
>>360
天文・気象板だと宇宙論とか地球外生命体にはいろいろ素人の入り込む
余地があって、脳内持論が展開されることもある。

物理板の荒らしと根っこは一緒だと推定。
398ご冗談でしょう?名無しさん:2011/05/12(木) 09:19:57.42 ID:JF3dN1uj
この実力で、この地位、資格、年収?っていう奴らが沢山いるけど、養護院のフィールドで活躍しているだけだよ。知っておいた方がいい。
399ご冗談でしょう?名無しさん:2011/05/12(木) 09:25:08.46 ID:???
高木仁三郎と出口王仁三郎の区別がついていなかった
400ご冗談でしょう?名無しさん:2011/05/12(木) 09:30:28.06 ID:???
近いうちに相対性理論は破綻しそうじゃないか?
同一理論内に特異点ができたり無限のエネルギーできたり・・・
量子もつれと光速度不変を組み合わせるとタイムスリップするw
401ご冗談でしょう?名無しさん:2011/05/12(木) 11:24:46.67 ID:JF3dN1uj
>>400
エネルギー増幅ってあるんですか?特に、循環に
402ご冗談でしょう?名無しさん:2011/05/12(木) 16:09:14.30 ID:???
>>400
EPRと光速不変を組み合わせてもタイムスリップ何かしないぞ

古澤のブルーバックスで良いからちゃんと読め
403ご冗談でしょう?名無しさん:2011/05/12(木) 17:09:13.71 ID:/7uQzlow
テンソルの変分問題の計算で
∂/∂g_αβ(g^αβg_αβ)
を計算するんですがどのように計算したらよいのでしょうか?
宜しくお願いします。
404ご冗談でしょう?名無しさん:2011/05/12(木) 18:24:10.11 ID:JF3dN1uj
循環関数が大きくなって循環関数が大きくなるのがある。
バカバカしいかもしれないけど、循環関数エネルギー増幅関数エンジン科学波
405ご冗談でしょう?名無しさん:2011/05/12(木) 18:42:11.50 ID:JF3dN1uj
循環関数が大きくなって循環関数が大きくなるのがある。
バカバカしいかもしれないけど、循環関数エネルギー増幅関数エンジン科学波
登記
406ご冗談でしょう?名無しさん:2011/05/12(木) 19:45:32.46 ID:???
円運動の加速度の公式
a=rω^2
r一定、ω一定。故にaは一定?

だったら円運動は速くなり続けるの?
407ご冗談でしょう?名無しさん:2011/05/12(木) 19:52:06.72 ID:???
>>406
加速度で変化するのは速度であって、速さではない
408ご冗談でしょう?名無しさん:2011/05/12(木) 19:58:03.86 ID:???
>>406
それ向心加速度の「大きさ」の式ね。加速度の向きは変化している。
一周する間の加速度を積分すると0、つまり一回りした時の速度変化は0。
409ご冗談でしょう?名無しさん:2011/05/12(木) 20:32:46.83 ID:kUpaOu+Q
さっきテレビでシャコ?がパンチで貝を粉々にしてたw
しかもパンチ力測定したら60キロw


すごくね?
410ご冗談でしょう?名無しさん:2011/05/12(木) 20:37:37.99 ID:???
手の指を叩かれたりしたら骨折するかもね
411ご冗談でしょう?名無しさん:2011/05/12(木) 20:46:49.02 ID:kUpaOu+Q
>>410
ああ

しかも水中でだぜ?
貝よりちょっと大きいだけのシャコが60キロのパンチするなんて聞いたら物理オタが喜ぶと思って報告しました(´・ω・`)
412ご冗談でしょう?名無しさん:2011/05/12(木) 21:01:01.90 ID:???
パイロット解釈について質問させて下さい

この解釈では、ダブルスリット実験において
観測したら干渉縞が消えた事をどう説明しているんでしょうか?
観測した事により、ガイドウェーブに変化が起こったという事でしょうか?
413ご冗談でしょう?名無しさん:2011/05/13(金) 01:23:39.62 ID:???
>>412
粒子的な性質は粒子として、波動的な性質は波として古典的な理論で
量子的な現象を説明しようとしたのがパイロット解釈。
粒子と波とを結びつける何らかの力が作用(一体化)することにより
ガイドウェーブとして機能し、観測することによってその作用が消失(分離)
してガイドウェーブとして機能しなくなると考えられる。
414ご冗談でしょう?名無しさん:2011/05/13(金) 02:12:13.31 ID:???
光の散乱について質問したい

大学で教授が「波長と粒径が等しい時散乱強度は最大になる」って言ってたんだけど
その考えに基づけば空が青く見えるのは
大気中に400nm(=青色の波長とだいたい一緒)の粒子が大量に存在していてるからだよね

でもウィキペディア見ると空が青く見えるのはレイリー散乱が起きているからで
レイリー散乱が起きるのは粒子の大きさが波長よりもはるかに小さい時だって言ってるんだけど

どっちが正しいの?俺文系だからとんでもなく初歩的な過ちを犯してるかもしれんが
答えてくれ
415ご冗談でしょう?名無しさん:2011/05/13(金) 02:34:44.94 ID:???
>>414
どっちも正しいんでないの。
空が可視光波長の粒子で満たされててら、もっと散乱が強くて地表は暗くなってたと思うね。
416ご冗談でしょう?名無しさん:2011/05/13(金) 02:43:22.52 ID:???
>>414
その時の講義は教科書の文面を丸ごと言ってるだけだったと違うん? 400nmの塵が〜』だけだと『朝、夕はナゼ赤い』が説明付かなく
為る。 日照角度だけだと 秋はなぜ赤っぽい- 春はソンナでも無い (銀塩写真だと差が良く判る)
 答案の点を取るなら”センセーの言うとおり”なんだが ワレもこれ以上の説明と為るとつらい

〜でココに来たのは 原発のスレで『液体窒素で凍らせろ』と言うネタを読み・フと思う。 絶対零度付近まで冷えた場合
放射性物質は中性子線を出すのか? 分子、原子運動が減速するのなら〜ほとんど出なくなるのでは?
誰か知見の有る方 実験or論説ソース 求む。
417ご冗談でしょう?名無しさん:2011/05/13(金) 02:43:40.94 ID:???
>>415
携帯からですまんがサンクス
そういうことか
二種類の散乱が起きていると考えていいのかな

一応すげー大学の工学部長やってる教授だから間違いないんだろうけど
俺が混乱してしまった
418ご冗談でしょう?名無しさん:2011/05/13(金) 08:12:27.79 ID:???
>>415
> 空が可視光波長の粒子で満たされ

満たされてたら真っ暗になるだろ
419ご冗談でしょう?名無しさん:2011/05/13(金) 11:23:10.34 ID:???
力学的エネルギー保存の法則についてですがA点で静止している質量1の物体を滑らかな平面で初速度0で動かしたらB点での速度が2だとしA点B点の位置エネルギーはともに0でA点とB点で力学的エネルギー保存の法則を使って式を表すと0=1/2・1・2^2で
0=2となりますが等式は成り立っていませんがどういうことですか?
420ご冗談でしょう?名無しさん:2011/05/13(金) 11:26:14.62 ID:???
>>419
どうやって動かしてんだよ
421ご冗談でしょう?名無しさん:2011/05/13(金) 11:30:43.84 ID:???
>>420
ゆっくりと押せば初速度0で動きますよね?
422ご冗談でしょう?名無しさん:2011/05/13(金) 11:39:27.81 ID:???
式では運動エネルギーしか考えてないじゃん
ゆっくり押すということも加えて式を考えなきゃだめでしょ
423ご冗談でしょう?名無しさん:2011/05/13(金) 11:51:24.46 ID:???
>>419
一番最初が速度0でもBでは2まで加速してるんだろ?
だったら加速の際加えたエネルギーを積分しないと。
仮に初速度0のまま加速させないと『動いてない』ということになり0=0で関係式は成り立つが。
424ご冗談でしょう?名無しさん:2011/05/13(金) 13:50:34.18 ID:ZDHCjceF
東京で最も博士号のとりやすい、
物理の院はどちらでしょうか?
425ご冗談でしょう?名無しさん:2011/05/13(金) 13:50:42.24 ID:???
>>421
>ゆっくりと押せば初速度0で動きますよね?

初速度0のまま力を加えなければ、
速度2にはならない。

従って、逆に言えば、
運動エネルギーは、
ある距離をゆっくり押すという仕事から供給される。

問題文にそっていえば、仕事を加えなければ初速度0のものを
「動かす」ことができない。
426ご冗談でしょう?名無しさん:2011/05/13(金) 21:27:06.21 ID:???
福島第一は
メルトダウン起こして燃料棒が格納容器突き破って外部に流れ出てる
と思っておいて間違いないよね?
427ご冗談でしょう?名無しさん:2011/05/13(金) 21:51:02.92 ID:???
だよね
今まで安心させるような発表ばかりで深刻な事実は後から後から出てくる
428ご冗談でしょう?名無しさん:2011/05/13(金) 22:35:13.36 ID:???
>>424
そんなとこ行ってどうすんの?
ゆとり思考?ただの怠慢?
それで研究者になれるとでも?
429ご冗談でしょう?名無しさん:2011/05/14(土) 00:50:20.22 ID:???
(原発とアキラから思い付いた素朴な疑問です)
核崩壊とかは絶対零度でもおきるのでしょうか。もし起きるのだとしたら、それは現実には本当の絶対零度にできないからおきるのでしょうか、それとも本当に絶対零度になっても理論上おきるのでしょうか。
430ご冗談でしょう?名無しさん:2011/05/14(土) 01:02:54.82 ID:???
別に原子核崩壊と熱量に相関性があるわけじゃないが
431ご冗談でしょう?名無しさん:2011/05/14(土) 01:14:21.27 ID:???
>>430
誰も熱量の話なんかしていないのでは?
それとも温度と熱量の違いもわからない文系シロートさん?
432ご冗談でしょう?名無しさん:2011/05/14(土) 01:15:58.82 ID:???
>>429
私も良くは知りませんが、量子力学によれば
原子は絶対零度においてさえ「零点振動」なる振動をしているのだそうです。
つまり、熱エネルギーが0では無い。

なので、崩壊もするんじゃないですかね?
433ご冗談でしょう?名無しさん:2011/05/14(土) 01:22:48.69 ID:???
>>429
そもそも核崩壊は有限温度だから起きてる現象じゃない。
むしろ絶対零度だと起こらないと思う方が謎

>>432
絶対零度っていうのは系が基底状態にあるときのことだろ
434ご冗談でしょう?名無しさん:2011/05/14(土) 01:30:26.60 ID:???
>>433
絶対零度が有限温度でないと思う謎
435ご冗談でしょう?名無しさん:2011/05/14(土) 02:30:44.81 ID:???
T=0を絶対零度、T>0を有限温度というのは普通の記法だぞ
436ご冗談でしょう?名無しさん:2011/05/14(土) 05:39:47.98 ID:gWn7FxWe
>>405
登記なし
437ご冗談でしょう?名無しさん:2011/05/14(土) 08:08:10.00 ID:???
物理じゃなくて機械工学とかのような気がするんだけどここで質問

今後、センサー類は全部有線じゃなくて無線でデータを観測ステーションみたいなところに飛ばすようになると思うんだけど

例えば原発事故みたいのが発生してそういうデジタルなデータを飛ばせなくなって
むしろアナログな水位計(そんなのが原発についてるのか知らんが)とかのが信頼できる

みたいな事態が生じた場合はどうやってバックアップを計るように計画されてるんでしょうか

身近な例では最近の自動車なんかもそんな感じの制御らしいけど、あれも計器類が、それこそハンドルやブレーキがエラー吐いたらどうするの
438ご冗談でしょう?名無しさん:2011/05/14(土) 08:43:59.87 ID:Q8651ifW
物理ド素人からの質問です。

ネットで色々見てるんですが、飛行機が飛ぶ際の揚力発生のメカニズムが
学者によっても説明の仕方が違いますよね?

@ベルヌーイの定理
A作用反作用の法則
B流線曲率の定理
などなど・・・

一体どれが正しいのでしょうか?
飛行の状態(迎角の有無など)によってメカニズムが異なるのでしょうか?
それとも全てがバランスよく発生している結果飛ぶのでしょうか?

アホな質問ですみません・・・。
439ご冗談でしょう?名無しさん:2011/05/14(土) 08:48:27.26 ID:???
>>437
>今後、センサー類は全部有線じゃなくて無線でデータを観測ステーションみたいなところに飛ばすようになると思うんだけど

そんなことにはならない
440ご冗談でしょう?名無しさん:2011/05/14(土) 09:01:30.22 ID:???
>>413
遅レスすみません

なるほど。ありがとうございました。
441ご冗談でしょう?名無しさん:2011/05/14(土) 09:06:00.94 ID:???
>>438
一つの事象に正しい説明が複数あっても構わない。
442ご冗談でしょう?名無しさん:2011/05/14(土) 09:45:56.55 ID:???
>>438
亜音速での揚力発生の原理は100年ぐらい前からクッタ・ジューコフスキーの定理(クッタの条件・ジューコフスキーの仮定)による循環(馬蹄渦)の発生が定説。
しかしこれは一般の人には難解なので、分かりやすく説明しようという努力がかえって変な説を蔓延させる羽目になった。
443ご冗談でしょう?名無しさん:2011/05/14(土) 09:50:28.98 ID:???
>>433
もし崩壊するならそれは系が真の基底状態にないってことだろ

131Iを集めた系Aと絶対零度の真空Bを用意して、束縛されてない電子だけ通す仮想的な壁で接触させると、
β崩壊で出た電子は自明にAからBへエネルギーを輸送するだろ
よってAは絶対零度ではあり得ない
444ご冗談でしょう?名無しさん:2011/05/14(土) 12:45:50.91 ID:???
そういう事じゃないと思うんだ。
445ご冗談でしょう?名無しさん:2011/05/14(土) 12:59:37.76 ID:???
じゃあどういうこと?
446ご冗談でしょう?名無しさん:2011/05/14(土) 13:19:03.26 ID:mQk3Fe56
重力加速度が測定地点によって変わる、重力異常というのがありますよね?

「測定地点の近くに山があると、その山の影響を受け重力加速度が文献値と変わる」
というのは分かりましたが、その地形補正をどうやってするかが分かりません。

与えられた課題には山の重量を最初に求めるようになっており、
いくら検索しても重量を使った地形補正の仕方を見つけることができません。

どうやって求めたらいいのでしょうか。
447ご冗談でしょう?名無しさん:2011/05/14(土) 13:46:11.56 ID:???
>>446
ベクトル合成
448ご冗談でしょう?名無しさん:2011/05/14(土) 14:47:22.17 ID:qbrtmh2J
宇宙物理に分類されると思うので ここで失礼します

宇宙にはすごい長い線があってそれによって力が発生する的な記事をどこかで読みました
もう一度読みたいのですが見つかりません
ご存知の方がいらっしゃいましたらURLを教えてください
449ご冗談でしょう?名無しさん:2011/05/14(土) 15:26:40.16 ID:2c8xzq1e
どれくらいのダメージが出るのか質問です
アルミ合金でできたM113装軌式装甲兵員輸送車は側面の装甲が 44mmあります
50キロほどある4tトラックのタイヤくらいの大きさの物凄く硬い球体が
直撃すると直径2mある大木が軽く折れる速度でこの装甲車の側面にぶつかったら
どれくらいのダメージがあると考えられますか?
450ご冗談でしょう?名無しさん:2011/05/14(土) 15:35:39.33 ID:???
ドガッ!となってガシャーン!グワアアン…となるぐらい
451ご冗談でしょう?名無しさん:2011/05/14(土) 15:56:23.81 ID:???
50キロほど
4tトラックのタイヤくらい
物凄く硬い
直撃すると直径2mある大木が軽く折れる


曖昧にもほどがあるw
452ご冗談でしょう?名無しさん:2011/05/14(土) 16:36:16.28 ID:2c8xzq1e
要は大木が簡単に折れる衝撃で装甲車にどんなダメージが出るかってことなんですが
おおまかな感じでいいのでこれでなんとかならないですか?
453ご冗談でしょう?名無しさん:2011/05/14(土) 17:01:47.16 ID:???
ゆとりここに極まれり。
454ご冗談でしょう?名無しさん:2011/05/14(土) 17:22:30.47 ID:???
>>448
エスパー回答してみるテスト。
「宇宙ひも」でググレ。
455ご冗談でしょう?名無しさん:2011/05/14(土) 17:24:08.41 ID:???
直径1mで重さ50kgの凄く堅い球体を直径2mの大木の中心にぶつけるとします。
ぶつける球体の速度をどんどん上げていくと、最終的に大木を軽く貫通するが
穴が空くだけで大木は倒れない、なんてことは起こりえるのでしょうか。
よろしくお願いします
456ご冗談でしょう?名無しさん:2011/05/14(土) 17:36:10.23 ID:???
>>448
宇宙ひものことかな?

ググればいいと思うよ。
ウィキペにも項目立ってるし。
457ご冗談でしょう?名無しさん:2011/05/14(土) 17:39:01.26 ID:???
>>455
ユゴニオ弾性限界を十分に突破し、物体を塑性変形させればいい。
現実に戦車砲のAPFSDSの侵徹原理がそれで、
貫通すべき装甲を流体(液体ではない)と見なすに十分な状態にする事を志向している。
458ご冗談でしょう?名無しさん:2011/05/14(土) 18:48:30.41 ID:???
装甲板に対して木は不均質だからユゴニオ限界って一意には見積もれないだろうなぁ。
方向性とかありまくりだし。
459ご冗談でしょう?名無しさん:2011/05/14(土) 19:14:50.62 ID:???
そういう場合はより拘束条件の強い方向で見積もるのが普通。
ただ、現実的な話をするならそれ以前に弾体が大きすぎ密度が低すぎるのが気になる。
460ご冗談でしょう?名無しさん:2011/05/14(土) 23:22:37.43 ID:???
くっつきます!「磁石少年」に世界が注目
http://www.tv-asahi.co.jp/ann/news/web/html/210514028.html

プライパンがくっつくそうですが、なぜこんなことができるのですか
461ご冗談でしょう?名無しさん:2011/05/14(土) 23:47:35.09 ID:???
たいてい身体から出る汗の水分でくっついてるだけ
その証拠に身体の汗をきれいに拭き取ると付かなくなります
462ご冗談でしょう?名無しさん:2011/05/15(日) 00:11:31.37 ID:???
>>460
鉄だけど表面の凹凸が大きくて密着しない物でも付くかとか
物と体の間に薄いシートを挟んでみるとか
プラスチック、陶器、非磁性体をくっつけさせるとか
試すべき事があるだろうに

取材から放送採用決定まで関わった人間は
文系の中でも特にバカな奴らしかいなかったんだろうな
自分たちはアホって堂々と放送して恥ずかしくないのかな
463ご冗談でしょう?名無しさん:2011/05/15(日) 00:20:30.46 ID:???
>>462
お前は単に文系を叩きたいだけだろうが

スタッフ連中はとりあえず視聴者を驚かせるため、数字を稼ぐために必死になってる
視聴者を驚かせなきゃいけないのに
なんでわざわざこの少年が磁石少年じゃないと示す実験をする必要があるのか
その点ではこのスタッフはお前より優秀

そしてお前はテレビがどういうものかわかってない
ネイチャーに投稿してるんじゃないんだぜ?
464ご冗談でしょう?名無しさん:2011/05/15(日) 00:20:50.70 ID:???
スプーン曲げの超能力者がファインマンのいるときにスプーン曲げやろうとすると
何故か全員ことごとく調子が悪くなってスプーンが曲がらないって話を思い出した
465ご冗談でしょう?名無しさん:2011/05/15(日) 00:26:25.54 ID:???
>>463
そうだったな

事実を伝える事でなく視聴者を釣る事が目的だったな
だからTVは嫌いだ
466ご冗談でしょう?名無しさん:2011/05/15(日) 00:36:33.44 ID:???
>>465
2ちゃんねらにしては嫌に素直だな


だまされやすいほうがテレビは楽しいんだろうな
最近テレビがつまらなくて仕方ないのは科学的思考が身についてしまったからだろうか
理系というか 勉強する人間の悲しい性だね
哲学者は早々に世界に失望して自殺するらしいし
467ご冗談でしょう?名無しさん:2011/05/15(日) 00:53:46.72 ID:???
あのーキモイんですけど
468ご冗談でしょう?名無しさん:2011/05/15(日) 01:43:53.47 ID:???
>>461
なんだ
手品みたいなものか
469ご冗談でしょう?名無しさん:2011/05/15(日) 05:49:07.84 ID:1TdnTaE5
板違いかもしれませんが、物理の教科書に似たようなのがあって
http://cis.k.hosei.ac.jp/~kano/Lecture_Entropy/STATISZTIKUS/amount%20of%20informationQA.pdf
の2ページ目の右下の「からベータが決まる」というのが分かりません。
どうやって「ベータ」が出るのでしょうか?
お願いします。
470ご冗談でしょう?名無しさん:2011/05/15(日) 07:49:45.15 ID:???
βの定義に立ち帰れよ。
471ご冗談でしょう?名無しさん:2011/05/15(日) 08:33:58.35 ID:gX0xDFwi
人類は宇宙に行けるのに、なぜサイコロの出目は予測出来ないのか?
472ご冗談でしょう?名無しさん:2011/05/15(日) 09:01:38.14 ID:???
思考実験みたいな質問なのですが、こちらでしてもよろしいでしょうか?
つい先ほど、「電子は観測されるまでは波の中にある確率分布で存在している」と学びました
そして観測された時点で波の性質を失い、粒子として扱われるとも書いてありました
では、もし私たちが世の中全ての電子を観測することに成功したと仮定すると
世の中の電子は全て波の性質を失い、粒子性だけを持つことになりますよね?
この粒子性しかもたない状態になるとマクロな視点で見た世界に何か影響はでてくるのでしょうか?
私の勘違いなどありましたら申し訳ありません。
473ご冗談でしょう?名無しさん:2011/05/15(日) 09:05:37.71 ID:???
>>471
出目は予測できなくても出目の確率は予測可能。
474ご冗談でしょう?名無しさん:2011/05/15(日) 09:22:52.82 ID:???
>>472
電子が粒子性しか持たない場合、原子核周囲の軌道を回り続けることができな
いと考えられています。
したがって、物質の基本構造が崩壊することになり、現在の世界の存在もなく
なります。
475ご冗談でしょう?名無しさん:2011/05/15(日) 09:39:16.12 ID:icquE7U0
>> 472 電子が波動性しか持たない場合、人間の神経やミトコンドリアが
働かなくなります。 なぜなら、神経伝達は、細胞膜の内側と外側での
電子の明確な極在変化によるスパイクを伝達させることによって起こるからです。
つまり、電子はある1点に極在する明確な荷電粒子です。
同じく、ミトコンドリアの電子伝達系もそうです。
つまり、あなたが今そこに存在しないのと同じです。
電子が波動性しか持たないと。
476ご冗談でしょう?名無しさん:2011/05/15(日) 09:45:23.92 ID:???
>>471
出目が分からなくても行けるから
477ご冗談でしょう?名無しさん:2011/05/15(日) 09:51:01.51 ID:???
>>454さん>>456さん
ありがとうございます
ビンゴでした
ググったら見つかりました
ウィキも読んでみますm(_ _)m
478ご冗談でしょう?名無しさん:2011/05/15(日) 14:01:58.87 ID:???
俺気づいたんだけど、携帯電話を机の端っこに置く
バイブレーションさす
すると、机と垂直になる方向に携帯電話がずれながら移動する
これは物理的にはどう説明すればいいの?
やじろべえみたいに、右に傾ければ左、左に傾ければ右に補正されるように動く
479ご冗談でしょう?名無しさん:2011/05/15(日) 14:39:49.32 ID:???
>>472
観測した直後には粒子性を示す粒子も、すぐに空間的に広がって波動性を示す
480ご冗談でしょう?名無しさん:2011/05/15(日) 14:57:55.73 ID:???
>>478
すまんが意味不明。
もっと詳しく説明するか図を書くか動画にとって上げるかしてくれ。
481ご冗談でしょう?名無しさん:2011/05/15(日) 15:00:37.54 ID:???
小麦粉をボウル入れて横からボウルをこつこつ叩くと叩いてる方向に小麦粉がせり上がってくる
というのと似たような状況?
482ご冗談でしょう?名無しさん:2011/05/15(日) 15:32:31.13 ID:g1+4Q+wG
>>472
粒子性しか持たない状態などにはなりません。貴方の言っていることは、他の
電子との相互作用を持たない1電子の状態の収縮というで、波動性を失うのであ
り、現実に存在する極めて多数の電子系は、個々の電子が他の電子や他の粒子や
場と相互作用するので、状態の遷移というものが起こり、多体系としての波動性
は失われません。ある一つの電子の状態を観測して、その状態を知った後、他の
電子の状態を観測して、その状態を知った時点で、始めに観測した電子の状態は、
確率論的解釈しか出来ないのです。ですから波動性が失われることはなく、マクロ
な状態に変化はありません。
483ご冗談でしょう?名無しさん:2011/05/15(日) 17:27:57.50 ID:???
生成消滅を考えなければ全粒子の位置の同時固有状態をとることは可能だろ
もちろん、一瞬の話だが
484ご冗談でしょう?名無しさん:2011/05/15(日) 18:32:17.61 ID:???
はやぶさを見に科学館に行きました


ところでそこで親子連れが居たんだが
父親が間違えまくった解説を子どもにしてたんだがこういう場合正しいことを教えてあげるべきなのか?
よその家の父親の面子を潰すのもどうかと思うし…
485ご冗談でしょう?名無しさん:2011/05/15(日) 18:33:16.73 ID:???
ほっとけ
486ご冗談でしょう?名無しさん:2011/05/15(日) 18:42:46.99 ID:???
解説員が、間違えまくったってほどでもないが多少不適切な解説してたときはどうすべきか悩ましい
487ご冗談でしょう?名無しさん:2011/05/15(日) 18:58:29.87 ID:???
>>484
絶対に口出しするな
小さな子供に取っては父親ってのは絶対の存在
その権威を潰してしまうのはその子供に取って、ちっとくらい間違った知識を覚えるよりはるかに悪影響がある
488ご冗談でしょう?名無しさん:2011/05/15(日) 19:01:08.31 ID:???
つーか、赤の他人がドヤ顔で親子の会話に
割って入るってどんだけ非常識だよ。
489名無しの歩き方@お腹いっぱい。:2011/05/15(日) 19:42:14.94 ID:BMSzIY9t
カオス解析初心者ですが、自己相関関数を求めたいんですが、フリーソフトありますか?また誤り最近傍法ができるフリーソフトありますか?
490ご冗談でしょう?名無しさん:2011/05/15(日) 19:55:15.18 ID:???
>>474>>475>>482
ありがとうございます。
まとめますと、もし「全て同時に」観測してしまった場合は、
物質が構造を保てなくなり崩壊する
「観測にタイムラグがある場合」は大きな系全体で見た場合波動性が失われることはなく
大きな変化はないということでしょうか
>>475さんのは逆に波動性しかなかったら・・・というお話ですね。参考になります。
491ご冗談でしょう?名無しさん:2011/05/15(日) 20:00:35.84 ID:Xb5M/oRE
>>472
482だけれど、「ある〜のです。」の部分を訂正。ある電子が他の電子から十分離れている(可弁別)とする。そのとき、
その電子の状態を2回続けて観測したとする。結可は1回目と2回目が同じとは限らない。すなわち、多体系を構成個々の電
子は定常状態には無く、状態の遷移に対応する波長の光量子を放出、吸収し得るから。そのことが電子の波動性を意味する。
状態の遷移が起こりうる(定常状態にない)=波動性
>>483
位置は連続的に変化しうる物理量だから、位置の固有状態という言い方はしない。それとある状態が確率論的に存在し得る
ことと、それを観測で厳密に求められることとは異なる。
492ご冗談でしょう?名無しさん:2011/05/15(日) 20:01:50.26 ID:g5kZwEO4
高校の宿題なんですが、
アルキメデスの証明した「平行四辺形を活用した『てこの原理』」について
まとめよ。
というものなんですがどなたか教えていただけませんか?
493ご冗談でしょう?名無しさん:2011/05/15(日) 20:39:28.99 ID:cMufIxFg
>>487 >>488
つくばではそういう保護者がいっぱいいて、
授業参観とかで「それは正しくない」とか言い出して、
授業中の先生を押しのけて講義を始め出したり、
ある意味モンペぶりに公立小中の先生達涙目らしい。
494ご冗談でしょう?名無しさん:2011/05/15(日) 20:43:42.62 ID:???
ハー、最近はいろんな権威を叩き潰すのが流行りだからな
だからと言って、子供の目の前で「先生」と言う権威を潰したらどうなるかも分からん奴が人の親か
495ご冗談でしょう?名無しさん:2011/05/15(日) 21:10:02.26 ID:???
496ご冗談でしょう?名無しさん:2011/05/15(日) 22:04:10.88 ID:BiguoPcU
>>494
いや、技術者・科学者特有の症状だろう

権威を潰すとかそういう動機じゃない。
497ご冗談でしょう?名無しさん:2011/05/15(日) 22:05:24.00 ID:g5kZwEO4
>>495
参考になりました
どうもありがとうございます。
498ご冗談でしょう?名無しさん:2011/05/15(日) 22:26:29.13 ID:???
>>496
いや、動機は何であれ結果的にそうなる事を予想できないのは、あるいは潰しても構わないと思ってるなら、まともな大人とは思えない
499ご冗談でしょう?名無しさん:2011/05/15(日) 22:37:52.17 ID:XA/6YS7r
円偏光フィルターというのを使うと、右巻きの波長を持った光、左巻きの波長を持った光だけを透過するメガネを作ることができ、
昔は赤と青のセロファンを貼ったメガネでやっていたような立体視をカラー画像でも見ることが出来ると聞きました。
すると、その眼鏡を左右逆にかけて3D映画を見れば、本来近くにあるように見えるべき部位が奥まって見える、通常とは逆の遠近感を持った映像が見られる、ということになるのでしょうか?
500ご冗談でしょう?名無しさん:2011/05/15(日) 22:56:35.46 ID:???
>>499
立体視は左右の視差だけでなく平面的な写真や絵画に見られる遠近感(大きさ、
影の付きかた、焦点深度など)からも得られるから、視覚的な混乱が起こる
場合のほうが多い。
もちろん被写体によって逆の遠近感が得られる場合もある。
501ご冗談でしょう?名無しさん:2011/05/15(日) 23:02:14.96 ID:???
>>500
なるほど、では視差以外の情報が多い、風景画のような映像よりも
単純な記号などの映像のほうが逆の遠近感を感じやすいんでしょうね。
ともかく、>>499は原理的には間違ってないわけですね。

どうも回答ありがとうございました。

502ご冗談でしょう?名無しさん:2011/05/15(日) 23:58:58.42 ID:???
物性の体心立方格子の逆格子を求める問題なんですけど
a1*=2π{(a2×x3)/a1・(a2×a3)}で

(a2×a3)={1/2a(x-y+z)}×{1/2a(x+y-z)}なんですけど
x×x=0より
=a^2/4((x×y)-(x×z)-(y×x)+(y×z)+(z×x)+(z×y))
=a^2/2(x×y+z×x) ←(これになる理由が分かりません。

何かベクトルで変換できるものってありましたか?
教えてください。
503ご冗談でしょう?名無しさん:2011/05/16(月) 00:00:14.67 ID:xBclSXBg
光の運動を考えていてふと思ったのですが、光の振動数が自然数である時は
イメージできるのですがそうでない時は難しいのです。
距離や時間の最小単位、となったとき素粒子や時間子?を考えますが同様に
振動の最小単位というのは考えられうるのでしょうか。また1より大きい時は、
有理数なり無理数なりでもそれは振動だと思えるのですが、1より小さい時は
つまり振動していない事になるのでしょうか。
504502:2011/05/16(月) 00:10:08.24 ID:???
自己解決しました恥ずかしい
505ご冗談でしょう?名無しさん:2011/05/16(月) 00:20:07.85 ID:???
>>503
例えば1.5Hzなら2秒でちょうど3回振動するじゃん
そんな感じでどうよ
506ご冗談でしょう?名無しさん:2011/05/16(月) 01:49:29.70 ID:???
すいません
素朴な質問ですがどうしたら物理が出来るようになるんでしょうか?
正直授業受けてても作業的な計算は出来るのですが何をしてるのかさっぱりです
これじゃあ応用が効かないので困ってます、、
507ご冗談でしょう?名無しさん:2011/05/16(月) 01:55:59.33 ID:???
授業で教師が言ってるのはいったい何の話なのか、真剣に考える事だと思う
要するに物理的イメージを掴むよう努力する事
そうすれば疑問がボロボロ湧いて来て、教師に質問してその答えから手がかりを得て、また考えてと言う感じかな
508ご冗談でしょう?名無しさん:2011/05/16(月) 03:04:00.32 ID:???
>>480 わかりにくくてすまん

| 机
----------- ←これが机だとする

|     □
-----□-----  ←斜めの2つの□が一つ携帯だとおもてくれ

机の端っこの線上にかいてるが、携帯は机から一部はみ出すように斜めにおいてある
この状態で携帯が振動すると

|   □
----□-----  ←まっすぐになる

さっきとは逆に傾けておいても

|  □
-----□-----  ←振動すると

|   □
----□-----  ←まっすぐになる

つまり
(゚д゚ ) → (゚д゚) ← ( ゚д゚)
509ご冗談でしょう?名無しさん:2011/05/16(月) 03:31:30.14 ID:???
ふしぎだと思うこと これが科学の芽です。

よく観察してたしかめ そして考えること これが科学の茎です。

そうして最後になぞがとける。これが科学の花です。

                       朝永振一郎(物理学者)
510ご冗談でしょう?名無しさん:2011/05/16(月) 11:38:43.42 ID:/f05HWMU
アルキメデスが証明した「平行四辺形を活用した『てこの原理』」なんだけど、URLor解説くれない?
511ご冗談でしょう?名無しさん:2011/05/16(月) 12:24:36.13 ID:???
>>508
摩擦分布の片寄り

動きやすさの片寄り

振動により大きく動く側と小さく動く側

まがーる
512ご冗談でしょう?名無しさん:2011/05/16(月) 12:26:55.33 ID:???
バイブは科学の陰茎です。

       朝永振一郎(物理学者)
513ご冗談でしょう?名無しさん:2011/05/16(月) 12:35:40.80 ID:???
>>510
外積でぐぐれ
514ご冗談でしょう?名無しさん:2011/05/16(月) 12:49:36.92 ID:???
>>508
携帯が帯磁してて南北になるとかのオチじゃねーだろうな。
机を任意の方向に回転してもそうなのか要確認。
515ご冗談でしょう?名無しさん:2011/05/16(月) 12:55:10.54 ID:???
>>510
ちょっと前にレスあるだろ
516ご冗談でしょう?名無しさん:2011/05/16(月) 13:32:09.84 ID:???
基本ベクトルと基底ベクトルの違いを教えてください。
基底ベクトルのほうはベクトル空間を意識した用語のようですが、
この場合はこっちを使ってはいけないなどの違いはありましょうか?
517ご冗談でしょう?名無しさん:2011/05/16(月) 16:43:59.92 ID:???
>>516
ggrks
518ご冗談でしょう?名無しさん:2011/05/16(月) 17:45:34.29 ID:???
素粒子物理はともかく高エネルギー加速器を作れば基本的には実験できるけど
ストリングを実験的に検証する方法って何かあるの?
量子コンピュータや何かみたいなSF的アイテムを持ち出しても実験的検証は無理に見えるんだが
519ご冗談でしょう?名無しさん:2011/05/16(月) 18:25:26.89 ID:???
>>517
ぐぐっても分かりませんよ。
520ご冗談でしょう?名無しさん:2011/05/16(月) 19:56:57.39 ID:???
強い電磁波は人体に悪影響がありますが、可視光線も強いと
何かしら影響があるのですか?失明以外での影響は?
521ご冗談でしょう?名無しさん:2011/05/16(月) 20:01:49.45 ID:???
>>520
レーザーメスって有りますね。
522ご冗談でしょう?名無しさん:2011/05/16(月) 22:10:09.87 ID:???
放射線には電磁波もある
523393:2011/05/17(火) 00:06:26.36 ID:???
遅くなってしまいましたが、
>>394
>>395の御二方ありがとうございます。

>>394さんの説明を読んでまた疑問が沸いてきたので質問したいのですが、
原子核からのクーロンポテンシャルについてはよく理解できました。
実際には他の電子からのポテンシャルも考えるとのことですが、他の電子からのポテンシャルもクーロン力(斥力)から生じるポテンシャルですか?

(24)式の後で1電子近似であると書かれているのにも関わらず波動関数ψはイオン殻と他の伝導電子の平均のポテンシャル中での動きだと書かれているのですか?
1電子近似なら他の伝導電子は関係ないのではないかと思ったのですが。なぜ平均なのかというのもイマイチピンと来ません。
よろしければ回答お願いします。
524ご冗談でしょう?名無しさん:2011/05/17(火) 01:20:54.96 ID:???
>>520
日焼け (正確には可視のみじゃないが) 〜PSのレーザは見える光だが紙を燃やせる。「やってみた」でググルといっぱい出る 人には向けない様に
525ご冗談でしょう?名無しさん:2011/05/17(火) 15:12:13.25 ID:6kHpmwZI
チョイ質問

タイムマシーンの原理についてなんだけど
幼稚園児に説明する感じでいうと(エレスカレータを使って)

時間の流れ&物質の移動限界スピード=エスカレーターの進行方向&スピード

現状の時間を止めることは不可能なので
エスカレーターを駆け上がれば、先の世界へ行けて

エスカレーターを駆け下れば過去の世界(元居た世界が限界?)に行ける!

って感じで説明したら正解かな?

教えて!物理学最強の人!
526ご冗談でしょう?名無しさん:2011/05/17(火) 16:08:38.92 ID:???
元ネタが何なのかによる

ドラえもんとハルヒとシュタゲじゃ
それぞれ理屈が違うっぽいし
527ご冗談でしょう?名無しさん:2011/05/17(火) 17:19:36.57 ID:6kHpmwZI
ん?元ネタ?

ワシが現役のときは
ウルトラQだった
ドラえもんは知ってるけど
528ご冗談でしょう?名無しさん:2011/05/17(火) 17:20:36.11 ID:???
>>525
普通に説明するよりも
「どうやったらタイムマシンを作れるか考えてみよう」
みたいな感じにする方がいいと思う

上手く話の流れを作れれば相対論や因果律の話まで持って行けるよ
529ご冗談でしょう?名無しさん:2011/05/17(火) 19:30:54.41 ID:???
高校の頃はx-tグラフで横軸が時間で縦軸が空間だったですけど
ミンコフスキー時空図ではどうして縦に時間、横に空間を取るのでしょうか?
どっちでも構わないとは思いますが、
何か便利なことがあるとしたら例えばどんなことでしょうか?
光円錐が横にならずに縦に書けるからでしょうか?
530ご冗談でしょう?名無しさん:2011/05/17(火) 20:24:43.46 ID:???
>>529
時間は虚数だから。

あと、直感的にも(数学的にも1行目と同じことだが)、ミンコフスキー時空を持ってくるとき、
速度は傾きになるので、等速が角度0度、すなわちx軸正方向に持ってくるほうが分かりやすい。
531ご冗談でしょう?名無しさん:2011/05/17(火) 20:40:40.06 ID:???
>>530
>時間は虚数だから。

これは空間を虚数にしてもかまわないです。
ようは光の世界線の長さが0になればいいわけですから、
x^2+ti^2=0
xi^2+t^2=0
どっちでも同じことです。
ただ、慣習的に時間のほうを虚数にしているだけで、空間のほうを虚数にしても全く同じことです。

>ミンコフスキー時空を持ってくるとき速度は傾きになるので

縦軸が時間で横軸が空間だと、その座標での世界線の傾きは速さにはならないです。
速さの逆数になります。
532ご冗談でしょう?名無しさん:2011/05/17(火) 20:41:25.10 ID:???
>>530
>時間は虚数だから。
一つの複素数の実部と虚部ってわけじゃないんだからこの説明は納得できない
533ご冗談でしょう?名無しさん:2011/05/17(火) 20:44:17.45 ID:???
2+1次元時空を考えるときにはふつうz軸があるところにt軸を持ってくるのが自然だから、
それをそのまま1+1次元時空に投影したらああなるんじゃないかな。
534ご冗談でしょう?名無しさん:2011/05/17(火) 20:53:15.10 ID:???
二重スリット実験で観測したら干渉縞ができないことは、
観測機器など周りのもの(あるいは場)まで含めて量子化すれば、状態ベクトルの収縮(射影)なしに説明できるという考え方は合ってる?
535ご冗談でしょう?名無しさん:2011/05/17(火) 23:15:32.04 ID:6kHpmwZI
>>534

間違ってるです
一般物理学で検索したら
出てくるです。
536ご冗談でしょう?名無しさん:2011/05/18(水) 01:06:03.08 ID:Nv1qSyuA
角波動関数と球面調和関数って同じものですか?
537ご冗談でしょう?名無しさん:2011/05/18(水) 01:11:23.16 ID:???
角波動関数は波動関数だけど球面調和関数は波動関数以外としても使う。
538ご冗談でしょう?名無しさん:2011/05/18(水) 02:31:31.71 ID:pmkNYHhh
カルノーサイクルについて質問です

同じ二つの温度の熱源を用いた場合、カルノーサイクルの効率を越える熱機関が存在しないのは何故ですか?
539ご冗談でしょう?名無しさん:2011/05/18(水) 02:35:32.63 ID:???
今しがた知恵袋で同じ質問を見た
マルチするな
540ご冗談でしょう?名無しさん:2011/05/18(水) 02:48:38.10 ID:pmkNYHhh
じゃあ こっちは発言撤回で
541ご冗談でしょう?名無しさん:2011/05/18(水) 03:16:20.29 ID:???
くだらない問題ですがなぜか分からないのでお願いします

有効桁数2桁で6×10^-7/√2の計算ですが√2は1.41なので計算すると4.25×10^-7なので4.3×10^-7になったのですが
答えは4.2×10^-7です

どこが間違っているのでしょうか
お願いします
542ご冗談でしょう?名無しさん:2011/05/18(水) 04:05:22.60 ID:???
1/√2 = √2/2
で計算してみれ
電卓とか使わないなら、分母を有理化するのがふつうだけど
1.4で割るのが間違いってわけではないはず
543ご冗談でしょう?名無しさん:2011/05/18(水) 04:29:16.77 ID:???
>>542
ありがとうございました
544ご冗談でしょう?名無しさん:2011/05/18(水) 04:51:28.57 ID:eVgVXGqg
超電導物質で、絶対零度を冷やしてみ、もうちょっと絶対零度が冷えるから。
超電導で、絶対零度を冷やしてみ、絶対零度が下がる
545ご冗談でしょう?名無しさん:2011/05/18(水) 09:15:58.86 ID:???
なんで物理板だけ電波君がいるんだろうねえ。
理論を理解しないでわかったつもりになれるからだろうね。
546ご冗談でしょう?名無しさん:2011/05/18(水) 09:53:17.94 ID:???
「絶対零度が冷える」の意味がさっぱりだな
547514:2011/05/18(水) 10:02:34.32 ID:???
>>508
方位磁針を自分の携帯に当てたら、長辺方向に帯磁してた。
別に机の縁に限らずどこでも同じ方角に落ち着くんじゃないのか?
548ご冗談でしょう?名無しさん:2011/05/18(水) 10:18:03.41 ID:???
平らな面に置いた時のバイブレーションの振動方向が面と垂直で、
かつバイブレーション位置が携帯電話の重心からズレた位置にあるから
じゃないかな?
549ご冗談でしょう?名無しさん:2011/05/18(水) 10:21:15.67 ID:???
同じ方向を向く説明になってないだろww
550ご冗談でしょう?名無しさん:2011/05/18(水) 12:58:02.08 ID:???
机の振動とケータイの振動の兼ね合いでそうなっちゃうんじゃないの
551ご冗談でしょう?名無しさん:2011/05/18(水) 12:58:57.38 ID:???
>>545
書き方からしていつもの「登記」ってやつだろ
552ご冗談でしょう?名無しさん:2011/05/18(水) 13:03:33.29 ID:???
> 兼ね合いで

そういうアバウトな説明は要らないから。
553ご冗談でしょう?名無しさん:2011/05/18(水) 13:15:46.21 ID:???
>>549
つまり、机の角にナナメの角度で打ち付けられるって事だよ
554ご冗談でしょう?名無しさん:2011/05/18(水) 13:37:15.35 ID:???
>>541
C/R^{1/2}についてルートを有効桁とそれ以外の部分に分ける
R^{1/2} = a+b、b>0
これを両辺に代入して有効桁を切り捨てれば
非有理〜C/a、有理〜{C/(R/a)}
となって不一致。近似値を真値で割れば
{C/(R/a)}/{真値}= 1 -b/(a+b)
{C/a}/{真値}= 1 +b/a
1/(a+b)<1/a だから、ずれは有理化したほうが小さい
555 忍法帖【Lv=23,xxxPT】 ◆VIVALDIPB0PH :2011/05/18(水) 15:12:56.80 ID:???
水に溶けにくいインスタントコーヒーが氷を入れると溶けやすくなるのはなぜですか?
556ご冗談でしょう?名無しさん:2011/05/18(水) 15:19:42.45 ID:???
そもそもインスタントコーヒーをアイスコーヒーに使えるの?
557 忍法帖【Lv=23,xxxPT】 ◆VIVALDIPB0PH :2011/05/18(水) 15:29:45.72 ID:???
使えるよ。だって実際やってみたら全部溶けたもん。
558 忍法帖【Lv=23,xxxPT】 ◆VIVALDIPB0PH :2011/05/18(水) 15:33:12.57 ID:???
種類によるかもね。俺が実際使ったのはパウダー状のやつだったよ。
559ご冗談でしょう?名無しさん:2011/05/18(水) 15:50:22.35 ID:???
>>555
外気温などを同じ条件にして、片方には氷入り、もう片方はただのにして溶かすと言うふうにきちんと実験してみてくれない?
YouTubeかなんかに動画でも上げてくれるともっと良い
そうすれば、誰か真剣に考えてくれるとオモ

氷を入れたらかき混ぜやすいから溶けやすいだけかもよ
560ご冗談でしょう?名無しさん:2011/05/18(水) 15:55:49.52 ID:???
1a. 水
2. 氷+水
3a. ビーズ+水
1b. 水の入った容器を氷水につける。
3b. {ビーズ+水}の入った容器を氷水につける。
561ご冗談でしょう?名無しさん:2011/05/18(水) 16:06:42.53 ID:???
室温の水と(氷抜きの)冷水のどっちが溶けやすいかを調べるのが先かな?
もしかしたら、粉末コーヒーは冷水の方が溶けやすいのかも知れないし
562 忍法帖【Lv=23,xxxPT】 ◆VIVALDIPB0PH :2011/05/18(水) 16:11:03.89 ID:???
>>559
最初はただの水でかき混ぜたところ、ある程度は溶けたんだけど、
三分の一ぐらいはどうしても溶けなくて、諦めて氷を入れて再び
かき混ぜたらあっという間に残っていたのが溶け始めたので、
不思議だなぁと思った次第です。
563 忍法帖【Lv=23,xxxPT】 ◆VIVALDIPB0PH :2011/05/18(水) 16:23:37.31 ID:???
ちなみに俺、文系なんでなるべく分かりやすく教えて下さい。お願いします。
564ご冗談でしょう?名無しさん:2011/05/18(水) 16:29:29.43 ID:???
どうでもいいけど文系の学生に
「足し算の『意味』を記述しなさい」
ってテストをしたのに結局同じ問題を出された理系の学生のがよっぽど出来が良かったって話を思い出した
565 忍法帖【Lv=23,xxxPT】 ◆VIVALDIPB0PH :2011/05/18(水) 16:35:24.27 ID:???
おバカな文系でごめんなさい。
566ご冗談でしょう?名無しさん:2011/05/18(水) 16:37:45.32 ID:???
>>563
ウーン、理由を考える前に追試をしてみる方が先だと思うので、そのインスタントコーヒーの銘柄等を教えてください
気が向いたら、追試してみて理由を考えるかも

でも、いちばん手っ取り早いのはそのメーカーにメールか何かで問い合わせる事だと思います
作った側はそう言う事を詳しく調べてて、何らかの溶けやすくなるような工夫を入れてるはずなので
567ご冗談でしょう?名無しさん:2011/05/18(水) 16:42:01.13 ID:???
>>557
そっかサンキュです。
余っているインスタントコーヒーの使い道ができた。
568ご冗談でしょう?名無しさん:2011/05/18(水) 16:43:30.00 ID:???
>>562
氷が溶けた分水が増えた?
569 忍法帖【Lv=23,xxxPT】 ◆VIVALDIPB0PH :2011/05/18(水) 16:47:44.11 ID:???
>>566
銘柄はAGFのBlendy北海道産ミルクのカフェオレです。
追試の方は残念ながら最後の一杯を数日前に作って飲んでしまったので出来ません。
570 忍法帖【Lv=23,xxxPT】 ◆VIVALDIPB0PH :2011/05/18(水) 16:49:26.75 ID:???
>>568
増えたと思います。
571ご冗談でしょう?名無しさん:2011/05/18(水) 16:50:17.66 ID:???
>>569
了解、カフェオレでしたか
572 忍法帖【Lv=23,xxxPT】 ◆VIVALDIPB0PH :2011/05/18(水) 16:51:29.87 ID:???
>>567
是非実験してみてください。
573ご冗談でしょう?名無しさん:2011/05/18(水) 16:54:03.91 ID:???
氷により表面の水が冷えたことによって濃度の低い水が
コップの底の方に落ちて溶けやすくなったんじゃないの
574ご冗談でしょう?名無しさん:2011/05/18(水) 16:56:01.69 ID:2qdtBsen
初心者でも読めるカタストロフ理論の良書を知りませんか?
575 忍法帖【Lv=23,xxxPT】 ◆VIVALDIPB0PH :2011/05/18(水) 17:09:51.23 ID:???
>>567
普通サイズのコップじゃなくて、居酒屋なんかでサワーを頼んだ時に出てくる
ような大き目のサイズのコップで実験してみて下さい。
576ご冗談でしょう?名無しさん:2011/05/18(水) 17:12:28.82 ID:???
将来理系分野の仕事で農業に関連した職に就きたいんだけど大学どこがいいんだろうか

農学部では北海道が有名らしいんだが
577ご冗談でしょう?名無しさん:2011/05/18(水) 17:37:24.38 ID:???
「氷を取り出している間にふやけた」が怪しいと思う。
そのまま混ぜていれば、氷を入れなくても同じタイミングで溶けたかも。
578ご冗談でしょう?名無しさん:2011/05/18(水) 17:38:29.61 ID:???
>>576
東京農業大学
東京農工大学
千葉大学園芸学科
579ご冗談でしょう?名無しさん:2011/05/18(水) 17:42:12.83 ID:???
>>576
北大に行けるだけの頭ないなら
帯広畜産大学ってのが北海道にある
580 忍法帖【Lv=23,xxxPT】 ◆VIVALDIPB0PH :2011/05/18(水) 18:09:20.56 ID:???
かき混ぜる際、氷が溶ける作用に何らかの理由で反応して、
一緒にコーヒーの粉末が溶け出した、と文系の俺の勝手な憶測は考えられない?
581 忍法帖【Lv=23,xxxPT】 ◆VIVALDIPB0PH :2011/05/18(水) 18:18:03.00 ID:???
まあ、その「何らかの理由」が何かが問題なだけに答になってないか。
582ご冗談でしょう?名無しさん:2011/05/18(水) 18:33:08.38 ID:???
氷の代わりに同じ量の水を追加したらもっとよく溶けるんじゃね?
583ご冗談でしょう?名無しさん:2011/05/18(水) 18:35:52.98 ID:???
>>552
では机の天板とケータイの接触箇所の変位が分かるようにハイスピードカメラで撮影してきて下さい
584ご冗談でしょう?名無しさん:2011/05/18(水) 19:25:58.69 ID:???
つか>>508>>511でFAじゃないの?
585ご冗談でしょう?名無しさん:2011/05/18(水) 19:53:25.34 ID:KmGPSiTw
物理学者になるには数学のなにを勉強すればいいですか
中3です
586ご冗談でしょう?名無しさん:2011/05/18(水) 19:54:50.12 ID:???
>>585
学校でやることは全部
587ご冗談でしょう?名無しさん:2011/05/18(水) 19:56:15.18 ID:???
>>585
2chをやめて真面目に教科書や参考書を読んで勉強する。
中学高校の内容なんて基礎中の基礎。
選択科目とかもあるけど
英数国地理世界史日本史物理化学生物地学
全部やるくらいの勢いがあればいい

こうやって全部やっていくと学問の間の関係が見えてきて自分が本当にやりたいのは何なのかが見えてくる。
絞るのはそこからでも遅くない
588ご冗談でしょう?名無しさん:2011/05/18(水) 20:24:15.30 ID:KmGPSiTw
教科書、参考書はたくさん読んでます。
それでもちょっとした疑問を多くの人から聞ける辞書の変わりに2chしてます。
物理の辞書があるかわからないし、何冊も買う必要がないと思うからです。
全く必要のないことは省きたいと思います。
589ご冗談でしょう?名無しさん:2011/05/18(水) 20:35:13.27 ID:4NLieKv4
>>588
物事を突き詰めて考えることはすごく重要だよ。
数学教師や理科教師に根掘り葉掘り訊くような
なぜなぜ質問魔になってみるのもいいかもね。
もしそれで不機嫌になったりするような教員がいたら
それはその教師が程度も能力も低いってことの証だから見切ってよろしい。
590ご冗談でしょう?名無しさん:2011/05/18(水) 20:36:54.45 ID:???
>>588
がんばりは認めるがそういうスタンスの人間は
「物理学者」どころか「まともな大人」にすらなれないのよ

人に聞く前に自分で考える。全ての基本だ。
とりあえずマジでなりたいなら>>587を実践してセンター試験レベルの問題なら全部9割は取れるようになれ。
中学生ならそれからだな
591ご冗談でしょう?名無しさん:2011/05/18(水) 20:39:02.11 ID:???
592ご冗談でしょう?名無しさん:2011/05/18(水) 21:15:01.87 ID:???
メコスジマスターになるにはメコスジ道のなにを修行すればいいですか
中出しです
593ご冗談でしょう?名無しさん:2011/05/18(水) 22:52:44.87 ID:LFK8KrLm
助けて貰えませんか?(マーチンゲール馬鹿論争です)

あまりに話が通じなくて意地になってます。dreamin_dreaminさんの数学と理論を冒涜した妄言を私が許せないだけで100%私怨なんですが、なんとか正論を伝えたいと思ってます。

前回の質問
http://oshiete.goo.ne.jp/qa/6735141.html
NO.5の補足に簡単な流れを書いてます。

http://detail.chiebukuro.yahoo.co.jp/qa/question
のkutibiru1003で論点をまとめてます。

他にも多数ありますが・・・
http://my.chiebukuro.yahoo.co.jp/my/myspace_coll

あまりにdreamin_dreaminさんの頭が悪すぎて会話としても成立しなくなってます。私が怒りすぎて馬鹿にした口調になってるせいもあるんでしょうがキャッチボールにもなりません。

そこで第三者の意見なら聞くんじゃないかと思い、こちらで質問してみました。

お手数ですが、dreamin_dreaminさんを説得して貰えませんか?

https://account.edit.yahoo.co.jp/registration?.s
こちらでyahooIDを取得→上のリンクから質問ページに飛んで回答して下さい。

宜しくお願いします。
594ご冗談でしょう?名無しさん:2011/05/18(水) 23:46:01.18 ID:???
棒の両端A.Bが固定され、中央の点Oで右向きの荷重Pが作用するとき、点Oの変位を求めよ。
棒の断面積をA、縦弾性係数をEとする。
左端点Aから点Oまでの距離はL/2、左端点Aから右端点Bまでの距離はLとする。

の問題がわかりません。

断面積と縦弾性係数を使うフックの法則の式を使うと思ったのですが考えてもわからなくて。
答えまでの式をどうやればよいか教えて頂けませんか?
595ご冗談でしょう?名無しさん:2011/05/19(木) 00:02:07.24 ID:???
>>585
高校までに習う数学は文字通り「全部」使う
596ご冗談でしょう?名無しさん:2011/05/19(木) 00:10:19.16 ID:CezSehOn
すみません。ageてID出して無かったのでもう一度書き込みますね(´・ω・`)

棒の両端A.Bが固定され、中央の点Oで右向きの荷重Pが作用するとき、点Oの変位を求めよ。
棒の断面積をA、縦弾性係数をEとする。
左端点Aから点Oまでの距離はL/2、左端点Aから右端点Bまでの距離はLとする。

の問題がわかりません。

断面積と縦弾性係数を使うフックの法則の式を使うと思ったのですが考えてもわからなくて。
答えまでの式をどうやればよいか教えて頂けませんか?
597ご冗談でしょう?名無しさん:2011/05/19(木) 00:18:10.24 ID:3BkO1yoX
馬鹿馬鹿しいが、過去スレにある桜島大根でつくる癌治療薬と、牛乳でつくった奴を、割合0.7:2.3で合わせると、ステージ2900から、ステージ2になる。毎日患部に3回貼って。後は、徐々に貼って、完治になる。
佐藤利奈&masa
登記
598ご冗談でしょう?名無しさん:2011/05/19(木) 00:27:40.10 ID:eLH4lVcq
流体力学の表面張力に関して悩んでいます。
表面張力は仮想した切口の単位長さあたりの引っ張り力のことをいうと習いました。

例題では球体の表面張力を求める時にちょうど真ん中を仮想断面としていたようです。
なぜ切口を仮想切口としなければならないのですか?
599ご冗談でしょう?名無しさん:2011/05/19(木) 00:55:39.58 ID:???
>>595
整数論とか平面幾何(代数使わないやつ)は使わないと思うけどなあ
600ご冗談でしょう?名無しさん:2011/05/19(木) 01:19:06.72 ID:3BkO1yoX
緊急を要したので、貼りを先に書いたが、
一般にはこれ、馬鹿馬鹿しいが、過去スレにある桜島大根でつくる癌治療薬と、牛乳でつくった奴を、割合1:2で合わせてかき混ぜて、常食として食べて、患部に張ると、3ヵ月後には、ステージ2900から、ステージ2になる。
毎日3回、食べて、貼って。後は、徐々に貼って、完治になる。
温泉治療も同時にすると、さらに効果がある。
佐藤利奈&masa
登記
601ご冗談でしょう?名無しさん:2011/05/19(木) 01:25:42.86 ID:oE/7pZGU
ヒッグス粒子が見つかったというのは本当なんですか、現在の状況を教えてください
602ご冗談でしょう?名無しさん:2011/05/19(木) 01:44:48.00 ID:3BkO1yoX
日本で、佐天さんは、癌ステージ6000から、ステージ1まで落ちて、回復しました。
603ご冗談でしょう?名無しさん:2011/05/19(木) 04:27:29.71 ID:???
特殊相対論のウラシマパラドックスの話なんですが、
地球から見たら宇宙船が遠ざかっている、
宇宙船から見たら地球が遠ざかっている、
で、宇宙船が地球に戻ってくるときに減速して地球に向けて加速する
しかし特殊相対論は加速は扱わないから、という話なんですが、
もし、宇宙船が地球に戻ってこない、つまり等速直線運動したままだと
どっちが年を取っているのか確認のしようがない
とのことですが、それは人間には確認できないだけであって、
事実は確定はしているのでしょうか?
それともシュレディンガーの猫みたいに確定していないのでしょうか?
604ご冗談でしょう?名無しさん:2011/05/19(木) 04:28:53.87 ID:???
確認しようがないというか
どちらも正しい
605ご冗談でしょう?名無しさん:2011/05/19(木) 05:32:27.76 ID:???
>>604
>とちらも正しい

例えば地球にAさんが残り、宇宙船にBさんが乗り、
Bさんは等速直線運動をして地球から遠ざかっています。
どちらも正しいとのことですが、
Aさんのほうが若い、かつBさんのほうが若い
ということでしょうか?
それって
Aさんのほうが若い、かつAさんのほうが若くない
つまり
AかつnotA
じゃないんでしょうか?
物理学は排中律を捨てたのでしょうか?
606ご冗談でしょう?名無しさん:2011/05/19(木) 05:35:10.87 ID:???
Aさんのほうが若い、かつAさんのほうが若くない
つまり
AかつnotA

1行目のAは人の名前で、3行目のAは命題の意味で書きました。
まぎらわしいので

Aさんのほうが若い、かつAさんのほうが若くない
つまり
PかつnotP

に訂正します。
607ご冗談でしょう?名無しさん:2011/05/19(木) 05:36:26.83 ID:???
視点が違えば見方も違うよ
だから相対性なんだよ
「Aから見るとBの方が若い」,「Bから見るとAの方が若い」というのはどちらも正しい
「誰から見てもAの方が若い,かつ,Bの方が若い」ということではない
608ご冗談でしょう?名無しさん:2011/05/19(木) 05:56:57.12 ID:???
>「Aから見るとBの方が若い」,「Bから見るとAの方が若い」というのはどちらも正しい

どう考えても排中律を捨てたとしか思えません。
Aから見ると、とか、Bから見ると、というのを考えないとします。
つまりAもBも互いに無関心だとします。
お互い、どっちが若いかなんて興味がないとします。
AもBも誰からも観測を受けていないとします。
その場合、どっちが若いかは確定しているのでしょうか?
609ご冗談でしょう?名無しさん:2011/05/19(木) 06:35:00.52 ID:???
>>596
その考え方であってますよ
610ご冗談でしょう?名無しさん:2011/05/19(木) 07:26:51.11 ID:???
>>603
特殊相対論は加速は扱わない、というのは、加速を扱えないという意味です。
そのため、加速を伴う思考実験では不完全な結果しか導き出せません。
一般相対性理論では加速した側が時間の収縮を体験します。
(ただしそのタイミングには2説あります)
またこの場合、人間には確認できないというのも誤りで、
遠隔地に地球と同じ慣性系に属するとみなせる星を想定した場合、
宇宙船の到着時に年をとっているのはどちらの系かという事を考える事ができます。
611ご冗談でしょう?名無しさん:2011/05/19(木) 07:57:56.20 ID:???
>遠隔地に地球と同じ慣性系に属するとみなせる星を想定した場合、
>宇宙船の到着時に年をとっているのはどちらの系かという事を考える事ができます。

その場合若いのはAとBどちらになりますでしょうか?
612ご冗談でしょう?名無しさん:2011/05/19(木) 08:07:12.91 ID:???
>>611
ある慣性系から別の慣性系へ加速した側、つまり宇宙船に乗ってるBです。
なお、一般相対性理論では加速した後の慣性系は区別できるという意見と、
加速による系の移行のみが時間のずれを作用させるという意見もあります。
この二つの解釈はニュートンのバケツに新たな論点を与えました。
613 忍法帖【Lv=23,xxxPT】 ◆VIVALDInSA :2011/05/19(木) 08:12:14.31 ID:???
おはようございます。>>555の者です。

今朝はちょっと寒かったのでホットにして飲んでしまいました。
昼間暖かくなったら再度実験してみます。
銘柄はHOT×ICE Cafe(ホット・バイ・アイス カフェ)
さっと溶ける「ミクロアグロ製法」粉末状のものです。
賞味期限がかなり前の'06.11.16.です。
製造者はコロラド株式会社CD、販売者は森永乳業株式会社です。

あと自分のおならはいい匂いだと感じるのはなぜですか?
614 忍法帖【Lv=23,xxxPT】 ◆VIVALDInSA :2011/05/19(木) 08:20:46.44 ID:???
重要なことをいい忘れてました。
最初に作った時もその次の時もコーヒーと一緒に砂糖を小匙3杯入れて、
かき混ぜていました。参考になるかどうか分かりませんが、それも
氷を入れたら溶けやすくなった要因の一つかなと思いレスしました。
615ご冗談でしょう?名無しさん:2011/05/19(木) 08:51:18.24 ID:???
条件の後出しするな、ボケ。
他に異物入れたら攪拌が促進されるのは明白だろ。
616 忍法帖【Lv=23,xxxPT】 ◆VIVALDInSA :2011/05/19(木) 09:00:31.04 ID:???
すみませんでした。ほんとにうっかり忘れてしまっていましたので・・・
617ご冗談でしょう?名無しさん:2011/05/19(木) 09:15:00.19 ID:???
氷や砂糖が撹拌子としてメカニカルに攪拌を促進するだけのことだろ。
618ご冗談でしょう?名無しさん:2011/05/19(木) 09:26:30.81 ID:???
また特殊相対論は加速を扱えないとか言ってるやつがいる
619ご冗談でしょう?名無しさん:2011/05/19(木) 10:01:27.82 ID:dHWBwgEq
課題で出た問題 が難しく、理解できなかったので助力お願いします。
2)空気を入れる前の熱気球とそれに 吊るす荷物の全質量が200kgである。
外部の空気は温度が10℃であり、圧力は1atmである
気球の風船部分の体積は400m^3である。
気球を上昇させるためには風船内の空気を何度に加熱しなければならないか。
(10℃における空気の密度は1.25kg/ m^3である。)
情けないですが、よろしくお願いし ます。
620ご冗談でしょう?名無しさん:2011/05/19(木) 10:30:02.59 ID:???
素人考えだと
(熱気球+荷物の全質量)の密度 < 空気の密度
になった時っぽいけど正解は如何に
621 忍法帖【Lv=23,xxxPT】 ◆VIVALDInSA :2011/05/19(木) 10:41:26.56 ID:???
>>617
お教え下さってありがとうございました。
622ご冗談でしょう?名無しさん:2011/05/19(木) 10:44:15.77 ID:???
>>619の者です。
答えは 472K
(気体をあためると、風船部分から空気は逃げていく。その軽くなった分で気球は浮き上がる)
とかいてありましたが、過程がさっぱりわかりません;
623ご冗談でしょう?名無しさん:2011/05/19(木) 11:30:51.66 ID:???
>>622
アルキメデスの原理を使えば良い
624ご冗談でしょう?名無しさん:2011/05/19(木) 11:34:40.32 ID:???
重力加速度に関連した質問をさせてください。
想定される状況は、二輪車が水平旋回する時です。

速度と旋回半径が決まっている場合
鉛直方向のGと水平方向のGの合成成分である
バンク角とほぼ一致する斜め下方向のGの算出方法です。

自転車がカントの付いた競技場を旋回する場合の
荷重倍率が知りたいのです。
空気圧セッティングの参考にしたいと思いまして。
625ご冗談でしょう?名無しさん:2011/05/19(木) 11:39:08.05 ID:???
>>608
相対論の理解は諦めて,それは神様に訊くといいよ
626ご冗談でしょう?名無しさん:2011/05/19(木) 11:46:58.40 ID:???
>>611
「遠隔地に地球と同じ慣性系に属するとみなせる星」というのは
結局は「Aから見て」どうなのかということを考えているんだよ
絶対的な基準なんて無いわけ
627624:2011/05/19(木) 11:53:11.50 ID:gbxwT2Fm
あ、すいません
質問はageるのが習わしなのですね。
628ご冗談でしょう?名無しさん:2011/05/19(木) 12:00:53.53 ID:???
>>624
三平方の定理
629ご冗談でしょう?名無しさん:2011/05/19(木) 12:04:27.69 ID:RelDIi0/
質問です
原子の大きさは1E-10m程度で
実際、電子顕微鏡で固体を観察すると原子が並ぶ様が
見えるらしいですが、地球内部のような高圧な場所では
原子は小さくなるのですか?また高温・高圧の場所でも
同様に小さくなった原子が衝突運動を繰り返している
状態なのですか?例えば原子の大きさが固体の時と
同じだと原子が動きようがないので液体にもなれないんじゃないかと
思うのですが。
630ご冗談でしょう?名無しさん:2011/05/19(木) 12:04:46.02 ID:???
(cosΘ)^-1
631 忍法帖【Lv=23,xxxPT】 ◆VIVALDInSA :2011/05/19(木) 12:24:32.17 ID:???
再実験したところ、

>銘柄はHOT×ICE Cafe(ホット・バイ・アイス カフェ)
>さっと溶ける「ミクロアグロ製法」粉末状のものです。

ホット兼アイス用で、砂糖を入れた状態でただの水のまま溶けてしまったので、
実験になりませんでした。
632ご冗談でしょう?名無しさん:2011/05/19(木) 12:30:37.11 ID:???
>>629
・原子が小さくなるというよりは、原子間距離が小さくなる。
 固体でも原子同士は離れている。
(というか、原子はスカスカなので、端っこが接触した後も、もっと近づける)
・実際、圧力をかけると液体は固体になる
でもその圧力でも、高温にすれば液体(か気体)になる。
「動きようがない」んじゃなくて、「動きづらい」だけだから。
633ご冗談でしょう?名無しさん:2011/05/19(木) 12:49:26.15 ID:???
圧力が強くなると電子と陽子がくっついて中性子星になるんだっけ?
634 忍法帖【Lv=21,xxxPT】 :2011/05/19(木) 12:51:17.12 ID:???
>>629
原子の大きさは電子殻の大きさだから周囲のポテンシャルによって大きさは変わるよ。
だけど地球内部の圧力なんかじゃ影響は0に等しいし、個体だ液体だなんだに原子の大きさなんか関係ない。
635ご冗談でしょう?名無しさん:2011/05/19(木) 12:58:20.69 ID:???
>>622
浮力ゼロ(10℃)の時の気球の風船部分の空気の重さ=体積×密度=500kg ……@

風船部分の空気で200kgの浮力を得られれば浮く
→@を300kgにすれば浮く
→@が300kgになるのは、体積の2/5が漏れ出すまで温めた時
→体積の2/5が漏れ出すのは、熱によって体積が5/3倍まで膨張した時

あとはシャルルの法則
283/400=χ/(400*5/3)
→283=χ*3/5
→χ=1415/3=471.6666...
636ご冗談でしょう?名無しさん:2011/05/19(木) 13:07:24.29 ID:???
>>629
電子縮退領域まで圧力が上がると、「原子が小さくなる」と言えると思う。
それ以下の圧力だと、結晶構造が変化してより密度の高い相となる。
637ご冗談でしょう?名無しさん:2011/05/19(木) 13:09:47.88 ID:???
岩石物質が地球内部の高温圧力下ではどのような状態に変化するか
ってのはまさに今研究が行われてる学問だな
ダイヤモンドアンビルとかで物質に圧力かけてどうなるかって実験してる
638622:2011/05/19(木) 13:14:14.47 ID:???
>>623さん >>635さん
ありがとうございました!!
639624:2011/05/19(木) 13:20:10.10 ID:gbxwT2Fm
>>628
鉛直方向は1Gですけど、水平方向のGはどうやって算出するんですか?
640ご冗談でしょう?名無しさん:2011/05/19(木) 13:24:36.44 ID:???
>>639
(125 * (速度[km/h])² ) / (15876 * 旋回半径[m]) [G]
641ご冗談でしょう?名無しさん:2011/05/19(木) 13:31:23.00 ID:???
508>>
あくまで仮説だけど、水の硬度が関係している可能性あり。

水道水が硬水の場合→粉末コヒーが解けにくい。
氷の外側が軟水化した状態→氷を入れて表面が溶け出すとコーヒーが解けやすくなる。

http://www.horiba.com/jp/application/material-property-characterization/water-analysis/water-quality-electrochemistry-instrumentation/ph-knowhow/the-story-of-conductivity/story-of-water-in-our-daily-lives/hard-water-soft-water/
家庭の冷蔵庫で作った氷を溶かしたとき、その導電率は元の水道水と同じで
しょうか。
水は凍るとき純粋な部分から、また表面から凍ります。このため氷の外側の
部分は不純物が少ないため導電率は小さくなります。逆に中央の部分は水道
水中のイオンや汚染物質が集まりますから、導電率も大きくなります。です
から、家庭の氷でジュースやウイスキーを飲むときは氷が完全にとける前に
飲むようにすれば、元の味を損なうことなく(多少薄くなるが)おいしく飲め
るんです。
642ご冗談でしょう?名無しさん:2011/05/19(木) 13:35:37.28 ID:???
>>639
速度が分かっている場合→遠心力の大きさ=>>640(「遠心力」でぐぐるだけで分かる)
地面に対して垂直に力がかかると想定する場合→合力の大きさ=>>630
643ご冗談でしょう?名無しさん:2011/05/19(木) 13:36:53.41 ID:???
>>640
円運動の向心加速度=速さ^2/半径
644ご冗談でしょう?名無しさん:2011/05/19(木) 13:37:17.51 ID:???
>>632 >>634 >>636 >>637
レスありがとうございます。

>>634 地球内部の圧力なんかじゃ影響は0に等しい

地球内部の圧力は凄いというイメージがあったのですが
よく考えたらあくまで万有引力の影響であり、
原子を構成するクーロン力レベルの力と比較すると
影響はないようなものという事なんですね。

つまり原子サイズ(原子間隔)にはさほど変化を与えず、
原子の周りの電子間の力が増加しているという感じでしょうか。
645641:2011/05/19(木) 13:42:41.76 ID:???
× 508>>
○ >>555
646ご冗談でしょう?名無しさん:2011/05/19(木) 13:43:12.14 ID:???
>>632 原子はスカスカなので、端っこが接触した後も、もっと近づける

原子核同士が接触するぐらい近づけば、それが圧力の限界でしょうか?
だとすれば、それは算定可能でしょうか?
647ご冗談でしょう?名無しさん:2011/05/19(木) 13:45:06.28 ID:???
>>641
もっともらしいウンチクだが、残念なことに裏付けがない。
というか、済んだ話をいつまで未練たらしく引っ張ってるの?
648ご冗談でしょう?名無しさん:2011/05/19(木) 13:47:45.28 ID:???
>>644
その万有引力の力で、矮星だと電子縮退領域まで、中性子星だと核心の縮退領域まで押し込められてる訳だが。
(それ以上だとブラックホールになっちゃうし)
649624:2011/05/19(木) 13:59:21.93 ID:gbxwT2Fm
>>640>>642
速度50km/h 旋回半径38m だとすると
水平方向に約0.52G
三平方の定理で約1.126G
となりましたけど、これでいいんですか?

>>643
う〜ん
値が違ってくるような気がするのですが
どうなんでしょう?
650ご冗談でしょう?名無しさん:2011/05/19(木) 13:59:49.94 ID:???
>>646
それが白色矮星のチャンドラセカール限界、それ以上の質量だと個々の原子核の形を保つ事が出来ず中性子星となる。
更に核子の縮退圧でも重力に抗する事が出来なければ、ブラックホールに成るしかないと考えられてる。

>>648を修正
×核心
○核子
651ご冗談でしょう?名無しさん:2011/05/19(木) 14:02:37.22 ID:???
>>649
>値が違ってくるような気がするのですが

すべからく 物理法則の式は単位を合わせないと正しい値とならない。
652ご冗談でしょう?名無しさん:2011/05/19(木) 14:03:25.08 ID:???
>>649
合ってるよ
653624:2011/05/19(木) 14:15:59.20 ID:gbxwT2Fm
>>651-652
経験則だけの運動脳にお付き合い頂きありがとうございました。
これからは理論も取り入れてやっていきます。
654ご冗談でしょう?名無しさん:2011/05/19(木) 14:30:16.76 ID:plUK0g7f
ゼーリガ−のパラドックスって何ですか、ググっても出てきません
655ご冗談でしょう?名無しさん:2011/05/19(木) 14:40:50.43 ID:???
>>654
ゼーリガーのパラドックスとは、 宇宙が一様かつ無限であれば、宇宙の
無限の数の星からの重力の総和も無限にならなければならないというも
のである。ゼーリガーはこのパラドックスに対して、アイザック・ニュ
ートンの万有引力の法則を修正することを提案した。
http://ja.wikipedia.org/wiki/%E3%83%95%E3%83%BC%E3%82%B4%E3%83%BB%E3%83%95%E3%82%A9%E3%83%B3%E3%83%BB%E3%82%BC%E3%83%BC%E3%83%AA%E3%82%AC%E3%83%BC
656ご冗談でしょう?名無しさん:2011/05/19(木) 14:48:04.46 ID:???
>>654
ていうか、ガ−の長音がマイナスな希瓦斯
657ご冗談でしょう?名無しさん:2011/05/19(木) 14:48:13.18 ID:???
>>650
良く分かりました。ありがとうございます。
658ご冗談でしょう?名無しさん:2011/05/19(木) 15:06:14.91 ID:???
>>655
どこがパラドックスなのかわからないんですが
総和は無限でもベクトル和はゼロだから無問題なのでは。
659ご冗談でしょう?名無しさん:2011/05/19(木) 15:10:38.09 ID:???
じゃ、やっぱり宇宙の大きさは有限なんだ
これで決まり!
660ご冗談でしょう?名無しさん:2011/05/19(木) 15:13:52.37 ID:???
無限だと地球の引力もキャンセルされるのでは
661ご冗談でしょう?名無しさん:2011/05/19(木) 15:16:52.63 ID:???
>>658
宇宙が一様かつ無限であれば一つの星の重力は僅かでも総和として地球は
あらゆる方向から無限に強く引かれるはずだというパラドックスだが膨張
宇宙の発見により回避された。
ttp://ja.wikipedia.org/wiki/%E3%83%91%E3%83%A9%E3%83%89%E3%83%83%E3%82%AF%E3%82%B9#.E5.AE.87.E5.AE.99.E8.AB.96.E9.96.A2.E9.80.A3
662ご冗談でしょう?名無しさん:2011/05/19(木) 15:45:33.04 ID:???
やはりあらゆる方向から無限に強く引かれてもベクトル和が0なら無問題な気がするが
663ご冗談でしょう?名無しさん:2011/05/19(木) 15:54:08.39 ID:???
>>662
地球と太陽との間の引力も含めてゼロになっても無問題なのですか?
664ご冗談でしょう?名無しさん:2011/05/19(木) 16:01:09.31 ID:???
何で地球と太陽との間の引力が0になるのかわからん
665ご冗談でしょう?名無しさん:2011/05/19(木) 16:25:44.03 ID:???
∞−∞=0なん?
∞+1≠∞なん?
666ご冗談でしょう?名無しさん:2011/05/19(木) 16:57:44.91 ID:???
ゼーリガーてのがバカだったんじゃないか
667ご冗談でしょう?名無しさん:2011/05/19(木) 17:22:57.78 ID:???
>>663
太陽系内の分布は一様じゃないんだから
偏重があるのは当たり前。そこはパラドックス以前の問題だ
668ご冗談でしょう?名無しさん:2011/05/19(木) 17:41:42.63 ID:???
>>667
どこまでが一様じゃなくてどこからが一様なんですか?
669ご冗談でしょう?名無しさん:2011/05/19(木) 17:46:58.25 ID:???
太陽系内の運動は太陽系内の計算で間に合うということ
(精密にやっても銀河系内でOK)
670ご冗談でしょう?名無しさん:2011/05/19(木) 17:51:41.92 ID:???
もし、大域的に見てあらゆる方向に無限の力で引っ張られるのが真実であり、かつ局所的に細かく見れば不均一であれば、要するに太陽系はバラバラに引きちぎられる事になるよね
671ご冗談でしょう?名無しさん:2011/05/19(木) 18:01:49.09 ID:???
>>669
太陽系とか銀河系とか人間の知識で構造がわかってる範囲は不均一で計算して、それ以上大きな構造は均一とするんですか?
知識が変わると範囲も変わるんですね。
672ご冗談でしょう?名無しさん:2011/05/19(木) 18:03:26.18 ID:???
変位電流は仕事をしない点が本質的に通常の電流と異なる

と習いました

複素数平面で考えたときに虚数部で表されることに対応していると思うのですが、
じゃあ結局この変位電流って一体何者なんですか?
スピンの元になってたり何か考えていくと訳が分からなくなる…
673ご冗談でしょう?名無しさん:2011/05/19(木) 18:11:38.09 ID:???
>>671
宇宙において、一様と言える分布をしてるのは銀河/銀河団単位でしょ。
それ以下の範囲では偏りを計算しなきゃいけないの。おk?
674ご冗談でしょう?名無しさん:2011/05/19(木) 18:13:55.50 ID:???
いや、数十億光年規模だと銀河の分布も構造を持つぞ
675ご冗談でしょう?名無しさん:2011/05/19(木) 18:52:39.19 ID:???
>>626
双子のパラドックスでググレ
676ご冗談でしょう?名無しさん:2011/05/19(木) 19:00:15.39 ID:???
>>675
よく、双子のパラドックスの説明で
「加速の向きを逆にするときの補正で辻褄が合う」
って説明を聞くけどアレがさっぱり理解出来ん
677ご冗談でしょう?名無しさん:2011/05/19(木) 19:08:16.84 ID:???
>>672
真空偏極と理解します。
678ご冗談でしょう?名無しさん:2011/05/19(木) 19:14:34.35 ID:???
>>677
キーワードを元にググったりしてみましたが思ったより複雑みたいですね…

変位電流→スピン

スピンノ向きが揃う→磁場発生→B=∇×A
の組み合わせとかが全く分からない…マクスウェル方程式や物性論の本に載ってそうな気はするんだけど
QEDまで行かないとダメなの?
679ご冗談でしょう?名無しさん:2011/05/19(木) 19:15:09.89 ID:???
>>676
それが理解できないってことは
相対性理論をまるでわかってないってこと
680ご冗談でしょう?名無しさん:2011/05/19(木) 19:18:41.85 ID:???
>>676
Uターンして所属する慣性系を大きく変えることが
それまで宇宙船が航行していた慣性系からの観測では
非常に強く加速したのと同義になる。
このときの加速で時間の収縮が観測され、
宇宙船主観では自分以外の全宇宙の時間が一気に進んだように見える。
もといた慣性系と加速した慣性系は区別できないという考え方だな。
681ご冗談でしょう?名無しさん:2011/05/19(木) 19:22:00.02 ID:VvgNGHB4
さいきん物理に興味を持った者です。
次の言葉を簡単に説明するとどういうことになるかわかりやすく教えてくれますか?
ちなみに高校までの物理は成績は良かったです。

●シュレーディンガーの猫
●マックスウェルの悪魔
●ハイゼンベルグの不確定性原理
682ご冗談でしょう?名無しさん:2011/05/19(木) 19:24:11.27 ID:???
683ご冗談でしょう?名無しさん:2011/05/19(木) 19:29:09.67 ID:VvgNGHB4
>>682
googleでいまいち分らないからここで聞いてみたのだ
684ご冗談でしょう?名無しさん:2011/05/19(木) 19:29:53.23 ID:VvgNGHB4
追加

●アインシュタインの特殊相対性理論
●アインシュタインの一般相対性理論
685ご冗談でしょう?名無しさん:2011/05/19(木) 19:33:37.86 ID:???
>>679
理解できてないから聞いてるんだろ?

>>683
何が分かってて何が分からないのか具体的に書けよ。
686ご冗談でしょう?名無しさん:2011/05/19(木) 19:37:46.27 ID:VvgNGHB4
猫は死んでるか生きてるかは箱をあけてみなければ分らないそうだけど、そんなのあたりまえじゃん。
あたりまえのことをなぜ騒いでるのかがわからない

気体分子は奇跡に近い偶然が起きればふたつの部屋に温度差ができるのはありえるのに
なにをさわいでるのかがわからない

位置と運動量が同時に確定できないそうだけど
それは人間から見てわからないのであって、神には分かってるんじゃ?
687ご冗談でしょう?名無しさん:2011/05/19(木) 19:38:32.21 ID:???
>>685
>理解できてないから聞いてるんだろ?

聞いてない
ただ理解できないって告白してるだけだし
688ご冗談でしょう?名無しさん:2011/05/19(木) 19:38:37.55 ID:VvgNGHB4
特殊と一般というのは、どのような観点でこれらの語を使ってるのかわからない
689ご冗談でしょう?名無しさん:2011/05/19(木) 19:40:09.86 ID:???
>>681
●量子力学の不思議さを「まあ量子力学だからw」で終わらせんな
●部屋っていつの間にか勝手に散らかってるけど、諦めんなよ。片付けたら片付くだろ。
●二兎を追うものは一兎をも得ず
690ご冗談でしょう?名無しさん:2011/05/19(木) 19:45:44.52 ID:???
>>687
質問スレで
「理解できない」
って言えば自動的に「理解できない。だから教えてくれ」
って文脈になると思うんだが

それと>>680サンクス。もう少し勉強してみる
691ご冗談でしょう?名無しさん:2011/05/19(木) 19:48:00.87 ID:VvgNGHB4
超ヒモ理論って何ですか?
ものを分割していくと究極の最小単位は原子じゃなかったですか?
原子はさらに小さくわけれてヒモからできているという認識で間違ってないですか?
692ご冗談でしょう?名無しさん:2011/05/19(木) 19:48:48.20 ID:VvgNGHB4
もしそうなら、
ヒモと、素粒子の陽子、電子、中性子、パイ中間子みたいなのもヒモでできているんですか?
693ご冗談でしょう?名無しさん:2011/05/19(木) 19:49:32.67 ID:???
>>686,688
●(量子力学がなければ、)箱の中の初期値が全て分かっているなら、
 猫がいつ死ぬかが計算で求められる。(ラプラスの魔)
 量子力学的に考えると、初期値が解っていても求められない。
●その偶然を意図的に起こす方法を考えたぜ、ということ
●「神には分かる」が何を意味するのか知らないが、
 測定技術の未熟さなどではなく、原理的に人間には測定しようがない
●特殊と相対の使い分けはぐぐれば分かる。
694ご冗談でしょう?名無しさん:2011/05/19(木) 19:51:55.21 ID:???

相対→一般
695ご冗談でしょう?名無しさん:2011/05/19(木) 19:54:15.98 ID:???
>>676
よく相対性理論に関してラフな文脈で「全ての観測者は互いに相対的であり平等」と書かれることがあるけれど、
少なくとも特殊相対性理論においてはそれは嘘で、特殊相対性理論では慣性系と非慣性系の厳然たる区別がある
互いに平等なのは慣性系同士のみ

よって、加速度運動する(=非慣性系にいる)双子の兄と、常に慣性系にいる双子の弟で物事が互いに相対的でないのは何ら不思議なことではない
696ご冗談でしょう?名無しさん:2011/05/19(木) 19:57:38.21 ID:???
というか、「全ての観測者は互いに相対的であり平等」とは言うもののあらゆる系にいる
観測者を平等に扱うのはとんでもなく大変なのでとりあえず一番簡単な
慣性系同士の関係を考えてみましょか、というのが特殊相対論だろ

若干書き方が変
697ご冗談でしょう?名無しさん:2011/05/19(木) 20:07:44.10 ID:???
>>686
確かに「人間に分からないだけで原理的には決まっている」と考えるのが素朴には自然だが、
色々調べていくとそう考えると今度は他のところ(局所実在性とか因果律とか)が奇妙なことにならざるをえないのが分かる
それでまあ色々な考え方があるわけだが。

そうした色々な考え方の主流の一つでは、「量子的な現象は人間が観測するまで本当に決まっていない」とする。
(あらゆる物事が観測するまで何事も決まっていないと言っているわけではない)
この考えは、量子的な現象っていうのが主に原子や電子などの物凄く小さい世界でのことなので、
「何か物凄く小さい世界では我々の常識では測れないやたら変なことが起こっているらしい」と無理矢理納得できなくもない。
しかしそこでシュレーディンガーが、
「たとえば原子核の崩壊という小さな世界の現象を、検出器と毒ガス発生装置を使って、猫の生死という日常サイズの物事とリンクさせられるよ?
このときどうなるの? 日常サイズの物事が観測するまで決まっていないなんて、そんな著しく非常識的なことがありうるの?」ということを言った。
これがシュレーディンガーの猫。
698ご冗談でしょう?名無しさん:2011/05/19(木) 20:12:39.94 ID:VvgNGHB4
なるほどぉ
さっぱり分りません

もう何か本を買って勉強することにしました
初心者がわかるのでオススメの本ないですか?
わたしは初心者といっても高校物理は完璧に理解してたので
ある程度は理系向けの本でも大丈夫です
でも基本的に「サルでもわかる系」の本で、イラストが豊富なの希望。
699ご冗談でしょう?名無しさん:2011/05/19(木) 20:15:50.93 ID:???
ブルーバックス
700ご冗談でしょう?名無しさん:2011/05/19(木) 20:17:25.90 ID:???
701ご冗談でしょう?名無しさん:2011/05/19(木) 20:21:11.82 ID:???
はいぱーあかでみっくらぼ
702 忍法帖【Lv=21,xxxPT】 :2011/05/19(木) 20:24:37.37 ID:???
>>644
万有引力とクーロン力は桁が違いすぎるから地球如きの重力じゃ原子サイズに影響は与えないと言える。
でも、原子間隔と原子サイズは別問題だから、
ギガパスカル、テラパスカルオーダーだと結晶構造のフェイズは変わってもおかしくない。
703ご冗談でしょう?名無しさん:2011/05/19(木) 21:17:12.58 ID:01WgL5+f
一般にはこれ、馬鹿馬鹿しいが、過去スレにある桜島大根でつくる癌治療薬と、牛乳でつくった奴を、割合1:2で合わせてかき混ぜて、上澄み液を取ると、細胞が年をとらないものができる。他の大根でつくる癌治療薬でもできるが、そっちのほうがいいかも。
登記
704ご冗談でしょう?名無しさん:2011/05/19(木) 21:20:42.97 ID:01WgL5+f
まあ、牛乳でつくった奴で、年を取らないが。
登記
705ご冗談でしょう?名無しさん:2011/05/19(木) 21:20:51.05 ID:???
>>675
君ググった程度で理解した気になってるの?
706ご冗談でしょう?名無しさん:2011/05/19(木) 21:22:45.34 ID:???
>>705
ググるのはともかくWikipediaは間違いを発見できるレベルになればまあ一人前かなと思ってる

そういう風に使えば結構良いメルクマールになるよ
707ご冗談でしょう?名無しさん:2011/05/19(木) 21:23:58.68 ID:???
>>706
ちょっと>>626の誤りを指摘してみてくれないか
708ご冗談でしょう?名無しさん:2011/05/19(木) 21:31:27.38 ID:???
>>707
>>626が何言ってるのかレス追跡するのが大変だったが
要するに
「地球から宇宙船に乗って加速しながら戻ってこなければ年の差を知る術はない」
って事だろ

同時性を考慮するなら地球から「今何歳か教えて下さい」信号を送って
それを宇宙船が受信して
「今何歳です」
って返信してそれを地球が受信すれば同じになってるはず

相対論は全くやってないから直感だけど
709ご冗談でしょう?名無しさん:2011/05/19(木) 21:47:11.99 ID:???
>>604=>>607=>>625=>>626=>>705=>>707だけど
>>626では質問者が>>608を認めないのにそれと大筋同じことを言ってる>>610に同じ質問を繰り返してることを指摘したつもりだった
ググるも何も真っ当な書籍で双子のパラドクスも勉強した

それにしても特殊相対論で加速度運動を扱うことが出来る場合と出来ない場合とが分かってない奴多すぎだろ
大学で単位とれたのかな
710ご冗談でしょう?名無しさん:2011/05/19(木) 22:00:32.89 ID:???
特殊相対論の誤解の大半は、
本来考察に「時間の遅れ」「長さの収縮」「同時刻の相対性」のうち2つ以上をいれなければならない問題を、
1つしか考えず他を見落とすことによって生じている
711ご冗談でしょう?名無しさん:2011/05/19(木) 22:20:24.47 ID:/4UYMnB/
電場E↑の中の電気双極子にかかる力は

F↑=qE↑(r↑+l↑/2) - qE↑(r↑-l↑/2) = (p↑・∇)E↑  (1)

ですが、どうやってもポテンシャルU=-p↑・E↑から計算した

F↑=-∇U = ∇(p↑・E↑)   (2)

と一致しそうにありません。何が悪いんでしょう?

(1)では力のx成分Fxには電場のx成分Exしか関係してきませんが、
(2)ではFxでもEx, Ey, Ezが全て入ってきます。
712鳳凰院凶真:2011/05/19(木) 22:27:14.41 ID:+aWwwFhM
CERNの素粒子衝突の加速度実験についてだが
本当にLHCでカーブラックホールができるのか?
そのカーブラックホールのリング状特異点の理論を詳しく聞きたい。
リフタ―を組み合わせることでタイムトラベルが現実的に可能なのかも教えて欲しい。
713ご冗談でしょう?名無しさん:2011/05/19(木) 23:06:39.83 ID:???
>>711
電場のrotがゼロであることを使うか、
一回ポテンシャルに直してから計算すれば同じになる。
714ご冗談でしょう?名無しさん:2011/05/19(木) 23:20:41.05 ID:???
>>713
>電場のrotがゼロであることを使うか、

なるほど。これを使うですか。やってみます。
ありがとうございます。
715ご冗談でしょう?名無しさん:2011/05/19(木) 23:23:50.56 ID:???
>>603>>605>>606>>608>>611です
やっと理解できました。
人間の数だけ時間軸空間軸がある
これで排中律を捨てずに済むことに気づきました。
つまり
慣性系Aからみたら「AさんはBさんより若い」
慣性系Bからみたら「BさんはAさんより若い」
そして
慣性系AとBをひっくるめて考えるとPかつnotPとなり排中律を捨てざるを得ませんが、
座標系というのは人の数だけあり、
慣性系A内だけで考える分には排中律を捨てる必要はない
慣性系B内だけで考える分には排中律を捨てる必要はない。
何が言いたいかといいますと多世界解釈みたいなものです。
そう考えていくと、僕らはいっしょに食事をして会話して楽しんでいても
僕らは所詮違う座標系にいるので実は会話などしていないのです。
Cさんの座標系にいるDさんと、Dさんの座標系にいるDさんは別人です。
Cさんと会話しているのはCさんの座標系にいるDさんであり、
CさんはDさんの座標系のDさんとは会話していないのです。
寂しい話ではありますが、排中律を捨てるより、多世界解釈を選択したほうが合理的です。
排中律を捨てたら背理法が使えないですから、背理法で証明した定理はすべて破棄せざるを得ません。
それにしても目の前にいる人は本当は違う座標系にいるなんて悲しい気がします。
716ご冗談でしょう?名無しさん:2011/05/19(木) 23:25:27.98 ID:???
ラグランジュ点って何故そこに天体がないのですか。長い年月の間には岩石や塵が集まって小さくとも天体が出来ていてもよさそうに思えるのですが。
717ご冗談でしょう?名無しさん:2011/05/19(木) 23:26:10.39 ID:gLgnUvym
すいません
今年から大阪の大学に通っている者ですが
物理に詳しい皆さんに助けていただきたいことがあります
最初に下のサイトにある動画を見ていただきたいんです

http://www.hiroiro.com/movie/1642.html

この現象を数式で表したいのですが、大学1年程度の知識で表すことはできますか
具体的には、ボールにメープルシロップを入れた状態と入れてない状態での坂を転がる速さを数式で表し、数学的に速さの違いを比較したいんです
転がり摩擦とかの知識が必要になると思うのですが、他に習得すべき知識などがあれば教えていただきたいです
お願いします
718ご冗談でしょう?名無しさん:2011/05/19(木) 23:44:57.92 ID:???
シロップ入りのは重心が回転軸から外れていて、
球体の中心から重心点 (内部の鉄球の位置) までを結ぶ位置ベクトルと、
球体に及ぶ重力のベクトルのとで、ベクトル積をすると 0 でない力のモーメントができる.。これが転がりを阻害している。
719ご冗談でしょう?名無しさん:2011/05/19(木) 23:53:27.74 ID:???
>>715
精神が異常ですね
720ご冗談でしょう?名無しさん:2011/05/19(木) 23:55:37.66 ID:gLgnUvym
>>718
少し分かった気がします
さっそく数式を立ててみたいと思います
ありがとうございます
721ご冗談でしょう?名無しさん:2011/05/19(木) 23:59:49.37 ID:VvgNGHB4
さっき庭に隕石が落ちてきたんですが
放射能を持ってる可能性ってありますか?
722ご冗談でしょう?名無しさん:2011/05/20(金) 00:00:35.39 ID:???
あるかないかで言えば可能性はあるよ
723ご冗談でしょう?名無しさん:2011/05/20(金) 00:01:46.61 ID:???
放射能どころか、なんか生物が出てくる可能性がある
724ご冗談でしょう?名無しさん:2011/05/20(金) 00:02:31.26 ID:VvgNGHB4
>>722
どのくらいの確率?
725ご冗談でしょう?名無しさん:2011/05/20(金) 00:03:14.34 ID:VvgNGHB4
>>723
冷めてから調べてみますわ
726ご冗談でしょう?名無しさん:2011/05/20(金) 00:06:24.83 ID:???
>>724
ほぼ100%
727ご冗談でしょう?名無しさん:2011/05/20(金) 00:08:29.35 ID:???
>>716
トロヤ群 で調べてみて
728724:2011/05/20(金) 00:10:21.51 ID:BfpviW+X
>>726
釣れた(藁)
729ご冗談でしょう?名無しさん:2011/05/20(金) 00:12:17.48 ID:???
おめでとう
セシウム茶あげる
730ご冗談でしょう?名無しさん:2011/05/20(金) 00:31:28.50 ID:FAyMQfP/
当方高校一年生です。
今力学分野の「剛体に働く力の作用とつり合い」について学んでます。

力のモーメントの部分でわからないことがありまして、
例えば 「壁に棒を立てかけるとき、棒を倒れないようにするための棒と床のなす角θの条件を求めよ。」という問題です。

水平方向、鉛直方向の力のつり合いについての式は理解できるのですが、
モーメントのつり合いに関して立てる式がよくわかりません。
棒にかかる重力と壁からの垂直抗力について式を立てているのはわかるのですが、
壁からの垂直抗力は正として処理して良いのですか?

731ご冗談でしょう?名無しさん:2011/05/20(金) 00:39:55.89 ID:cKeyL7Ep
文系卒です、自分の位置が気になったので質問しに来ました
アメリカ風に住所を逆から書いていって、最終的にここまで調べたんですが、
これ以降の住所を書こうとするならば、どこまで書けばいいんでしょうか?
乙女座超銀河団が止まってしまいました(それ以降どこで調べればいいんだろ?)

最終的に宇宙って書いて終わると思うのですが、書けるところまででいいですので誰か追記してください

アパート○○5階 ○町 ○番地 ○○市 東京都 日本 地球 太陽系 オリオン腕 天の川銀河 →
→ 局部銀河群 乙女座超銀河団 
732ご冗談でしょう?名無しさん:2011/05/20(金) 00:45:36.31 ID:???
>>730
力の向きによって正負違う
つーか教科書見るなり教師に聞くなりしたほうが早いし確実だと思う
733ご冗談でしょう?名無しさん:2011/05/20(金) 01:00:39.78 ID:???
>>732
すみません。ありがとうございました。
734ご冗談でしょう?名無しさん:2011/05/20(金) 01:30:12.42 ID:c/IpaAzQ
ヒッグス粒子の反粒子は何ですか?
735ご冗談でしょう?名無しさん:2011/05/20(金) 05:27:11.16 ID:???
例えばエネルギーの量子化というのがありますが、
エネルギーの単位はkg*m^2/s^2です。
そうなると質量、空間、時間も量子化から逃れることはできないと思います。
何故なら質量、空間、時間という物理量が連続量で、
それらの掛け算割り算したものであるエネルギーが離散量であることはありえないからです。
そう考えると、あらゆる物理量は離散量であるから、グラフを書くと任意の点で不連続、
つまり微分積分が使えない、ということになります。
具体的にはx-tグラフのt軸、x軸は実数軸ではなく整数軸ということになります。
つまり自然を数学で記述することはできないということになります。
かなりモチベーションが下がってしまいました。
もう物理は単位だけ取って文系就職しようか迷ってます。
アドバイスお願いします。
736 忍法帖【Lv=24,xxxPT】 ◆VIVALDInSA :2011/05/20(金) 05:42:20.19 ID:???
>>567さん
実験は上手く出来ましたか?こっちは再度実験しましたが、>>631の通りで
出来ませんでした。一応、砂糖も一緒に入れてやってみて下さい。砂糖の件に
関しましては、>>613-617の通り叱られましたので条件を同じにするためにも、
絶対に入れて実験してみて下さい。
737ご冗談でしょう?名無しさん:2011/05/20(金) 07:24:23.98 ID:???
>>735
そう思ってんのは君だけだと思う。
そういう意味じゃさっさと辞めちまえば、とも言えるし
辞める必要はない、とも言える。

どうぞご自由に。
738ご冗談でしょう?名無しさん:2011/05/20(金) 07:29:37.46 ID:???
>>735
> 何故なら質量、空間、時間という物理量が連続量で、
> それらの掛け算割り算したものであるエネルギーが離散量であることはありえないからです。
ここからして意味不明だけど、
そもそも粒子数みたいな離散量しかとらない変数で微分したって、適切な条件下では何の問題もない
数学でε-δ習わなかったのか?
739ご冗談でしょう?名無しさん:2011/05/20(金) 07:33:41.12 ID:???
>>735
>例えばエネルギーの量子化というのがありますが、
「エネルギーの量子化」の意味をよく学んだ方がいいと思う

そして仮に前半、君が言っていた通りだとしても、
> つまり自然を数学で記述することはできないということになります。
これはおかしい。離散量は数学で扱えないのか?
740ご冗談でしょう?名無しさん:2011/05/20(金) 10:00:05.77 ID:???
>>735
そもそもエネルギーは連続量もとりうるので出発点から破綻しているが、それはさておき
連続量の掛け算割り算で離散量になることはないというのも間違っている。
実際、位置rと運動量pはいずれも連続量だが、軌道角運動量L=r×pはh/2πの
整数倍しか取れないという例がある
741ご冗談でしょう?名無しさん:2011/05/20(金) 10:08:38.99 ID:YYoD120X
アロエなど、薬草のパッチ
登記
742ご冗談でしょう?名無しさん:2011/05/20(金) 11:51:09.16 ID:???
>>686
>猫は死んでるか生きてるかは箱をあけてみなければ分らないそうだけど、そんなのあたりまえ

観測者がまだ知らないというだけでなく、
箱の中で、ネコの死んだ状態とネコの生きた状態が重ね合わせられていると考える。
相反する状態を同時に「重ね合わせる」というのが理解しづらい。

>気体分子は奇跡に近い偶然が起きれば

奇跡に近いくらい起きないということを示した想像上のキャラクタ。
その悪魔が頑張れば、不思議なことがおきるということ。

>位置と運動量が同時に確定できないそうだけど
>それは人間から見てわからないのであって、神には分かってるんじゃ?

一般的な量子力学の数式ではそうは考えない。

だが、そうではなくてやはり人間の知識不足だというのが、「隠れた変数」という考え方。
「隠れた変数」がどういうものか?それは存在しうるかを検討したのが、「ベルの不等式」。
現在では、「隠れた変数」が存在するにしても、非局所的な非常に奇妙なものを仮定する必要がある。

>特殊と一般というのは、どのような観点でこれらの語を使ってるのかわからない

重力のない特殊な例を扱うのが特殊相対性理論。
重力がある場合も視野にいれたのが一般相対性理論。

で、いいかな?とりあえず。
743ご冗談でしょう?名無しさん:2011/05/20(金) 14:01:00.31 ID:???
>>729
ありがとうございます。
トロヤ群も天体になるぐらい集積していないのですね。
さらに月と地球のラグランジュ点は霞ぐらいしか無いとありました。
なぜなんでしょうか。月と地球のフォーメーションができたときからラグランジュ点は存在しますよね。それ以後何十億年もたっているのだから小さめの小惑星クラスの岩石かあるいは大量の塵の塊ぐらいは捕らえられていてもいいように思うのですが。
それともラグランジュ点に関する私の理解が間違っているのでしょうか。一度そこで静止した物体でも摂動とかで出ていくことがあるのでしょうか。
744ご冗談でしょう?名無しさん:2011/05/20(金) 15:01:11.69 ID:???
科学哲学の話でアレなんだけど
ある日突然神様が気まぐれで物理法則を組み替えて
例えば太陽が西から昇ってくるようになるとかそういうことが「有り得ない」と言い切れるだけのものって現代科学の枠組みにはあるの?
ただの経験則に信頼性を与えてるものは一体何なの?
745ご冗談でしょう?名無しさん:2011/05/20(金) 15:28:37.73 ID:???
じゃあ空間を量子化しない理由はなんでですか。
実数軸が使えなくなるからでしょ。
かくしても無駄だぜ。
746ご冗談でしょう?名無しさん:2011/05/20(金) 15:34:30.19 ID:???
エネルギーの量子化はある系の持つエネルギーが離散化されるということ。
同じ意味である粒子の持つ座標が離散化されることはある。
「空間」の離散化は少なくとも量子力学では扱えない
747ご冗談でしょう?名無しさん:2011/05/20(金) 15:36:10.10 ID:???
>>744
>ただの経験則に信頼性を与えてるものは一体何なの?

別にないだろ。
だから、再現性とか普遍性が重要視されるんだろ。

あるいは、神様はそんな意地悪なことしないと神様を信じているのでは?
748ご冗談でしょう?名無しさん:2011/05/20(金) 15:41:18.73 ID:QQ0gyji/
運動方程式を立てる際,
剛体の振り子は,重心の並進運動と回転運動を考えて運動エネルギを考えますが,
質量のない棒に取り付けられた質点の場合は,なぜ並進運動を考慮しないのでしょうか.

cf)
剛体:T=1/2 mv^2 + 1/2 Iθ'^2
失点:T=1/2 Iθ'^2

Tは運動エネルギ,mは質量,vは速度,θ'は角速度,Iは慣性モーメントです.

749ご冗談でしょう?名無しさん:2011/05/20(金) 15:49:58.85 ID:???
>>743
ラグランジュ点のポテンシャルは、すり鉢状の穴なので、
そこにいったん入り込んでも、はじめに十分なスピードを持っていれば外に出る。
いったん底で安定すれば、(外力を受けない限り)外には出られない。

>>744
「グルーのパラドクス」と呼ばれている問題だけど、
実際、「有り得ない」とは言い切れない。
科学の役割は「最もあり得そうなこと」を見つけることであって、
全ての科学的な命題は断定はできない。
750ご冗談でしょう?名無しさん:2011/05/20(金) 15:59:41.10 ID:???
>>744
法則自体が代わるかどうかというのは、科学の枠組みよりも上位のメタの話だから
現代科学を使って語ることはできない。
経験則に信頼を与えてるのは、現代科学が発展してから今までは法則が変わってなさそうだということ。
科学法則が時間と共に劇的に代わることがありえない、というのは原理的に主張しできないことだから、
科学の立場としては科学法則は変わらないと仮定している。
751ご冗談でしょう?名無しさん:2011/05/20(金) 16:18:26.86 ID:???
>>749
全然>>743の疑問への答えになっていない件
752ご冗談でしょう?名無しさん:2011/05/20(金) 16:31:09.16 ID:???
>>749
>>750
ふーむ、じゃあメタ的な解釈だけど
この世界は神が毎日全て構築し直していて一日一日で成り立つ法則が本当はバラバラ何だけど
法則性自体はあるからそれに気づくことすら出来ない
って考えもありな訳か
753ご冗談でしょう?名無しさん:2011/05/20(金) 17:30:12.87 ID:???
> ラグランジュ点のポテンシャルは、すり鉢状の穴なので、

違うよ。
L1,2,3は鞍部状、L4,5は凸状ポテンシャルになっている。
L4,5が安定と言われるのは摂動を与えるとコリオリ力で
ラグランジュ点を周回する軌跡を描くから。
754ご冗談でしょう?名無しさん:2011/05/20(金) 17:37:26.66 ID:7rND3Ew2
今獨協ではやぶさ見てきた。
展示パネルに大気圏突入時は空気(大気?)の運動エネルギーが急激に熱にかわり、ガスの膜で加熱を抑える(断熱と解釈)ってあったんだけど詳しい方解説お願いします。
自分ではカプセルの運動エネルギーが大気摩擦で熱に変わりカプセルが直接加熱される(ガス膜形成なら摩擦減のため)と思っていたんだけど。
755ご冗談でしょう?名無しさん:2011/05/20(金) 17:54:35.92 ID:???
摩擦よりも空気自体が圧縮されての加熱が大きくて、
その熱い空気の層に直接触れないようにガス膜を形成して対処、じゃなかった?
人間が寒いときにふかふかの服で空気の層を造り寒気を遮るのと同様に。
756ご冗談でしょう?名無しさん:2011/05/20(金) 17:58:08.07 ID:BfpviW+X
「超ヒモ理論」って一時期流行ったけど
その理論は現代解明されてきてる「原子や素粒子の構造の理論」とcompatibleなんですか?
それとも独自が独自の道を突き進む理論なのですか?
757ご冗談でしょう?名無しさん:2011/05/20(金) 19:18:24.81 ID:???
>>754
乾いたフライパンを熱しまくってから水滴を落とすと逆に蒸発しにくいのと同じ原理。
ガスのフィルタが表面を覆って断熱する。
専門的には耐熱シーリングっていう。
758ご冗談でしょう?名無しさん:2011/05/20(金) 20:15:26.52 ID:???
>>749
グルーのパラドックスとはまた別だろ
あれはどういう帰納を全うなものだとみなすかという問題

科学でやってる帰納はあんな直観的におかしなものではないし
759ご冗談でしょう?名無しさん:2011/05/20(金) 20:36:52.82 ID:???
>>758
科学は帰納的な論証が根底にあるし、
(帰納的に導いた原理から演繹することはあるど、土台は帰納)
>>744にある「経験則」は帰納的に導いた法則という意味じゃない?

あと、その「直感的におかしな命題」と「科学の命題」が、
論理的には区別できないから、パラドクスなんじゃない。
760754:2011/05/20(金) 21:00:39.27 ID:7rND3Ew2
いや、ガスによる断熱の仕組み自体は知ってるんだけど、上で書いた持論の摩擦熱の場合には熱伝導は関係無いだろうってことです。
で、圧縮加熱は気づかなかったけど、言われてみればごもっともです。
ただこれも「空気の運動エネルギー」って言葉では説明されないと思うのですが
761ご冗談でしょう?名無しさん:2011/05/20(金) 21:05:40.30 ID:BfpviW+X
超ヒモ理論は?
762ご冗談でしょう?名無しさん:2011/05/20(金) 21:23:07.01 ID:???
>>760
カプセルの静止系で見る
763ご冗談でしょう?名無しさん:2011/05/20(金) 21:28:40.59 ID:???
>>759
マトモだと思っていい帰納の様式と明らかにヤバい帰納の様式の間に言語的に境界を引くことはできなくても、
直観的には明らかに分類できて(だからパラドックス)、
しかも科学で使ってる帰納は全部、とりあえずマトモだと思える帰納に分類できるわけでしょ?

そういう投影可能性の問題とまた別件で、
直観的に明らかにヤバい帰納を取り去ったあとのとりあえずマトモそうな帰納でも、
論理的には成立する根拠がないじゃんってのが>>744の言ってる自然の斉一性の問題
764ご冗談でしょう?名無しさん:2011/05/20(金) 21:37:16.79 ID:BfpviW+X
わたしゃ聞いておるのよ
超ヒモ理論と原子内部構造の研究は、お互い両立できるのかと
765ご冗談でしょう?名無しさん:2011/05/20(金) 21:40:51.81 ID:???
>>763
「太陽は2011年までは東から登って、2012年からは西から昇る」は
直感的にまともなの?
766ご冗談でしょう?名無しさん:2011/05/20(金) 21:51:52.92 ID:???
>>764
できるよ
767ご冗談でしょう?名無しさん:2011/05/20(金) 21:54:00.46 ID:BfpviW+X
>>766
釣れた(ワラ
768ご冗談でしょう?名無しさん:2011/05/20(金) 21:54:26.97 ID:BfpviW+X
超ヒモ理論と原子内部構造の研究
どう違うのですか?
769 ◆ly/TAatdog :2011/05/20(金) 22:17:42.48 ID:???
>>748
慣性モーメントを考える際の中心点の取り方が食い違っているために
見掛けが異なっているのでは?
同じ I という記号を使っているけど、
片方は物体の重心、もう一方は振り子の支点を中心とした慣性モーメントを使っているのでは。
770754:2011/05/20(金) 22:19:00.27 ID:7rND3Ew2
そういうこと?
普通に考えるとカプセルの運動エネルギーが熱に〜ってなると思うんですが、あえてカプセル視点で書くのって何か意味というか意図があるんですかね。
それとも、この表現に違和感ある自分がおかしい?
771ご冗談でしょう?名無しさん:2011/05/20(金) 22:22:28.54 ID:???
>>765
その命題が偽だと断言できるものが現代科学のパラダイムにないって話だよ
772ご冗談でしょう?名無しさん:2011/05/20(金) 22:28:12.88 ID:???
>>765
そういう帰納はマトモじゃないから、なぜマトモじゃないと言えるかってのは議論されるべきとしても、
そういう帰納によって得られた法則が信頼できないのは特に議論の余地はないでしょ?
だから最初からそんな変なのは議論の枠に入れてないわけ

そうじゃなくて、もっとマトモで信頼できると思ってる帰納は、本当に信頼できるのかってのが問いでしょ
773ご冗談でしょう?名無しさん:2011/05/20(金) 22:33:56.31 ID:???
>>753
重力は距離が大きくなるにつれて一様に減少していくのに2つの天体からの重力の合成のポテンシャルが凸形になるのは不思議な気がする。
774ご冗談でしょう?名無しさん:2011/05/20(金) 22:36:59.03 ID:QaoJ2xkM
緯度がθである地球上の場所Pの地球の中心からの距離の計算の仕方を教えてください。
当然、地球は回転楕円体とします。つぎの二点で困ってます。

1. 緯度の定義を調べると、天頂の方向と赤道面のなす角、つまり、接平面の法線方向と赤道面のなす角という事で、
法線を地球の中心側に延長した線は地球の中心を通らない。よって、地球の中心とPを結ぶ線が赤道面となす角αとθは等しくない。

2. 楕円上の点を表すのはx=Acosβ, y=Bsinβと媒介変数表示で書くのが簡単だが、このβは外接円上の点と中心を結ぶ角度なので、αとも緯度θとも一致しない。

ということで、θ、α、βの三つの角が出てきてどう整理していいかわからなくなりました。
775ご冗談でしょう?名無しさん:2011/05/20(金) 22:43:03.85 ID:???
経験的に正しい、の言い換えに過ぎないけど、もう少し踏み込んで表現するとしたら、
我々がこうして今ここに存在できている限り、そのような世界ではある程度の、理想的には少ない誤差の範囲で物理法則は変わらない。
という人間原理が働いているような気がする。
776ご冗談でしょう?名無しさん:2011/05/20(金) 22:44:31.01 ID:BfpviW+X
北半球が夏のとき南半球は冬、
南半球が夏のとき北半球は冬、と言います。

ではその境界の赤道上で行ったり来たりを繰り返すと
暑くなったり寒くなったりを経験できますか?

もし万が一できないなら、その理由を物理学的観点でサルでもわかるように説明してくれますか?
777ご冗談でしょう?名無しさん:2011/05/20(金) 22:48:15.59 ID:???
できない。
夏か冬かというのは太陽からの光がどの程度降り注いでいるかということで決まるが、
その日射量は連続的に変化する。つまり、赤道の少し北側と少し南側では日射量は大差ない。
赤道から遠く離れて例えばイギリスと南アフリカを往復すれば体験できる
778ご冗談でしょう?名無しさん:2011/05/20(金) 22:48:37.57 ID:???
赤道近辺では夏も冬も存在しない。
779ご冗談でしょう?名無しさん:2011/05/20(金) 22:58:29.75 ID:???
なぜ宇宙マイクロ波背景放射=空間みたいに言われるんですか?
780ご冗談でしょう?名無しさん:2011/05/20(金) 22:58:49.86 ID:???
北極、南極は夏でも冬でも寒い
熱帯は夏でも冬でも暑い

南北の中間緯度を往復すれば暑くなったり寒くなったりする
781ご冗談でしょう?名無しさん:2011/05/20(金) 23:04:29.36 ID:???
>>770 >>754
風洞試験か何かの視点に立った説明でしょうか。

ttp://userdisk.webry.biglobe.ne.jp/004/970/07/N000/000/003/126970921132416408361.jpg
再突入カプセルは直径40cm、質量17kgの「蓋付き中華鍋」形で、アブレータ
と耐熱材が大部分を占め、表面にはアルミ蒸着カプトンの薄膜が貼られてい
る。地球帰還後は月軌道付近で本体から分離し(実際には 70,000 kmまたは
74,000 kmで分離した)、突入角12.0度、秒速12.2kmの超軌道速度で再突入
する。地球周回軌道から再突入するスペースシャトルの約30倍(淀み点総加
熱率)もの空力加熱によってカプセル周囲の気流は摂氏10,000度に達するが、
アブレータから揮発したガスが熱を遮り、ヒートシールド表面は3,000度、カ
プセル内は50度程度までに抑えられる。
782ご冗談でしょう?名無しさん:2011/05/20(金) 23:09:42.45 ID:???
>>770
主流の流速(突入時の対気速度)を増やせば温度境界層が薄くなり熱流束が大きくなる。
逆にシーリングを施せば流速が減じるのみならず、ガスと主流の境界面は衝撃波で覆われ流束は極端に小さくなる。
また、外気より断熱ガスの密度の方が遥かに高い。
故に主流から受け取った熱は平均自由行程の短いガスの層で十分に拡散・緩和される。
783ご冗談でしょう?名無しさん:2011/05/20(金) 23:32:40.70 ID:???
>>770
基本的に気体や液体の熱伝導率はかなり低いんだよ。
身の回りの物質で熱伝導率が高いのは自由電子が熱を媒介できる導体がほとんど。
水を入れた試験管の下の方を過熱すると対流で全体が暖まるが上を過熱すると下は冷たいままって実験小学校でやっただろ?
784ご冗談でしょう?名無しさん:2011/05/20(金) 23:44:44.46 ID:???
>>773
回転する座標系で考えてるわけだから
遠心力のポテンシャルもあるよ
785ご冗談でしょう?名無しさん:2011/05/21(土) 00:05:45.19 ID:???
>>770の疑問点は
× どう熱を遮断するか
○ はやぶさの運動エネルギーがどう熱に変わるか
なのか?
786ご冗談でしょう?名無しさん:2011/05/21(土) 00:20:32.35 ID:QdNwZCbE
>>774、わかりませんか?
787754/760/770:2011/05/21(土) 00:39:53.57 ID:???
すいません全部にアンカーつけないとわかりづらいですね

>>781-783
それ自体はわかってるんです。二重ガラスとか火渡りの荒行とかね。

>>785
>>755が空気の圧縮によると答えてくれたので納得。

自分がどうしても理解できないのは「空気の」運動エネルギーが熱に変わる
と展示パネルに書いてあったこと。
>>762がカプセルの静止系で見ると答えてくれて、たしかにそうなんだけど、
普通は地面を基準に空気を(ほぼ)静止、カプセルを運動と考えますよね?
もし、762の答えが正しいなら、わざわざ、自分的には回りくどくてわかりにくいとしか
思えない表現にした科学的理由があるのか知りたいです。
788ご冗談でしょう?名無しさん:2011/05/21(土) 00:50:50.87 ID:???
股メコスジに恋してる
789ご冗談でしょう?名無しさん:2011/05/21(土) 00:57:56.26 ID:???
>>787
把握した。ガリレイ変換で検索すれ。
それはわかる、なぜカプセルを基準に考えるのか、というならそっちの方が便利だから。
カプセルを静止させた系なら外気流をポテンシャル流として扱えばいいだけだが
動かすと強大な空間を想定するはめになる。余計な労力いくない。
これの考え方を元にした実験装置を風洞とよぶ。
790ご冗談でしょう?名無しさん:2011/05/21(土) 01:17:28.69 ID:???
>>787
たぶん、このあたりかと。

ttp://www.jspf.or.jp/Journal/PDF_JSPF/jspf2006_06/jspf2006_06-368.pdf
飛翔体が地球大気に再突入する際,(機体上に固定した
座標系で見れば,わかりやすいが)軌道速度で大気が機体
に衝突することと同等である.この際,大気の運動エネル
ギが熱エネルギに変換されて高温になり,機体を加熱す
る,いわゆる空力加熱現象が発生する.
791754/760/770/787:2011/05/21(土) 01:35:45.28 ID:4AbBwKKh
>>787
まだ噛み合ってない気がする。もどかしい。
だれか、実際に展示を見た者はおらぬのかっ?

ガリレイ変換て初めて聞く言葉だけどウィキをちょっと見る限り
理解してると思います。要はカプセルを静、外気を動としても
計算上は違いがないみたいなことですよね。風洞実験もわかります。

でも、(自分もそうかもしれませんが)探査機名がイトカワとか言ってる大学生や
ハヤブサが微生物を取ってきたと自慢げに説明するオヤジみたいな
にわかが群がる展示場で何の説明もなくそう書くのは不自然というか、
むしろ、なぜカプセルの運動エネルギーが熱に変わると書かなかったのか?
(計算方法ではなく、そういう言い方をした必然性?みたいな)
792ご冗談でしょう?名無しさん:2011/05/21(土) 01:42:03.21 ID:???
>>791
>ガリレイ変換て初めて聞く言葉だけど
おいおい……。
抛物線運動とか知らない人?

>(計算方法ではなく、そういう言い方をした必然性?みたいな)
流体力学では常識以前だから先入観があったんだろうね。
つか俺も引っ掛かる奴がいるとは思わなんだ。
793754/760/770/787:2011/05/21(土) 02:22:41.82 ID:4AbBwKKh
だんだん自分でも何言ってるかわからなくなってきた。
しかし>>790のおかげでわかった気がする。
この(実際である大気圏突入と同等である大気の衝突)の際に大気の運動エネルギーに〜
これならわかります。
要は「同等」=「計算上同じでも、そのものズバリではない」

実際と異なる前提?で出てきたのに、大気の運動という言葉だけを抜き出して
パネルに引用したのでしょう。
単にパネル製作者がしっかり理解していなかったためのミスと解釈して
自己解決となりそうです。

長らくお騒がせしました。(おかしいと思う人はまだ意見ください)
794ご冗談でしょう?名無しさん:2011/05/21(土) 02:47:10.58 ID:???
>>793
>単にパネル製作者がしっかり理解していなかったためのミス
地球を中心とした絶対静止系、俗にいう天動説的な考え方が排他的に正しければその通りだね。
相対性原理が否定されてない前提なら間違いでも何でもないが。
795ご冗談でしょう?名無しさん:2011/05/21(土) 03:11:23.07 ID:???
>>793
一方の見方が"実際"でもう一方は"仮想" というようなことではないよ。
2つの見方にそういう差別はない。

2つの物体の衝突を考えるときに
ある見方ではこう見えるし
 ● ←○

別の見方ではこう見える。
 ●→ ○

どちら側が運動エネルギーを持っていたかというのも
見方(座標系)によって変わる。
796ご冗談でしょう?名無しさん:2011/05/21(土) 05:40:07.79 ID:???
>>793
例えは悪いが「カーブでガードレールがぶつかって来た!」とか言うような連中
相手にしても仕方ないよw
797ご冗談でしょう?名無しさん:2011/05/21(土) 06:43:41.56 ID:???
ジェットコースターで自由落下すると、おなかがくすぐったくなりますが、
多分、重力が0になるとそうなるんだと思いますが、
宇宙で暮らすとなると一日中あの感覚を味わわないといけないんでしょうか?
798ご冗談でしょう?名無しさん:2011/05/21(土) 07:45:52.01 ID:???
>>793
理解してるからこその記述だろ。
ミスと言えるのは、それが素人を含む一般向けの説明としては良くなかったという点だろう。

だからといって>>795みたいに差別はない(キリッ)とか、不親切な説明でも間違いじゃないから良いというのもアホ。
>>796はギャグでいってるんだよなそれ
799ご冗談でしょう?名無しさん:2011/05/21(土) 07:57:31.11 ID:sfkS0BF+
クーロン力とか核力を人為的に強めたり弱めたり出来るの?
800ご冗談でしょう?名無しさん:2011/05/21(土) 07:59:09.67 ID:???
無理
801ご冗談でしょう?名無しさん:2011/05/21(土) 08:05:22.21 ID:???
>>792
>抛物線運動とか知らない人?
俺も知らないということだけ最初に断っておくが、
その気持ち悪い字は「おれ知ってんだぜ」ってひけらかす以上に何か価値はあるのか?
802ご冗談でしょう?名無しさん:2011/05/21(土) 08:22:52.90 ID:sfkS0BF+
>>800
その理由を教えてほしいです。または、将来的に出来るようになるか?というのも詳しく。
803ご冗談でしょう?名無しさん:2011/05/21(土) 10:34:11.53 ID:pbm/KqOi
距離そのままで電荷を増減することによる制御なら広く実用化されている
そうではなく何か根本的な物理定数を変えたいとかか?
804ご冗談でしょう?名無しさん:2011/05/21(土) 10:34:19.06 ID:1QD/Py3L
理由って、この宇宙の法則がそうなっているからとしか言いようがないでしょ。
数学の公理に理由付けを求めるようなものかな?
805ご冗談でしょう?名無しさん:2011/05/21(土) 12:04:49.97 ID:???
文字数規制かかってしまうので申し訳ありませんが質問を知恵袋で失礼させていただきます。
http://detail.chiebukuro.yahoo.co.jp/qa/question_detail/q1062732470?fr=chie_my_notice_newans
806805:2011/05/21(土) 12:06:52.29 ID:6dmidc6E
間違えて途中でsageで書き込みしてしまいました。ご助言頂けると大変ありがたいです。よろしくお願いいたします。
807ご冗談でしょう?名無しさん:2011/05/21(土) 12:07:10.89 ID:yrio1dLi
東北沖地震のエネルギー量ってどれくらいなんですか?
808ご冗談でしょう?名無しさん:2011/05/21(土) 12:11:31.99 ID:???
>>799
2つの電荷A・Bを物質の中に埋めて
物質に生じる電荷に目をつぶると 見かけ上 A・B間の力が小さくなったように見える
というのはあるけど そういうのではなく?
809ご冗談でしょう?名無しさん:2011/05/21(土) 12:13:56.36 ID:???
>>803
>>804
なるほど。クーロン力とか弱めたり出来れば人工的に新しい原子を作れたりする気がしたんですが
宇宙の法則をねじ曲げるようなもんだから無理って事ですね。ありがとうございました
810ご冗談でしょう?名無しさん:2011/05/21(土) 12:18:28.50 ID:???
>>808
そういうのでは無いです。それも面白そうですが
811ご冗談でしょう?名無しさん:2011/05/21(土) 12:30:52.55 ID:???
>>807
3/11はおおむね2EJくらいだな。

http://www.japanquakemap.com/dailyEnergy
812ご冗談でしょう?名無しさん:2011/05/21(土) 12:37:44.60 ID:yrio1dLi
>>811
返答ありがとうございました
813791:2011/05/21(土) 14:38:08.59 ID:4AbBwKKh
返信があったのでまた来ました。解決してなかったのか。
とりあえず不適当な表現と製作者に対する暴言に謝

>>792.794.795
高大と生物メインだったんで数式とか用語は詳しくないけど、
少なくとも今回の疑問に対するレスの理屈は大雑把にはわかっています。

相対的なのも承知の上で、むしろ見方(座標系)によって変わるなら
その基準をはっきりさせなければいけないのに、それが一切書かれていない
のでミスと書いてしまった。
(地球が絶対静止系とは言わないが、大気圏突入という言葉自体や
ハヤブサの軌道修正等も地球側を基準にしていると思われるし、
特に説明がなければ自分も含め一般人はそうとらえる、というか他に思いつかないだろう)
←結局>>762のレス以降ずっと気になってたのこういう事。

それで、>>792が言うとおり「流体力学では(カプセル基準が)常識以前」なので、
ギョーカイ的にはあえて書かないのが一般的ってことでFA?
814ご冗談でしょう?名無しさん:2011/05/21(土) 15:21:24.64 ID:???
空気が入ってない箱が10kgとして
その箱の上から1kgの鉄球を落としたら
落ちてる最中の、はこの下に置いてある体重計は10kgを指しますっけ?
815ご冗談でしょう?名無しさん:2011/05/21(土) 15:24:05.55 ID:???
>>813
規制が解除になったのでコメントする。

やっぱり地球基準で考えて飛翔体の運動エネルギーが熱に変わるという言い方のほうが適切だと思う。
それに、静止した気体の断熱圧縮なら一般人でも納得しやすい。

一般向けの説明で飛翔体基準を使うメリットはないでしょう。
816ご冗談でしょう?名無しさん:2011/05/21(土) 15:27:27.75 ID:???
相対論で質問なんだけど

星の光はン十億年もかけて地球に届いてるって説明を聞くけど
同時性を考慮すれば「今輝いてる光が届いてる」って解釈でも間違いではないよね?
817ご冗談でしょう?名無しさん:2011/05/21(土) 15:31:05.98 ID:???
>>813
そんな些事にそこまで粘着する奴が存在するなんてそれこそ想定外だろ。
つうかもう物理関係なくね?

>>814
箱の外の空気の粘性が無視できない→変わる
箱と鉄球の間の引力が無視できない→変わる
上記二つを無視できる→変わらない
818ご冗談でしょう?名無しさん:2011/05/21(土) 15:33:13.29 ID:???
>>816
全く持ってその通り。
819ご冗談でしょう?名無しさん:2011/05/21(土) 15:35:07.34 ID:???
>>818
ありがと。
でもこれを将来子どもに説明するとなるとどうかみ砕けばいいのかなあ…と思う

まあ結婚相手がいないけどw
820ご冗談でしょう?名無しさん:2011/05/21(土) 15:37:49.97 ID:???
>>816
そうはならんでしょ
いったいどんな座標系を考えてる?
821ご冗談でしょう?名無しさん:2011/05/21(土) 15:56:24.41 ID:???
>>820
同時性の話だよ。座標は関係ない。関係あるのかも知れないけど。
観測できるのはまさに「ン十億年前の今」って話。文章としては少し変だけど
822ご冗談でしょう?名無しさん:2011/05/21(土) 16:03:06.64 ID:???
>>716
>>727
>>743
>>749
>>753
それで結局、地球と月のラグランジュ点に大きな物体が集積していない理由は何なんでしょうか。
823ご冗談でしょう?名無しさん:2011/05/21(土) 16:11:22.40 ID:???
>>822
その話題は以前天文板の質問で出たことあるんで、そちらの回答でも
探してみれば?
824ご冗談でしょう?名無しさん:2011/05/21(土) 16:15:21.80 ID:???
>>821
何が言いたいのかぜんぜんわかんね
825813:2011/05/21(土) 17:33:07.82 ID:jSimxK7k
>>815
ありがとう

>>817
自分だってこんな長引くとは思わなかったが、
説明下手なのかこちらの聞きたいことがうまく
伝わらないんだから粘着言われようがしかたない。

ただ、確かにスレの趣旨から外れてきたようだし
ウザがられてまで聞く様な話ではないのでいいかげんにします。お騒がせしました。
826ご冗談でしょう?名無しさん:2011/05/21(土) 18:04:38.45 ID:???
・エネルギーが分散している方が物質的には安定する
・トレードオフ(二律背反、等価交換の原則)

この2つは万物の真理だと思うが他にも何かある?
827ご冗談でしょう?名無しさん:2011/05/21(土) 18:48:40.96 ID:Jkva8AYg
「福島原発 臨界」でググると「再臨界」の記事ばかりヒットしますが、
これは文字通り、

福島原発では臨界が起こった。そして再び臨界が起こる(かもしれない)

という意味に捉えて良いのでしょうか?
臨界が起きた事が前提の「再臨界」なのか、「臨界」とは別の現象の「再臨界」なのか分かりません
828ご冗談でしょう?名無しさん:2011/05/21(土) 19:00:17.43 ID:???
「福島原発では臨界が起こった。そして再び臨界が起こる(かもしれない)」

という内容のサイトが沢山有るなら、そうかもね
829ご冗談でしょう?名無しさん:2011/05/21(土) 19:04:19.15 ID:???
>>828
>「福島原発では臨界が起こった。そして再び臨界が起こる(かもしれない)」

こう書いてあれば質問しません
最初の前提の「臨界」を飛ばしていきなり「再臨界」の話になってるから混乱してるんですが
830ご冗談でしょう?名無しさん:2011/05/21(土) 19:07:52.59 ID:???
だれも切れてないのに、いきなり逆切れって言うみたいなものだね
831ご冗談でしょう?名無しさん:2011/05/21(土) 19:09:28.21 ID:Jkva8AYg
>>830
文系臭え例え話要らねーから、さっさと質問に答えろや、カス
832ご冗談でしょう?名無しさん:2011/05/21(土) 19:12:47.12 ID:???
臨界以前にメルトダウン起こして格納容器に穴空けて海にでも流れていった後だろ
833ご冗談でしょう?名無しさん:2011/05/21(土) 19:29:48.64 ID:???
>>831
記事がたくさんなるならそれを読めばいいじゃねえか
記事の大部分は臨界の意味がちゃんと分かっているかも怪しいものだが
834ご冗談でしょう?名無しさん:2011/05/21(土) 19:34:43.47 ID:IGqszIP4
「超ヒモ理論」は原子がヒモでできてるという理論っぽいけど
最近いろいろ解明されてきてる素粒子はどうなった?
あれは全部無視か?
原子をもっと細かく分けると、ヒモなのか素粒子なのか、どっち?
835ご冗談でしょう?名無しさん:2011/05/21(土) 19:35:51.79 ID:IGqszIP4
>>700
いや、その本は難易度が高すぎます。
わたしはいくら高校で物理が得意だったとはいえ、もっと図やイラストがメインの本をお願いします。
836ご冗談でしょう?名無しさん:2011/05/21(土) 20:33:58.43 ID:???
単位に1物体の質量の二乗が含まれて定義されてるものはありますか?
なかったとしたらそのような概念を考えるのは意味がないのでしょうか
837ご冗談でしょう?名無しさん:2011/05/21(土) 20:37:23.42 ID:???
実験レポートの書き方について質問です。
結果と考察の分け方が分かりません。

直接得られた測定値はともかく、そこから計算やグラフの読みなどにより二次的に得られた値は
結果と考察のどちらに書くべきなんでしょうか。
実験値と理論値の比較もどこに書くべきか分かりません。
あと、理論値は結果の前と後(考察のとこ)のどちらに書くべきかも分かりません。
よろしくお願いします。
838ご冗談でしょう?名無しさん:2011/05/21(土) 20:38:53.87 ID:???
>>827
福島第一は原子力発電所なので、通常
運転中に普通に「臨界」してます。
それが地震で緊急停止した後、なんじゃらかんじゃらあって、「制御されてない状態で」もう一度「臨界」したら、それが「再臨界」です。
839ご冗談でしょう?名無しさん:2011/05/21(土) 20:41:28.60 ID:???
>>837
どっちでもいいだろ。
どっちかにとりあえず書いて、最後まで書いて読み返して構成がおかしいなと感じられたら変えればいい
840ご冗談でしょう?名無しさん:2011/05/21(土) 20:48:19.28 ID:???
工房なんだが、問題でどの公式使えばいいかわからん。鉛直系はすぐにわかるから簡単なんだが

外力0→力学エネルギー保存
摩擦とか込み→運動量保存

って感じか
841ご冗談でしょう?名無しさん:2011/05/21(土) 20:55:12.93 ID:???
>>839
ありがとうございます
実験結果には直接得られた測定値を書いておくだけで構わないということですね。
842841:2011/05/21(土) 20:56:30.61 ID:???
「だけでも」の間違いです。
843ご冗談でしょう?名無しさん:2011/05/21(土) 21:33:21.73 ID:IGqszIP4
超ヒモ理論のヒモってなんなの?
844ご冗談でしょう?名無しさん:2011/05/21(土) 21:41:58.35 ID:???
女に養って貰う男
845ご冗談でしょう?名無しさん:2011/05/21(土) 21:53:53.98 ID:???
>>834
原子をもっと細かく分けると、膜。
846ご冗談でしょう?名無しさん:2011/05/22(日) 00:46:57.75 ID:f6C1nXB4
>>837
もういないかな

実験値とは、実験で得られた値のみをいい、一切操作してはならない
議論の余地がない部分を、そうとわかりやすくまとめるところ

計算やグラフの読みには、「なぜ」その計算をしたのか、「なぜ」そう読んだのかに、あなたの存在が含まれている
つまり、あなたの考察であり他者に反駁を試みてもらうことで正しさを測るところ

結果と考察を、なぜ分けるのか目的を明確にすればすっきりする

理論値は、既に証明された定理は議論の余地がない部分で、あなたが定理を正しく使ったかどうかは検証の必要がある
未証明の仮説は引用か否かを明らかにすること
847ご冗談でしょう?名無しさん:2011/05/22(日) 01:04:58.27 ID:???
>>846
ありがとうございます、分かりやすいです。

あと、理論値や予測をどこに書くかと聞いたのは、結果よりも前のところ(実験原理のところにくっつけるか、独立して節を立てるか)か、
あるいは考察の中、実験値と比較する直前の部分でいいのか、という意味で聞きました。
実験の際にはあらかじめ予測を立てて実験しろとよく言われるので、実際の手順に沿って書くなら実際結果よりも前に書くべきかなとも思いまして。
848ご冗談でしょう?名無しさん:2011/05/22(日) 01:07:21.11 ID:???
力の釣り合いについて教えて欲しいのですが
授業で、自由落下している質点mは、下向きの力重力(mg)と、その場に留まろうとする上向きの力
慣性力(-mx'')と力が釣り合っていると教えられたのですが、物体が動いているのに力は釣り合っているのですか?
高校までの知識で行くとmx''<mgになるから下向きに動いていくと思うのですが・・・
また、慣性力というものもいまいちピンと来ません・・・どなたか分かりやすい説明を下さい。
849ご冗談でしょう?名無しさん:2011/05/22(日) 01:10:51.91 ID:???
すいませんもう一つ

>>846
その理屈でいくと、測定値の誤差の評価にも議論の余地があるので、
実験結果の節では数値に誤差(±いくつ、みたいなの)を付けてはいけないんでしょうか?
またそうすると、グラフにもエラーバーを付けない?(あるいはグラフを考察の方に書く?)

あと、測定値に付記した誤差の要因って全部記しておくべきなんでしょうか。
機器の分解能による誤差がいくら、統計的なゆらぎがいくら、合わせていくら、みたいな感じで。
850ご冗談でしょう?名無しさん:2011/05/22(日) 01:12:29.70 ID:???
慣性は計算しやすく考えやすくするために使うだけ。数学の公式といっしょ

例えば自由落下する物体に君がいるとしたら、自分の足場は動かずに世界が動くことになる
そういう前提で考える事
851ご冗談でしょう?名無しさん:2011/05/22(日) 01:15:56.51 ID:???
>>848
> 自由落下している質点mは、下向きの力重力(mg)と、その場に留まろうとする上向きの力
> 慣性力(-mx'')と力が釣り合っている
質点と一緒に動く観測者(この観測者から見たら質点は止まってる)から見ればそうだけど、
慣性系から見たら違う。

> 高校までの知識で行くとmx''<mgになるから下向きに動いていく
これは間違い


まず、「ニュートンの運動方程式が成り立つのは慣性系から見たときだけである」という事実がある。
しかし非慣性系から見たときも、運動方程式にある補正項を足せば正しい式になる。
この補正項が慣性力。
852ご冗談でしょう?名無しさん:2011/05/22(日) 01:20:58.67 ID:???
>>850-851
どうもありがとうございます
慣性形と非慣性形について調べてみます
853ご冗談でしょう?名無しさん:2011/05/22(日) 01:47:11.37 ID:???
>>848   慣性力というものもいまいちピンと来ません・・・  の部分のみにレス
毎日感じているはず ⇒ 交通事故。 車の重さ x 速度 = 慣性。 止まってる車にぶつかっても ”イテッ” で済むが
走ってるのに当ると死ぬ。  そのエネルギーの差。
"男の子"ならみんな大好き 武器 兵器 軍 自衛隊 / 拳銃弾をウィキ→重さ 数グラム なのに 音速と言うスピードに成ると死ぬ。

で。弾道がお辞儀する。 (野球ボールでも良いが) / 自由落下 / 宇宙空間ほどに大きく解釈、スケールに成ると
落ちて行く先に地面が無い。 イツまでも落ち続ける状態に成った時を”周回軌道に乗った”と言う。
854ご冗談でしょう?名無しさん:2011/05/22(日) 01:55:34.67 ID:???
「慣性力」なんて気取って書くから分かりにくい
昔みたいに「見かけの力」って書きゃ良いじゃん
でも、その座標系から見れば「見かけ」じゃ無くて、本物の「力」なんだよな
却って分かりにくいか?
855ご冗談でしょう?名無しさん:2011/05/22(日) 01:59:01.67 ID:???
ごく低温を除いてマグノンの比熱って電子比熱より大きい?
856ご冗談でしょう?名無しさん:2011/05/22(日) 02:30:08.95 ID:f6C1nXB4
>>847
実験1つとっても、「なぜ」その実験をしたのかの動機と実験そのものに分かれ、
実験を「どのような条件で」結果がどうであったのか、他者が追試することで議論の余地がなく、
誤差を客観的に定義して確認できることと、実験条件そのものの妥当性について他者による検証を要す

あらゆる考察が、動機・事実・考察・主張の起承転結に分かれ、
それらの詳細もまた起承転結からなるフラクタルであるべき
857ご冗談でしょう?名無しさん:2011/05/22(日) 10:57:23.89 ID:???
静止したブランコに人が立っているとして、外力を加えずに
これを動かすにはどうすればいいのでしょうか?
858ご冗談でしょう?名無しさん:2011/05/22(日) 11:51:59.24 ID:???
こぐ
859ご冗談でしょう?名無しさん:2011/05/22(日) 13:01:38.07 ID:KZQalDnJ
>>857

>>858も言っているように漕げば良いのですが、その力学系が分からないのでしょうね。

ブランコの動作は、踏み込む足の力が座板に作用して振り子運動に変換されている様に思うかも知れませんが、実は、その殆どは足の裏と板の間に生じた圧縮空気の作用によるものなのです。

詳しくは、岩並抽象物理学辞典前科6版をご覧ください。
860ご冗談でしょう?名無しさん:2011/05/22(日) 13:06:00.46 ID:???
作用点の定義ってなに

垂直効力とか面での場合作用点以外でも作用してるじゃん
861ご冗談でしょう?名無しさん:2011/05/22(日) 13:07:30.30 ID:???
外力に加えずにと言ってもブランコの振れ幅大きくするには
重力利用するしかないと思うんだが
862ご冗談でしょう?名無しさん:2011/05/22(日) 14:07:14.51 ID:???
>>857
こぎ始めはひもを握った部分から下を二重振り子のように前後に揺らして、
ゆれ始めたら体重の上下移動(足の屈伸)によって振幅を増しているようです。
もしブランコがひも(ロープ、クサリ等)ではなく、一本の棒で吊り下げられ
ていると、こぎ始めることが困難か不可能と言えます。

ttp://www008.upp.so-net.ne.jp/takemoto/buranko.htm
863ご冗談でしょう?名無しさん:2011/05/22(日) 15:02:11.30 ID:???
ブランコの動きは振動、すなわち波動である。
漕ぐときは一旦しゃがむような姿勢から身体を伸ばす。
このとき大地の気を吸い取って天に向かって放散している。
つまり宇宙の波動エネルギーがブランコを動かしているのだ。
864ご冗談でしょう?名無しさん:2011/05/22(日) 15:51:45.44 ID:???
量子力学的な運動量のゆらぎを増幅してマクロな運動に変えているのがブランコです。
865ご冗談でしょう?名無しさん:2011/05/22(日) 15:57:53.80 ID:???
共鳴周波数でこぐと大変なことになるからブランコは即刻撤去するべき
866ご冗談でしょう?名無しさん:2011/05/22(日) 16:16:46.75 ID:???
>>865
ブランコがひもで吊り下げられているかぎり、180度以上回ることは不可能でしょう。
たぶん。
867ご冗談でしょう?名無しさん:2011/05/22(日) 16:26:53.07 ID:A2pefAwo
5周ぐらいならまわれると思う。そのうちチェーンの長さが足りなくなるけど。
868ご冗談でしょう?名無しさん:2011/05/22(日) 16:52:41.13 ID:Zny2AAib
知恵袋でも尋ねましたがわかりませんでした。
加速によって徐々に大きくなる円運動についての質問です。
http://ci.nii.ac.jp/naid/110000194008
このページのプレビューから見れる論文についてなのですが最初の188ページのFig2.1のθの求め方がわかりません。
次の189ページにはφがθを用いて表されているので必要なのですが、速度方向がつねに変わると思うので一周でどれくらい遠地点距離が変化するかわかりません。
最終的には190ページにあるFig2.4のグラフを描きたいのです…お分かりになられる方がいらっしゃれば考え方か、こういった運動の名称などヒントを教えていただけるとありがたいです。
869ご冗談でしょう?名無しさん:2011/05/22(日) 17:01:50.98 ID:???
>>868
>Fig2.1のθの求め方がわかりません。
求め方も何も、単にセイルの速度と太陽光のなす角度としてθが定義されているだけのような
870ご冗談でしょう?名無しさん:2011/05/22(日) 17:10:30.33 ID:dcEUXgzO
光って1秒間に地球を7周半するっていう例えでその速さを強調してるけど
よく考えると地球を7周半ってそんなもんなのかって遅く感じてしまう

太陽からの光が地球に届くのも約8分もかかるもんな。
871ご冗談でしょう?名無しさん:2011/05/22(日) 17:25:38.22 ID:???
1m飛ぶのに3.3ナノ秒もかかるしな
872ご冗談でしょう?名無しさん:2011/05/22(日) 17:29:02.86 ID:???
不確定性原理や光速度による因果律みたいな限界を超える方法ってないのかね

スクイーズド光は何か違うし。スクイーズド光でサーチ出来るなら新しい素粒子見つけることも可能な気がするけど…
873ご冗談でしょう?名無しさん:2011/05/22(日) 17:59:47.37 ID:???
>>872
スクイーズド光は位相と振幅のように二律背反する揺らぎのどちらを犠牲にするか
という不確定性原理に従っている。
カシミール効果のように揺らぎそのものを制御できたとしても、時空が縮小するだけ
かも知れない。
874ご冗談でしょう?名無しさん:2011/05/22(日) 18:01:31.79 ID:???
T字帯ってなんでしょうか教えてください
875ご冗談でしょう?名無しさん:2011/05/22(日) 18:09:31.37 ID:???
>>873
そうそう。
結局スクイーズされた光を原子とかに照射しても観測できるのは不確定性関係から増幅された側の光なんだよね

上手いテクニックで回避できれば新しい観測方法を生み出せるかもだが…
876ご冗談でしょう?名無しさん:2011/05/22(日) 18:11:34.11 ID:???
>>869
ご返信ありがとうございます。初めから、190ページにあるFig2.4のグラフを描く方法…と尋ねるべきだったかもしれません。要するに増速度最大制御での時間と遠地点の関係を周ごとに求めたいのです。
>求め方も何も、単にセイルの速度と太陽光のなす角度としてθが定義されているだけのような
おっしゃる通りです。ただ、速度の方向も変化するので表し方がわからないのです。
φを表すθが式で表せないと、加速度の方向がわからず、速度の方向も、楕円の形がわからないので求めることもできないかと思います。
Fig2.4のグラフを見るところでは加速度は周期的に考えると一定の様に思えるのですが、良い方法はありませんでしょうか。何度もすみません。
877ご冗談でしょう?名無しさん:2011/05/22(日) 18:19:34.37 ID:KZQalDnJ
そもそも、なぜ総ての物質は、光速を超えて運動出来ないのでしょうか?ただ単に赤方偏移のため観測出来ないだけのはなしですか?

仮に、Aから見てaが光速で西へ、Bからみてbが光速で東へ移動へ運動した場合、aからみたbの速度は度の様に成りますか?

a←A B→b
878ご冗談でしょう?名無しさん:2011/05/22(日) 18:39:20.85 ID:???
>>877
相対性理論によれば全ての観測者は光速に達せないので、
相対性理論が正しいならばあなたの示した問題は実験で確かめられません。
実験で確かめられないことは物理の外のことなので、あなたの質問に物理的観点から答えることはできません。
以上が、相対性理論が正しいとした場合の回答になります。
879ご冗談でしょう?名無しさん:2011/05/22(日) 18:42:26.99 ID:???
>>877
> そもそも、なぜ総ての物質は、光速を超えて運動出来ないのでしょうか?ただ単に赤方偏移のため観測出来ないだけのはなしですか?

こんな不見識じゃ>>859も信用ならんね。
880ご冗談でしょう?名無しさん:2011/05/22(日) 19:19:00.59 ID:KZQalDnJ
>>878
光速→光速の90%とした場合はどうですか?

仮に、Aから見てaが光速の90%で西へ、Bからみてbが光速の90%で東へ移
動へ運動した場合、aからみたbの速度は度の様に成りますか?

a←A B→b

>>879
嫌らしい奴だな!
881ご冗談でしょう?名無しさん:2011/05/22(日) 19:24:41.39 ID:???
>>880
AとBの関係が抜けてる
882ご冗談でしょう?名無しさん:2011/05/22(日) 19:25:28.55 ID:???
なぜAとBが出てくるのだろう
Aだけでいいんじゃないだろうか
883ご冗談でしょう?名無しさん:2011/05/22(日) 19:26:41.15 ID:???
後だしで屁理屈を並べるための下準備だろうか
884ご冗談でしょう?名無しさん:2011/05/22(日) 19:27:09.14 ID:???
宇宙最速なのが光速ってほんとか?
光速に近づくほど時間がゆっくり進んで時間が停止するんだろ?
光速超えたら虚数空間へいくの?過去へいくの?
光速を基準するのがおかしくないか。
たまたま、どの観測者から見ても同一速度に見えるのかもしれないが。
光速より速くてもそうかも知れない。
885ご冗談でしょう?名無しさん:2011/05/22(日) 19:29:41.97 ID:???
光速*2を基準に相対論を展開したら矛盾するの?
いまの光速の値が唯一なのか。
886ご冗談でしょう?名無しさん:2011/05/22(日) 19:41:58.65 ID:???
>>876
数値的に解くしかないのでは。解析的に出せるものならその式が載っていると思う。
887ご冗談でしょう?名無しさん:2011/05/22(日) 19:43:29.72 ID:???
>>859はネタだと思うが
「前科6版」とか見る限り
888ご冗談でしょう?名無しさん:2011/05/22(日) 19:48:14.78 ID:???
>>884
光速を基準にしてみたらいろいろな観測事実がうまく説明できた。
おかしいとか文句言ってみても始まらない。

未来の発見を予言することはできないから、将来、光速を基準にしない理論で
よりうまくいくものができるかどうかはわからないとしかいいようがない。
889ご冗談でしょう?名無しさん:2011/05/22(日) 19:50:39.24 ID:???
>>887
ブランコのパラメータ励振は重心移動によるものですから圧縮空気とか関係ないですね
890ご冗談でしょう?名無しさん:2011/05/22(日) 20:03:43.89 ID:???
>>880
AとBは互いに静止しているとするなら、
aから見てbは光速の99.4%で動いて見える。
891ご冗談でしょう?名無しさん:2011/05/22(日) 20:04:02.77 ID:???
物理学者は、光速が特別なことを説明すべき。
たまたま、マクスウェル方程式で、光速不変が出てきて、それを利用しただけの気してる。
光速度でしか、不変が知られてないだけだろ。
それ以上の速度では、どうなるのかよく分からない。
892868:2011/05/22(日) 20:08:41.56 ID:???
>>886
数値的に、とは与えられたデータをグラフにしているということですか?
893ご冗談でしょう?名無しさん:2011/05/22(日) 20:12:14.17 ID:???
>>891
物理学者が説明すべきじゃなくてあんたがもう少し勉強すべき。
教えて君のものの言い方もずいぶん変わったものだなぁ・・・・
894ご冗談でしょう?名無しさん:2011/05/22(日) 20:17:44.53 ID:e7GEcZaL
上記は、
相対論せ考えてみて
すべて答えがでるから。
あと、教科書読め、参考書読め。過去スレ読め。落ちたスレにも、答えが出ているから。
では。
895ご冗談でしょう?名無しさん:2011/05/22(日) 20:20:00.96 ID:???
誤解を恐れずに言えば、物理学者の仕事は既存の観測事実を体系的にまとめて未来を予測するのに役立てることであり、
物理学者は何の理由も説明したりはしないよ。
それは哲学者の仕事。
896ご冗談でしょう?名無しさん:2011/05/22(日) 20:59:20.67 ID:???
>>891
ペンローズのツイスター理論を勉強しる。

物理学者の仕事が>>895で説明された通りであることは事実だが、
ペンローズという物理学者はちょっと変わってて、物理学は真理の
探求であるべしと考えている。つまり、理由を説明すべきであると。

で、自分でも光速が特別である理由も考えて、ツイスター理論で
説明を試みてる。
897ご冗談でしょう?名無しさん:2011/05/22(日) 21:00:49.13 ID:???
>>877
講談社ブルーバックス の 石原藤夫 (玉大理工学博士 の本を。  (早川 SFマガジンの連載コラムを纏めたもの が判り易い)
http://ja.wikipedia.org/wiki/%E7%9F%B3%E5%8E%9F%E8%97%A4%E5%A4%AB

ありゃ〜 絶版!?か? 古本屋&Net で探すしか無いか。 (要約してレス と言われても名。 丸ごと自炊に為るし)
898ご冗談でしょう?名無しさん:2011/05/22(日) 21:28:55.96 ID:???
5000年後ぐらいの物理学ってどうなってんだろ

タイムマシーンは実現できる!とかまだやってるかなぁ
899ご冗談でしょう?名無しさん:2011/05/22(日) 21:36:58.11 ID:???
その頃、人類がまだ文明を保ってるかどうかの方が心配
900ご冗談でしょう?名無しさん:2011/05/22(日) 21:57:39.17 ID:???
この宇宙はゆがんでるそうですが、
それでもピタゴラスの定理って成り立つんですか?
時間の遅れの部分でピタゴラス定理使ってますけど。
901ご冗談でしょう?名無しさん:2011/05/22(日) 22:23:20.76 ID:dcEUXgzO
太陽がなんらかの原因でヒョコっと消えると
地球がそれまで感じてた太陽からの引力は瞬間的になくなりますか?
それとも光が太陽から地球に到達するのと同じ約8分かかりますか?
902ご冗談でしょう?名無しさん:2011/05/22(日) 22:26:09.70 ID:???
あらゆる影響は8分以降に来ます
903ご冗談でしょう?名無しさん:2011/05/22(日) 22:27:29.28 ID:TWP14Agx
超対称性粒子が考え出された理由はなんですか
904ご冗談でしょう?名無しさん:2011/05/22(日) 22:32:32.29 ID:e7GEcZaL
虚数解を考えるとわかりやすい。
905ご冗談でしょう?名無しさん:2011/05/22(日) 22:35:26.89 ID:???
>>904
虚数 x*x = -1

x * x * x = -1
になるような数による解だとどうなるのでしょうか
906ご冗談でしょう?名無しさん:2011/05/22(日) 22:40:01.91 ID:???
(-1+i√3)^3 / 8 = 1
907 忍法帖【Lv=22,xxxPT】 :2011/05/22(日) 23:51:44.08 ID:???
>>891
>たまたま、マクスウェル方程式で、光速不変が出てきて、それを利用しただけの気してる。
光速度不変ってマイケルソンモーリーから来てるんじゃないの?
908ご冗談でしょう?名無しさん:2011/05/22(日) 23:54:59.34 ID:???
たしかにマクスウェル方程式が一つの問題提起のきっかけではあったけど
アインシュタインは古典電磁気学でなくあくまでマイケルソンモーリーの実験に光速度不変の原理の根拠を求めたね
909ご冗談でしょう?名無しさん:2011/05/23(月) 00:09:15.72 ID:B+MRvUlb
密度が一様でない半径a厚さdの円盤がある。密度は半径方向に中心から図ってq(r)=crで与えられてる。円盤の質量求めよ。
これ積分とかつかう?
910 忍法帖【Lv=22,xxxPT】 :2011/05/23(月) 00:11:34.46 ID:???
>>908
そうなんだ、マイケルソンモーリーの実験って宇宙の絶対静止系を突き止めるっていうモチベーションだったと記憶してるから
それまでは光速度は可変って考えられてるものかと思ってた。
911ご冗談でしょう?名無しさん:2011/05/23(月) 00:13:04.77 ID:???
>>910
いんない。r=a/2入れればおk

でももっと複雑な式だったら使う
912 忍法帖【Lv=22,xxxPT】 :2011/05/23(月) 00:14:42.16 ID:???
>>909
密度がca/2の一様な円盤と質量は等価じゃん。
913911:2011/05/23(月) 00:16:17.11 ID:???
あ、いるいるごめん
914ご冗談でしょう?名無しさん:2011/05/23(月) 00:16:48.09 ID:???
誤差のありそうな実験結果に基づかないだろ。
マクスウェル方程式の誤差の無い、光速度不変の式を
電磁気以外のケースに一般化して使ったんだろ。
915911:2011/05/23(月) 00:16:51.64 ID:???
円周×密度を積分する。
916ご冗談でしょう?名無しさん:2011/05/23(月) 00:18:39.78 ID:???
>>914
一つの理論の上に理論を作るより
実験事実を認めてそこから理論組み立てたほうが安全
917 忍法帖【Lv=22,xxxPT】 :2011/05/23(月) 00:19:52.01 ID:???
>>913
いらねぇだろ
918ご冗談でしょう?名無しさん:2011/05/23(月) 00:21:34.41 ID:???
>>912
幅aの長い棒ならそうだけど、円だと違うくない?
919 忍法帖【Lv=22,xxxPT】 :2011/05/23(月) 00:22:03.13 ID:???
>>917
いや、いるのか。
何度もレス申し訳ない。
920ご冗談でしょう?名無しさん:2011/05/23(月) 00:22:35.96 ID:???
いまの認識からしたら、特殊相対性理論は、アインシュタイン以前に出来てただろ。解釈の違いだけで。

ヘンドリック・ローレンツ - Wikipedia
1905年、アインシュタインはそれらの概念や数学的手法やローレンツの考察を利用し、
"Elektrodynamik"(電気力学)と題した論文を書き、これが後に特殊相対性理論と呼ばれるようになった。
アインシュタインの成果はローレンツの成果に基づいているため、もともとは「ローレンツ-アインシュタイン理論」と呼ばれていた。


アルベルト・アインシュタイン - Wikipedia
以前から論理的に展開されていた相対性原理(アンリ・ポアンカレ、ジョゼフ・ラーモア、ヘンドリック・ローレンツなど)をもとに、
ニュートン力学とマクスウェルの方程式を基礎とする物理学の体系を根本から再構成した。


特殊相対論の基本的考え方はポアンカレがアインシュタイン以前にすべて出している、
ということをポアンカレやアインシュタインその他の人々が書いた論文・文章を豊富に引用しながら実証している本。
登場する主要人物は、ローレンツ、ポアンカレ、アインシュタインの3人。
アインシュタインの業績と考えられている相対論について、ポアンカレが特殊相対論の父であることを納得させられる本である。

アインシュタイン、特殊相対論を横取りする  Jean H.Ladik  深川 洋一 訳 発行元:丸善出版
http://pub.maruzen.co.jp/book_magazine/book_data/search/4621076701.html
921 忍法帖【Lv=22,xxxPT】 :2011/05/23(月) 00:23:33.45 ID:???
>>918
うん、円盤だと違うかった。
922ご冗談でしょう?名無しさん:2011/05/23(月) 00:25:19.84 ID:???
>>920
まあその解釈の違いこそが特殊相対論をアインシュタインの功績たらしめてるんだけどね
923ご冗談でしょう?名無しさん:2011/05/23(月) 00:26:59.27 ID:???
>>514
MM実験はちゃんと誤差評価もして、
少なくとも光速が地球の公転速度分変化することは得られた誤差からはあり得ないて示してるはずだが
924 忍法帖【Lv=22,xxxPT】 :2011/05/23(月) 00:36:01.33 ID:???
マクスウェル「光速度って不変じゃね?」
マイ モリ「そんなバカな、実験するぞ」

実験後
マイ モリ「不変でした」
アインシュタイン「これが相対論です」

こういう時代背景なの?
925ご冗談でしょう?名無しさん:2011/05/23(月) 00:47:15.32 ID:YPJaEfav
円周×密度の積分ですか?
926ご冗談でしょう?名無しさん:2011/05/23(月) 00:47:49.34 ID:???
当時は、マクスウェル方程式はエーテルに対する静止系でしか成り立たないと考えられていて、
マイケルソンとモーレーは、地球のエーテルに対する速度を求めるために実験を行った

って先生が言ってた
927ご冗談でしょう?名無しさん:2011/05/23(月) 00:53:05.56 ID:???
アインシュタインはマイケルソン・モーレーの実験結果を知らなかったと聞いたことがあるけど、それは眉唾なの?
928ご冗談でしょう?名無しさん:2011/05/23(月) 00:57:07.82 ID:???
知っていても、知らなくても、理論物理にその実験結果を土台に出来ないだろ。
929ご冗談でしょう?名無しさん:2011/05/23(月) 01:04:46.24 ID:???
お前は何を言っているんだ。
アインシュタインがどうだったかは知らないが、理論物理が基本的には実験に基づくのは当たり前のことだろう。
930ご冗談でしょう?名無しさん:2011/05/23(月) 01:07:55.45 ID:???
  1. 相対性原理(慣性系の同等性)
  2. マクスウェル方程式
  3. ニュートン力学
の3つを仮定すると矛盾する。

矛盾を解消する方法は何通りかあるが、
1. 2. は正しいと仮定して 3. を修正するのを選ぶと
特殊相対論が出てくる。

MM実験によって 1.&2. が検証されるので
3. を修正すべしということの論拠になる。
931911:2011/05/23(月) 01:09:09.07 ID:???
>>909
やっぱりいらないかも。まあどちらにせよ>>911は間違ってた。

この円盤は、中心からの距離に比例して密度が増えるので、
密度1で一様、高さdca、半径aの円柱から、高さdca、半径aの円錐をくりぬいた場合と同じ重さになる。

だから重さは2/3*円柱の質量=2/3*(a^2*π*dca)
合ってるならこれが一番スマート

もう寝よう。スレ汚しスミマセンでした
932ご冗談でしょう?名無しさん:2011/05/23(月) 01:09:36.81 ID:???
>>929
結果に矛盾しないようにするのと、土台にするのは違う。
933ご冗談でしょう?名無しさん:2011/05/23(月) 01:11:35.28 ID:???
数学とごっちゃにするなよ
934ご冗談でしょう?名無しさん:2011/05/23(月) 01:24:41.48 ID:???
光って空間のひび割れの伝搬みたいなものですか

重力波は空間の伸び縮みの伝搬だとすると
935ご冗談でしょう?名無しさん:2011/05/23(月) 01:31:10.23 ID:???
>>927
知ってはいたけど、意図していなかったといったところだろう。

今日では特殊相対性理論がマイケルソン=モーリーの実験に対する「解」
であると考えられているが、当時はそのような共通理解はなかった。
アインシュタイン自身でさえ1920年頃に、「空間は物理的な実在性を備え
ている」ことから「空間が持つ特質そのものをエーテルと呼ぶことができ
る」と述べた。
ttp://ja.wikipedia.org/wiki/%E3%83%9E%E3%82%A4%E3%82%B1%E3%83%AB%E3%82%BD%E3%83%B3%E3%83%BB%E3%83%A2%E3%83%BC%E3%83%AA%E3%83%BC%E3%81%AE%E5%AE%9F%E9%A8%93
936ご冗談でしょう?名無しさん:2011/05/23(月) 02:58:07.37 ID:YxBfIILc
話は変わりますが、
E. Nelson が語っている「Lagrange adjoint」とはなんですか?

今読んでいる彼の論文で、

ρ^{-1}(∂/∂t + b・▽ + ν△)^†ρ = -∂/∂t - b^*・▽ + ν△

という式の左辺の()の右上の「†(ダガー)」が、「Lagrange adjoint」
を表している、とあります。

英語の原文では、数式の直後に
「where the superscript † denotes the Lagrange adjoint
 (with respect to d^3x dt).」
と書かれています。
937ご冗談でしょう?名無しさん:2011/05/23(月) 04:09:33.71 ID:???
光速度不変の原理よりガリレイ変換
x'=x-vt
が間違いなのはわかりますが
x'=a(x-vt)
と置く理由がわかりません。
aは何を意味しているのでしょうか?
お願いします
938ご冗談でしょう?名無しさん:2011/05/23(月) 05:35:41.09 ID:???
939ご冗談でしょう?名無しさん:2011/05/23(月) 07:11:30.65 ID:x3gHCywC
>アホがうざいから物理板から出張。

とか、上から目線でニュー速にお前ら来てるけど
能無しの役立たずと判明したんだから自重してください。
わかりました?
940ご冗談でしょう?名無しさん:2011/05/23(月) 07:51:12.31 ID:???
股メコスジに恋してる
941ご冗談でしょう?名無しさん:2011/05/23(月) 23:25:17.32 ID:lUaBa4Ad
光電効果は金属に光を当てると電子が飛び出ると習ったけど
例えば1円玉(10円玉でも金属ならなんでも良い)に懐中電灯で光を照らせば
空気中に電子が飛び出てるの?
942ご冗談でしょう?名無しさん:2011/05/23(月) 23:32:03.09 ID:???
仕事関数以上のエネルギーの光子を当てれば飛び出す。
懐中電灯ではどんな金属でも無理じゃないか?
943ご冗談でしょう?名無しさん:2011/05/23(月) 23:52:36.52 ID:lUaBa4Ad
仕事関数とは?
可視光線では飛びださせるためのエネルギー不足ってこと?
944ご冗談でしょう?名無しさん:2011/05/23(月) 23:53:13.08 ID:lUaBa4Ad
もし仮に飛び出せばそれはベータ線という放射線のこと?
945ご冗談でしょう?名無しさん:2011/05/23(月) 23:57:12.52 ID:tInqZulV
日本で質量中心なんて言葉使う事あるんですか?教科書には括弧付きで書いてあるけど。
重心座標系はあっても質量中心座標系なんて絶対ないよね。
946ご冗談でしょう?名無しさん:2011/05/24(火) 00:06:46.33 ID:???
>>943
電子を金属から飛び出させるための最小のエネルギー。
光子1個と電子1個の相互作用だから光子のエネルギーがhν、仕事関数がWなら
飛び出した電子の運動エネルギーはK=hν-W>0じゃないといけない。
つまりν>W/hをみたす周波数の光じゃないといけない。

>>945
center of mass の訳語じゃないか?
947ご冗談でしょう?名無しさん:2011/05/24(火) 00:11:08.37 ID:???
電磁気と力学の統合ができるのはわかったが。
しかし光速が特別で、光速を基準にする説明はされていない。
なんで光速を特別視するのか。
948ご冗談でしょう?名無しさん:2011/05/24(火) 00:17:40.25 ID:???
特別視するっつうか光の振る舞いが普通の粒子や波とは違って
真空中ではどの慣性系から見ても同じ速さに見えるっていう
観測事実から理論を再構築した結果、光速cがいろんなところに
顔を出すのは当たり前だろ。別に基準にしてるわけじゃない。
949ご冗談でしょう?名無しさん:2011/05/24(火) 00:23:43.01 ID:???
うろ覚えだけど、ランダウ流に言えば情報の伝達速度には限りがあって、物質はある一定の速度を超える事が出来なくて、それがたまたま光速だってことだそうな。
限界速度は質量0粒子の速度に相当するけど光子は質量0なのでそれが光速と一致するという流れだろうか。
950ご冗談でしょう?名無しさん:2011/05/24(火) 00:36:16.80 ID:???
興味本位なんだが放物線運動のときの空気摩擦はどう計算するのだろう
水平方向は等速だからいいが垂直方向は時間で速度変わるよね

それともk(V-gt)とかで時間込みでもなりたつのか
951ご冗談でしょう?名無しさん:2011/05/24(火) 00:38:51.11 ID:???
>>950
もっとも簡単なのは速度の逆向きに抵抗力を受けると考えて運動方程式を解く
952ご冗談でしょう?名無しさん:2011/05/24(火) 00:51:32.06 ID:???
v'_vec = -g(0, 1) -{1/T}*v_vec
(d/dt + {1/T}) v_vec = -g(0, 1)
v_vec = ( C1*exp[-{1/T}t] , C2*exp[-{1/T}t] - gT)
C1 = v_x(t=0)
C2 = v_y(t=0) + gT
953ご冗談でしょう?名無しさん:2011/05/24(火) 00:53:38.69 ID:???
よくこんな式をタイプできるなあと感心するんだが、そういうソフトでもあんの?
954ご冗談でしょう?名無しさん:2011/05/24(火) 00:56:22.59 ID:???
>>953はWord使いか
955ご冗談でしょう?名無しさん:2011/05/24(火) 00:56:22.43 ID:???
人力。でもTeXとかエクセルとかでも似たようなのは手打ちするしそんなに苦労はないと思うけど。
956ご冗談でしょう?名無しさん:2011/05/24(火) 00:56:28.54 ID:???
>>944
"原子核"から電子が出てくればβ線。
これ豆な。
957ご冗談でしょう?名無しさん:2011/05/24(火) 00:58:46.06 ID:???
でも実際問題どれが何由来の電子かっていうのは悩ましいよね。
958ご冗談でしょう?名無しさん:2011/05/24(火) 01:07:28.25 ID:???
>>954
いやただの工房で物理も受験科目としてしかやってないんだが、
ソフトとかで計算するのはかっこいいなーと思う。大学に行けて時間出来たらマスターしたいな
959ご冗談でしょう?名無しさん:2011/05/24(火) 02:26:07.62 ID:???
>>953
普通の人はいきなりああは書けないでしょう
そういう仕事をしている人か学生さんか教師か
960 忍法帖【Lv=23,xxxPT】 :2011/05/24(火) 06:39:05.51 ID:???
>>956
全然違うよ、β線は中性子だよ。
それに原子核に電子は存在しないよ。
961 忍法帖【Lv=23,xxxPT】 :2011/05/24(火) 06:45:05.66 ID:???
>>957
電子分光法で光電子が何に由来してるかを調べるのは物性物理の基礎だけどね。
まぁ、ある意味悩ましいってのは合ってる。
962ご冗談でしょう?名無しさん:2011/05/24(火) 07:18:05.92 ID:???
>全然違うよ、β線は中性子だよ。
何言ってんだ?

>それに原子核に電子は存在しないよ。
誰もそんな主張してないが?原子核から電子が出てくることと
原子核に電子が存在することは違う。

こんなやつが
>物性物理の基礎
とか何の冗談だ
963 忍法帖【Lv=23,xxxPT】 :2011/05/24(火) 09:00:44.28 ID:???
>>962
ググったら俺が間違ってたw
すまぬ、中性子のβ崩壊によって放出された電子がβ線か、どっかで中性子線と記憶が混同していた。
素粒子屋じゃないから許しておくれ。
でももう一つのレスに関しては間違ってないよ。
964ご冗談でしょう?名無しさん:2011/05/24(火) 09:03:10.05 ID:???
> 素粒子屋じゃないから許しておくれ。

俺の頃は義務教育で習った記憶があるんだが。
素粒子屋とかいうレベルじゃねえ。
965 忍法帖【Lv=23,xxxPT】 :2011/05/24(火) 09:17:47.64 ID:???
>>964
だから記憶違いぐらい誰でもあるだろめんどくせぇやつだな。
義務教育以来使ってないから間違えたんだよ。
966ご冗談でしょう?名無しさん:2011/05/24(火) 09:19:54.10 ID:???
記憶違いというか常識レベル。
科学の好きな小学生に負ける。
967ご冗談でしょう?名無しさん:2011/05/24(火) 09:51:51.45 ID:QVyAYe1h
アルツハイマーでも
968ご冗談でしょう?名無しさん:2011/05/24(火) 09:52:06.36 ID:QVyAYe1h
わかる
969 忍法帖【Lv=23,xxxPT】 :2011/05/24(火) 10:03:29.08 ID:???
別に常識でも間違えることはあるだろうに。
右と左間違えたことはないのか?
970ご冗談でしょう?名無しさん:2011/05/24(火) 10:10:07.75 ID:???
怒ってる怒ってる
971ご冗談でしょう?名無しさん:2011/05/24(火) 11:10:53.82 ID:pkgAeOzF
v(t)=kt3/2で表せる球の体積があるんだがここから半径rの時刻tにおける増加速度を求めるのってv微分してけばでますか?
972ご冗談でしょう?名無しさん:2011/05/24(火) 11:18:35.51 ID:???
>>971
まず問題が分からないが半径の速度ならdr/dtじゃないか?
973ご冗談でしょう?名無しさん:2011/05/24(火) 11:34:41.92 ID:???
微小体積変化 = 球の表面積*微小半径変化
dV = 4πr^2dr

V = (3k/2)t より
dV = (3k/2)dt (kは時間に因らない数と見た)

よって
4πr^2dr = (3k/2)dt

t=0のときr=0として積分して整理
r^3 = (9k/8π)t
r = [(9k/8π)t]^(1/3)

dr/dt = [9k/8π]^(1/3)*[t^(-2/3)/3]

さて合ってるんですかねこれ。
974ご冗談でしょう?名無しさん:2011/05/24(火) 12:00:01.39 ID:???
r出すまでが正気の沙汰じゃない
975ご冗談でしょう?名無しさん:2011/05/24(火) 12:00:04.50 ID:pkgAeOzF
なるほど!積分か!ありがとうございます!
976ご冗談でしょう?名無しさん:2011/05/24(火) 12:16:38.23 ID:???
回折格子に光が当たって回折するときフーリエ変換図は
横軸が位置、縦軸が波長の振幅って感じでしょうか?



回折像におけるフーリエ変換がよく分かりません。

977ご冗談でしょう?名無しさん:2011/05/24(火) 12:41:40.20 ID:???
>>964
「溺れる犬を棒で叩く」
978ご冗談でしょう?名無しさん:2011/05/24(火) 12:51:49.35 ID:???
>>977
魯迅をパクった韓国のことわざだな
979ご冗談でしょう?名無しさん:2011/05/24(火) 13:49:02.59 ID:???
ヤングの実験難しいよ
高校の内容じゃないよあれ
980ご冗談でしょう?名無しさん:2011/05/24(火) 13:57:22.42 ID:???
コイルに電流を流してその中に磁石を入れるとコイルの中央付近で磁石が止まります
これはどのように説明できるでしょうか
どうぞよろしくお願いいたします
981ご冗談でしょう?名無しさん:2011/05/24(火) 14:01:08.86 ID:???
単振動
982ご冗談でしょう?名無しさん:2011/05/24(火) 14:32:05.35 ID:???
Wikipediaの軌道(力学)の項に離心率が1より大きい彗星は見つかっていないとあります。
しかし軌道離心率の項にはマックノート彗星は離心率が1より大きく太陽系から無限に離れて行くとあります。
どちらが正しいのですか?
983ご冗談でしょう?名無しさん:2011/05/24(火) 16:37:37.18 ID:???
次スレ立てます
984ご冗談でしょう?名無しさん:2011/05/24(火) 16:40:44.70 ID:???
次スレ立てました
■ちょっとした物理の質問はここに書いてね144■
http://kamome.2ch.net/test/read.cgi/sci/1306222766/
985ご冗談でしょう?名無しさん:2011/05/24(火) 20:28:33.25 ID:???
>>937
A=0に対応するものがB=0だとする。
そのとき、
A=Bとはやらないよね。
A=f*B (fは何らかの関数)

ここで慣性系Oに対してO'が速度vで動いてるとすると
x'=0 に対応しているのは x=vt
x'=0 に対応しているのは x-vt=0
だからx'=a(x-vt)
aというのは何らかの関数。
具体的にはaはvの関数。
986ご冗談でしょう?名無しさん:2011/05/24(火) 21:40:59.24 ID:???
>>979
音波はと一緒に振動・波動として扱ってるけど光学は全体的に高校レベルを逸脱してるよね
987ご冗談でしょう?名無しさん:2011/05/25(水) 00:52:41.26 ID:???
なんとなく気になって空気抵抗込みの鉛直投げあげを考えてみたんだが
ma=-mg-k(v+at)
a=-(mg+kv)/(m+kt)ってなって加速度に時間tが入っちゃってるんだけどこれっていいの?

低脳の俺にもわかるようにおしえてぷりーず
988ご冗談でしょう?名無しさん:2011/05/25(水) 00:55:10.79 ID:???
>>987
ぜんぜん違う
989ご冗談でしょう?名無しさん:2011/05/25(水) 01:03:04.98 ID:???
次元解析くらいしろよ高校生
990ご冗談でしょう?名無しさん:2011/05/25(水) 01:31:01.39 ID:???
a=Cexp[-kt/m]
991ご冗談でしょう?名無しさん:2011/05/25(水) 01:48:33.03 ID:???
次元は合ってるがね
992ご冗談でしょう?名無しさん:2011/05/25(水) 01:50:45.34 ID:???
ちょっと調べたら抵抗力はkv^2じゃねーか
雨粒についての問題の時はkvだったぞどういうこっちゃ
993ご冗談でしょう?名無しさん:2011/05/25(水) 01:52:41.90 ID:???
そういうモデルを考えているだけ
994ご冗談でしょう?名無しさん:2011/05/25(水) 01:53:30.13 ID:???
一生に一度くらい本気で調べてみろよ
995ご冗談でしょう?名無しさん:2011/05/25(水) 02:03:26.16 ID:???
雨粒の場合は空気中を落ちてるうちに抵抗と重力による加速がやがて釣り合って一定速度になって落ちてくるからかと

普通に空気中でボールを投げるような時にはむしろ、速度の二乗に比例するのだろう
996ご冗談でしょう?名無しさん:2011/05/25(水) 02:09:03.42 ID:???
慣性抵抗と粘性抵抗というのがあってだな
997ご冗談でしょう?名無しさん:2011/05/25(水) 02:14:46.60 ID:/KquAUHk
まえに速度の二乗に比例する空気抵抗のときの斜方投射を質問したら、解けないって結論だったけど、ほんとうに解けないのかな?
998ご冗談でしょう?名無しさん:2011/05/25(水) 02:16:07.79 ID:???
>>987
その場合の速度変化は時間に比例しないからv+atとは置けない。
999ご冗談でしょう?名無しさん:2011/05/25(水) 02:25:40.66 ID:???
>>997
解けるか解けないかなんてその問題の条件の与え方次第だと思うが
1000ご冗談でしょう?名無しさん:2011/05/25(水) 02:37:19.97 ID:???
一様重力場での斜方投射で慣性抵抗だけなら解ける気がする
10011001
このスレッドは1000を超えました。
もう書けないので、新しいスレッドを立ててくださいです。。。